MS1 Final- 2/2 Tem

¡Supera tus tareas y exámenes ahora con Quizwiz!

28. Which is a sign of acute angle closure glaucoma (AACG)? a. Large fixed pupil b. Nystagmus c. Bluish color in sclera d. Drooping eyelid

ANS: A Signs of AACG would be eye pain, large fixed pupil with reddened sclera, decreased vision, nausea, and vomiting. PTS: 1 DIF: Cognitive Level: Comprehension REF: Page 1867 OBJ: 9 TOP: Glaucoma KEY: Nursing Process Step: Assessment

4. What does a tympanoplasty correct? a. Conductive hearing loss b. Sensorineural hearing loss c. Congenital hearing loss d. Functional hearing loss

ANS: A Tympanoplasty can correct a conductive hearing loss. PTS: 1 DIF: Cognitive Level: Knowledge REF: Page 1889 OBJ: 17 TOP: Tympanoplasty KEY: Nursing Process Step: Implementation

6. Which assessment is of the greatest concern to a nurse when caring for a patient just admitted with a pelvic fracture? a. Pain level rating of 8 on a scale of 1 to 10 b. No urinary output for 8 hours c. Evidence of bruising along the patient's hips and buttocks d. Complaints of the need for back care from resting in bed

ANS: B The absence of urinary output could indicate a perforated bladder. DIF: Cognitive Level: Analysis REF: p. 987 OBJ: 3 TOP: Pelvic Fracture KEY: Nursing Process Step: Assessment

10. Myopia is a medical term meaning which visual disorder? a. Farsightedness b. Blurred vision c. Nearsightedness d. Halos around lights

ANS: C Myopia—nearsightedness. REF: Pages 1844, 1848, Table 53-3 TOP: Visual acuity

35. When performing CPR on an infant, the breastbone is depressed approximately to inch(es).

ANS: 0.5 to 1 one half to one The breastbone is depressed 0.5 to 1 inch when doing CPR on an infant. REF: Page 745

34. A burn patient is brought into the emergency department with the following burns: half of the front torso, entire left arm, front of left leg. The nurse should record that the patient has a % burn.

ANS: 27 twenty-seven Half of the front torso = 9, entire left arm = 9, front of the left leg = 9 REF: Page 759, Figure 24-16 TOP: Rule of Nines

32. When two nurses perform two-person CPR, there should be slow breaths for every compressions.

ANS: 2; 30 two; thirty Two slow breaths are given after every 30 compressions. REF: Page 743 TOP: Two-person CPR

38. is a medical term for blurred vision.

ANS: Astigmatism Astigmatism is a defect in the curvature of the eyeball surface, resulting in blurred vision. REF: Page 1847, Table 53-3 TOP: Visual acuity

43. Arrange the parts of the eye from the exterior to the most interior. (Separate letters by a comma and space as follows: A, B, C, D) a. Choroid b. Cornea c. Aqueous humor d. Retina e. Lens f. Iris

ANS: B, C, F, E, D, A The cornea is the outermost, followed by the aqueous humor, iris, lens, retina, and the choroid. PTS: 1 DIF: Cognitive Level: Application REF: Page 1840, Figure 52-1 OBJ: 2 TOP: Eye structure KEY: Nursing Process Step: Assessment

44. Place the nursing intervention in appropriate order for the immediate care of a patient with a penetrating wound of the eye. (Separate letters by a comma and space as follows: A, B, C, D) a. Assess eye, do not remove object b. Cover both eyes with an eye shield or cup c. Lay the patient down flat d. Check for the irregularity of the pupil e. Obtain medical attention immediately

ANS: C, A, D, B, E The patient should be placed on his back to prevent loss of the aqueous humor, assessment of the eye for the location of the object and whether the pupil is regular, cover the eye to prevent movement, and obtain medical attention immediately. PTS: 1 DIF: Cognitive Level: Analysis REF: Page 1870, Safety Alert OBJ: 10 TOP: Penetrating wound of the eye KEY: Nursing Process Step: Implementation

39. therapy is performed to alleviate or decrease uncomfortable symptoms without curing the problem.

ANS: Palliative Palliative therapy is designed to relieve or reduce intensity of uncomfortable symptoms without cure. REF: Page 1254, Table 42-1 TOP: Palliative therapy

33. The acronym RICE directs the nurse in the care of a sprain. The "C" in the acronym stands for .

ANS: compression The acronym stands for Rest, Ice, Compression, Elevation. REF: Page 760 TOP: Sprain Step: Application

38. The home health patient complains of tearing and a feeling of dryness in the right eye. The nurse assesses that the eyelid is turned inward and the sclera is red. The nurse documents the presence of a(n) .

ANS: entropion An entropion is the abnormal turning in of the eyelid, causing irritation and tearing of the eye. PTS: 1 DIF: Cognitive Level: Application REF: Page 1859 OBJ: 8 TOP: Entropion KEY: Nursing Process Step: Assessment

40. The total removal of an eye is a(n) .

ANS: enucleation The surgical removal of the eyeball is an enucleation. PTS: 1 DIF: Cognitive Level: Knowledge REF: Page 1869 OBJ: 9 TOP: Enucleation KEY: Nursing Process Step: Assessment

39. The nurse explains that a pneumatic retinopexy is a repair of a retinal detachment using a bubble of to put pressure on the damaged retina.

ANS: gas A pneumatic retinopexy uses a bubble of gas to put pressure on the damaged retina. PTS: 1 DIF: Cognitive Level: Knowledge REF: Page 1866 OBJ: 11 TOP: Pneumatic retinopexy KEY: Nursing Process Step: Implementation

31. If a spinal injury is suspected, before the rescuer starts CPR, the trachea should be opened with a maneuver.

ANS: jaw thrust The jaw thrust maneuver does not hyperextend the neck. REF: Page 739

35. The most deadly skin cancer is .

ANS: melanoma Malignant melanoma is a cancerous neoplasm that invades the epidermis, dermis, and sometimes the subcutaneous tissue. REF: Page 1328 TOP: Tumors of the skin

41. The surgical incision into the eardrum with either a knife or a heated wire loop to relieve pressure in the middle ear is a(n) .

ANS: myringotomy The opening of the eardrum with a specialized knife or a heated wire loop to relieve pressure in the middle ear is a myringotomy. PTS: 1 DIF: Cognitive Level: Knowledge REF: Page 1883 OBJ: 17 TOP: Myringotomy KEY: Nursing Process Step: Assessment

36. Decreasing visual acuity is a sign of .

ANS: normal aging As an individual ages, the crystalline lens of the eye hardens and becomes too large for the eye muscles, thus causing a loss of accommodation, which often results in a need for bifocals or trifocals. REF: Pages 1842-1843, Life Span Considerations box TOP: Normal aging

42. Progressive deafness caused by the ankylosis of the stapes is the condition of .

ANS: otosclerosis Progressive deafness related to the ankylosis of the stapes is diagnosed as otosclerosis. PTS: 1 DIF: Cognitive Level: Knowledge REF: Page 1885 OBJ: 16 TOP: Otosclerosis KEY: Nursing Process Step: Assessment

40. Discharge planning for a surgical procedure begins in the period and continues through the period.

ANS: preoperative, recuperative Discharge planning for a surgical procedure begins in the preoperative and continues through the recuperative period. REF: Page 1291 TOP: Nursing diagnosis

34. The three major glands of the skin are , , and .

ANS: sudoriferous (sweat), ceruminous, and sebaceous (oil). sudoriferous, ceruminous, sebaceous sweat, ceruminous, oil Sudoriferous glands—sweat glands open into pores on the skin surface and excrete sweat. Ceruminous glands—secrete a waxlike substance called cerumen and are located in the external ear canal. Sebaceous glands—secrete their substance, sebum (an oily secretion), through the hair follicles distributed on the body. REF: Page 1297 TOP: Glands of the skin

33. A nurse clarifies that a postmenopausal woman who is not taking hormone replacement therapy should take ____ mg elemental calcium on a daily basis.

ANS: 1500 Women who are not taking hormone replacements need calcium, 1500 mg/day. DIF: Cognitive Level: Knowledge REF: p. 961 OBJ: 8 TOP: Osteoporosis KEY: Nursing Process Step: Implementation MSC: NCLEX: Physiological Integrity:

28. When assessing the capillary refill, a nurse should document as normal a refill time of __________ seconds.

ANS: 3 Capillary refill is a method of quick assessment of perfusion to the extremities. A normal capillary refill time is 3 to 5 seconds or less. DIF: Cognitive Level: Comprehension REF: p. 1189 OBJ: 7 TOP: Fluid Shift in Burn Patient KEY: Nursing Process Step: Assessment

30. Arrange the process of stair climbing with crutches in the correct sequence: (Separate letters by a comma and space as follows: A, B, C, D.) A. Body weight is supported with crutches. B. Crutches are moved to the next step. C. The affected leg moves to the next step. D. The unaffected leg is moved to the next step. E. Body weight is transferred to the unaffected leg.

ANS: A, D, E, B, C When climbing steps on crutches, the body weight is supported with the crutches. While the unaffected leg is moved to the next step, the body weight is transferred to the unaffected leg; while the crutches are moved up, the affected leg moves up. DIF: Cognitive Level: Comprehension REF: p. 982 OBJ: 5 TOP: Stair Climbing with Crutches KEY: Nursing Process Step: N/A

30. A nurse uses a diagram to show the physiologic sequence of hearing. After entering the external ear, the sound is then conducted through the (Arrange the options in sequence. Separate letters by a comma and space as follows: A, B, C, D.) A. tympanic membrane B. sensory receptors C. oval window D. acoustic nerve to the brain E. malleus, incus, and stapes

ANS: A, E, C, B, D The sound impulse, after entering the external ear, is conducted through the tympanic membrane; into the malleus, incus, and stapes; through the oval window; into the sensory receptors in the inner ear; and then through the acoustic nerve to the brain. DIF: Cognitive Level: Comprehension REF: p. 1252 OBJ: 1 TOP: Physiology of Hearing KEY: Nursing Process Step: Implementation MSC:

29. A nurse uses a diagram to show the process of a fractured bone healing. (Arrange the options in the appropriate sequence. Separate letters by a comma and space as follows: A, B, C, D.) A. Ossification B. Hematoma C. Fibrocartilage D. Consolidation E. Callus

ANS: B, C, E, A, D The sequence of healing is hematoma, fibrocartilage, callus, ossification, and consolidation. DIF: Cognitive Level: Comprehension REF: p. 972 OBJ: 2 TOP: Bone Healing KEY: Nursing Process Step: Implementation

29. A nurse collecting tissue for a Tzanck smear should (Select the appropriate interventions and place the steps in sequence. Separate letters by a comma and space as follows: A, B, C, D.) A. Open the lesion with a hypodermic needle. B. Place the specimen in a culture tube and take it to the laboratory. C. Saturate the sterile swab with exudates. D. Wash the lesion. E. Place a pressure dressing on the lesion.

ANS: D, A, C, B The nurse washes the lesion, punctures the lesion with a needle, saturates a sterile cotton swab, places the swab in a culture tube, and takes the collected tissue to the laboratory. A pressure dressing is not needed. DIF: Cognitive Level: Application REF: p. 1187 OBJ: 3 TOP: Capillary Refill KEY: Nursing Process Step: Implementation

27. A nurse is alert for the expected fluid shift in the patient who was burned 24 hours earlier. (Place the events in the appropriate sequence. Separate letters by a comma and space as follows: A, B, C, D.) a. Fluid volume deficit occurs. b. Blood is shunted from the kidneys to compensate for a loss of fluid volume. c. Urine output decreases. d. Generalized edema occurs. e. Hypoproteinemia causes fluid to move from the bloodstream to extracellular space

ANS: E, D, A, B, C Hypoproteinemia causes a fluid shift from the bloodstream to extracellular space, causing generalized edema; fluid volume deficit occurs; blood is shunted from the kidneys and gastrointestinal tract to make up for the fluid loss in the circulating volume; and urine output is decreased. DIF: Cognitive Level: Comprehension REF: p. 1189 OBJ: 4 TOP: Tzanck Smear KEY: Nursing Process Step: Assessment

29. When planning care for a patient who cannot perceive or interpret sounds, a nurse takes into consideration that the patient may have a(n) _____ hearing loss.

ANS: central The inability to perceive or interpret sounds is referred to as a central hearing loss. DIF: Cognitive Level: Comprehension REF: p. 1261 OBJ: 5 TOP: Central Hearing Loss KEY: Nursing Process Step: Planning MSC:

30. A nurse clarifies that the precise term for the patient's amputation, which will be through the knee joint, is called _____.

ANS: disarticulation Disarticulation is the appropriate term for an amputation through the knee joint. DIF: Cognitive Level: Knowledge REF: p. 990 OBJ: 2 TOP: Disarticulation KEY: Nursing Process Step: Implementation MSC:

31. To decrease osteoporosis, a nurse explains that women can benefit from _____ for 15 years after the onset of menopause.

ANS: estrogen A program of oral estrogen replacement therapy can decrease the occurrence of osteoporosis. DIF: Cognitive Level: Comprehension REF: p. 961 OBJ: 5 TOP: Hormone Replacement Therapy (HRT) KEY: Nursing Process Step: Implementation MSC: NCLEX: Health Promotion and Maintenance:

28. A fracture that occurs because of osteoporosis is classified as a(n) _____ fracture.

ANS: pathological A fracture that occurs as a result of a tumor of another pathologic condition is classified as a pathologic fracture. DIF: Cognitive Level: Knowledge REF: p. 971 OBJ: 3 TOP: Pathological Fracture KEY: Nursing Process Step: N/A

32. To reduce the possibility of hip contractures in a patient with an above-the-knee amputation, a nurse periodically places the patient in a(n) _____ position.

ANS: prone The prone position will cause the muscles of the thigh to stretch and prevent contracture. DIF: Cognitive Level: Application REF: p. 999 OBJ: 4 TOP: Nursing Care to Prevent Contractures KEY: Nursing Process Step: Implementation MSC:

31. An amputation of a gangrenous limb that is left open for 10 days before closure is classified as a(n) _____ amputation.

ANS: staged guillotine A staged or guillotine amputation is one in which the wound is left open until the area is free of infection or evidence of gangrene. DIF: Cognitive Level: Knowledge REF: p. 993 OBJ: 1 TOP: Staged Amputation KEY: Nursing Process Step:

33. Preoperative exercises for a patient undergoing a lower-extremity amputation include _____ training.

ANS: upper body Upper body training will strengthen the arms to aid in movement after the loss of a lower extremity. DIF: Cognitive Level: Knowledge REF: p. 999 OBJ: 4 TOP: Upper Body Training KEY: Nursing Process Step:

32. A nurse assesses ischemic spots around the nail beds of a patient with rheumatoid arthritis and recognizes that these are a complication of medical diagnosis, rheumatoid arthritis, related to _____.

ANS: vasculitis Vasculitis occurs when the vessels become inflamed and cause ischemia and necrosis. DIF: Cognitive Level: Comprehension REF: p. 957 OBJ: 6 TOP: Vasculitis KEY: Nursing Process Step: Assessment MSC: NCLEX: Health Promotion and Maintenance:

15. When other methods have failed to stop the bleeding and the victim's life is in danger, the rescuer at the scene applies a tourniquet to a young woman's leg above the knee. It is essential that the rescuer: a. never release the tourniquet. b. wrap the tourniquet around the limb twice. c. mark the patient with a "T." d. leave the limb elevated.

ANS: A A tourniquet must never be released once it is in place. All other options are enhancements to the procedure of the tourniquet application, but not essential. REF: Page 750, Skill 24-1

14. What does diabetes retinopathy result from? a. Capillaries in retina hemorrhage b. Long-term overdosing of insulin c. Retinal detachment d. Aging

ANS: A Retinopathy is caused when the capillaries in the retina have aneurysms or hemorrhage. PTS: 1 DIF: Cognitive Level: Comprehension REF: Page 1861 OBJ: 9 TOP: Glaucoma KEY: Nursing Process Step: Assessment

8. Which of these terms describes a client who is in shock resulting from vasodilation and abnormal fluid distribution within the circulatory system? a. distributive b. cardiogenic c. hypovolemic d. obstructive

ANS: A Shock, a condition of profound hemodynamic and metabolic disturbance characterized by inadequate tissue perfusion and inadequate circulation to the vital organs, must be recognized and immediately treated. Distributive shock results from vasodilation and abnormal fluid distribution within the circulatory system.

22. Which early postoperative observation is abnormal and should be reported immediately? a. Emesis that is red b. Complaint of feeling cold c. Nausea d. Complaint of pain

ANS: A Any emesis that is red should be reported immediately. REF: Page 1282, Box 42-7 TOP: Assessment

15. Assistive devices such as canes, crutches, and walkers are used for people who need to limit weight-bearing activities on joints. Which statement by a nurse best illustrates an understanding of the appropriate use of these devices? a. "Canes provide minimal support and balance and are carried on the unaffected side." b. "When using a cane, slide it as you go to decrease the arm strain." c. "A three-point gait is used when walking with a walker." d. "When using crutches, the unaffected leg goes down the steps first."

ANS: A "Canes provide minimal support and balance and are carried on the unaffected side" is the only true statement. DIF: Cognitive Level: Comprehension REF: p. 982-983 OBJ: 5 TOP: Assistive Devices KEY: Nursing Process Step: Evaluation

18. A patient, age 76, is partially blind. His physician has diagnosed open-angle glaucoma. The goal of treatment in glaucoma is to a. decrease aqueous humor. b. increase aqueous humor. c. decrease discomfort. d. restore vision.

ANS: A A beta-blocker, such as Betoptic, will reduce intraocular pressure. Miotics such as pilocarpine constrict the pupil and draw the iris away from the cornea, allowing aqueous humor to drain out of the canal of Schlemm. REF: Page 1861, Table 53-5 TOP: Glaucoma Step: Planning

22. Which diagnostic test result should the nurse expect for a patient with polymyositis? a. Muscle biopsy positive for muscle degeneration b. Positive antinuclear antibody (ANA) blood test result c. Positive 24-hour urine test result for urate crystals d. Urate crystals in the synovial fluid

ANS: A A biopsy, positive for muscle degeneration, is the only result that pertains to polymyositis. DIF: Cognitive Level: Comprehension REF: p. 967 OBJ: 4 TOP: Polymyositis KEY: Nursing Process Step: Assessment MSC: NCLEX: Health Promotion and Maintenance:

9. Why is a closed amputation usually performed? a. To create a weight-bearing residual limb b. To alleviate the effects of trauma c. To allow infection to heal and drain d. To treat a limb with gangrene

ANS: A A closed amputation is performed to create a weight-bearing limb. DIF: Cognitive Level: Comprehension REF: p. 992 OBJ: 3 TOP: Surgical Treatment KEY: Nursing Process Step:

8. The priority nursing responsibility while caring for a patient with vertigo is a. safety. b. comfort. c. hygiene. d. quiet.

ANS: A A nursing diagnosis of Risk for injury related to disturbed sensory perception is the most important diagnosis for a patient with vertigo. REF: Pages 1875-1876, Patient Teaching boxes, NCP 53-2 TOP: Vertigo

10. What controls the movement with a myoelectrically controlled prosthesis? a. Patient's muscle movement and the prosthesis b. Battery-operated muscles implanted in the prosthesis c. Motion-sensing mechanism that swings the prosthesis forward d. Internal computer chip in the prosthesis

ANS: A A patient's muscle movement and the prosthesis control movement with a myoelectrically controlled prosthesis. DIF: Cognitive Level: Knowledge REF: p. 993 OBJ: 3 TOP: Prostheses KEY: Nursing Process Step: Implementation MSC:

11. Most of the deaths from burn trauma in the emergent phase that require a referral to a burn center result from a. infection. b. dysrhythmias with cardiac arrest. c. hypovolemic shock and renal failure. d. adrenal failure.

ANS: A A possible complication that should be addressed at the time of the burn is infection in the wound. REF: Page 1334, Figure 43-24 TOP: Burns Step: Evaluation

4. When making an initial assessment on a patient with a hearing deficit, the patient reports that he often feels off balance and is dizzy when he stands up. Which diagnosis might explain these symptoms? a. Sinus infection b. Rubella c. Otalgia d. Presbycusis

ANS: A A sinus infection can be an acute cause of hearing deficits and can create problems with balance. DIF: Cognitive Level: Comprehension REF: p. 1252 OBJ: 5 TOP: Hearing Assessment: Medical History KEY: Nursing Process Step: Assessment MSC:

10. A patient, age 65, underwent a right hemicolectomy. On postoperative day 4, her surgical wound dehisced. This means that a. there is partial or complete wound separation. b. there has been inadequate wound closure. c. abdominal viscera protrude through the walls. d. the wound will not heal well when it is resutured.

ANS: A A surgical incision may separate; this action of dehiscence (the separation of a surgical incision or rupture of a wound closure) may occur within 3 days to over 2 weeks postoperatively. REF: Page 1284, Figure 42-15 TOP: Postoperative complications

15. A slow-growing and usually benign tumor of the vestibular portion of the inner ear is known as: a. acoustic neuroma b. Ménière's disease c. otitis media d. otosclerosis

ANS: A Acoustic neuroma is a slow-growing and usually benign tumor of the vestibular portion of the inner ear. The main symptoms are dizziness, tinnitus, and hearing loss.

3. A patient who has osteomyelitis after multiple fractures inquires what the physician meant when he said that surgery would follow the antibiotic therapy. What is the nurse's most helpful reply to explain why this surgery will be performed? a. To remove dead bone b. To close the open draining wound c. To close the area with casting material d. To amputate

ANS: A After the antibiotic has controlled the infection in the bone, surgery will be performed to remove the dead bone. DIF: Cognitive Level: Comprehension REF: p. 973 OBJ: 4 TOP: Osteomyelitis KEY: Nursing Process Step: Implementation

5. The anesthesiologist provides anesthesia by inhalation and IV administration routes. a. general b. regional c. specific d. preoperative

ANS: A An anesthesiologist gives general anesthetics by IV and inhalation routes through four stages of anesthesia. REF: Page 1273 TOP: Anesthesia

4. The client is seeing a specialist who evaluates hearing and determines the extent and types of hearing loss. The nurse explains the client will be seeing: a. an audiologist b. a hearing-aid specialist c. an ophthalmologist d. an otolaryngologist

ANS: A An audiologist is a specialist who evaluates hearing and determines the extent and types of hearing loss.

9. A male patient, age 80, has had a total hip replacement. Anxiety, hypotension, and jarring during transfer from the recovery room to his room can cause a postoperative increase in which of his vital signs? a. Pulse rate b. Temperature c. Blood pressure d. Pain

ANS: A An increase in pulse rate is an objective, detectable sign that the body is responding to "pain." Other objective changes include a decrease in blood pressure in the immediate postoperative period, restlessness, diaphoresis, and pallor. REF: Page 1284, Box 42-8 TOP: Postoperative complications

16. The nurse teaches a patient with epistaxis that the best way to control bleeding is to: a. place ice on the nose and pinch the nostrils. b. maintain a flat position. c. pack nostrils with cotton. d. lean backward.

ANS: A Apply steady pressure to both nostrils while applying ice to the nose is the best way to attempt to control the bleeding of epistaxis. REF: Page 750

2. A nurse explains to a patient with rheumatoid arthritis that the drug leflunomide (Arava) is a disease-modifying antirheumatic drug (DMARD). What is the action of this medication? a. Retards the progress of the disease b. Builds new bone c. Decreases inflammation d. Increases flexibility

ANS: A Arava is a DMARD and slows the progression of the disease. DIF: Cognitive Level: Knowledge REF: p. 958 OBJ: 5 TOP: DMARDs KEY: Nursing Process Step: Implementation MSC: NCLEX: Physiological Integrity:

20. A nurse is caring for a patient immediately after total knee replacement surgery. What assessment requires priority? a. Quality of pulses in the affected limb b. Degree of nausea and vomiting c. Understanding of the procedure d. Amount of pain

ANS: A Assessments related to postoperative circulatory efficiency are priority assessments. DIF: Cognitive Level: Application REF: p. 954 OBJ: 6 TOP: Postoperative Care of Total Knee Replacement KEY: Nursing Process Step: Assessment MSC: NCLEX: Physiological Integrity:

15. Astigmatism is a medical term meaning which visual disorder? a. Blurred vision b. Inability to detect colors c. Color blindness d. Farsightedness

ANS: A Astigmatism—blurred vision. REF: Page 1847, Table 53-2, Table 53-3 TOP: Visual acuity

29. In addition to medication management of systemic lupus erythematosus, which supportive therapy would help control the disease? a. Balanced diet and balance between rest and activity b. Increase in activity to promote mobility c. Exposure to the sun to increase vitamin D absorption d. Increased sodium consumption

ANS: A Balanced rest, activity, and diet will support medication management. Limited sunlight exposure is recommended to prevent photosensitivity. SLE often has kidney involvement, which would require reduction of sodium. REF: Page 1265 TOP: Systemic lupus erythematosus Step: Planning

12. A patient, age 26, is admitted to the burn unit with partial- and full-thickness burns to 20% of his body surface area as well as smoke-inhalation injury. Carbon monoxide intoxication secondary to smoke inhalation is often fatal because carbon monoxide a. binds with hemoglobin in place of oxygen. b. interferes with oxygen intake. c. is a respiratory depressant. d. is a toxic agent.

ANS: A Carbon monoxide poisoning is likely if the patient has been in an enclosed area. Carbon monoxide displaces oxygen by binding with the hemoglobin. REF: Page 1333 TOP: Burns Step: Evaluation

29. Why is otitis media found more frequently in children 6 to 36 months? a. Eustachian tubes in children are shorter and straighter. b. Infection descends via the eustachian tube to the throat. c. Children's eustachian tubes are more vertical and longer. d. Otitis media is seen equally in both children and adults.

ANS: A Children's shorter and straighter eustachian tubes provide easier access of the organisms from the nasopharynx to travel to the middle ear. PTS: 1 DIF: Cognitive Level: Analysis REF: Page 1881 OBJ: 16 TOP: Otitis media KEY: Nursing Process Step: Evaluation

29. Otitis media is more frequently seen in children 6 to 36 months because a. eustachian tubes in children are shorter and straighter. b. infection descends via the eustachian tube to the throat. c. children's eustachian tubes are more vertical and longer. d. otitis media is seen equally in both children and adults.

ANS: A Children's shorter and straighter eustachian tubes provide easier access of the organisms from the nasopharynx to travel to the middle ear. REF: Page 1872 TOP: Otitis media Step: Evaluation

14. Rapid onset of decreased vision, halos around lights, and severe eye pain are indications of a. closed-angle glaucoma. b. open-angle glaucoma. c. retinal detachment. d. diabetic retinopathy.

ANS: A Closed-angle glaucoma causes rapid vision loss and dramatic symptoms. Closed-angle (acute) glaucoma produces severe pain, decreased vision, and nausea and vomiting. The patient sees colored halos around lights. REF: Page 1860 TOP: Glaucoma

4. A patient with a crushed forearm cannot get pain relief with opioid medications. The injury is swollen, cool, and cyanotic, with weak distal pulses. What should the nurse suspect? a. Compartment syndrome b. Overwhelming infection c. Fat embolus d. Osteomyelitis

ANS: A Compartment syndrome may occur after a massive injury or an inappropriately tight cast. The tissues become swollen to the point that they cut off their own circulation. DIF: Cognitive Level: Analysis REF: p. 974 OBJ: 3 TOP: Compartment Syndrome KEY: Nursing Process Step: Assessment

28. The most common cause of congenital hearing loss from birth or early infancy is: a. Anoxia or trauma b. Tumor c. Infection d. Occasional loud noise

ANS: A Congenital hearing loss is present from birth or early infancy. Anoxia or trauma during delivery may be causes. DIF: Cognitive Level: AnalysisREF: Page 1869, Health Promotion box OBJ: 19 TOP: Hearing loss Step: Evaluation

8. What do connective tissue diseases affect? a. Bones, ligaments, cartilage, and tendons b. Bones, ligaments, and tendons c. Spurs, ligaments, cartilage, and tendons d. Tendons, cartilage, and tophi

ANS: A Connective tissue diseases affect bones, ligaments, cartilage, and tendons. DIF: Cognitive Level: Knowledge REF: p. 939-940 OBJ: 1 TOP: Connective Tissue Disease KEY: Nursing Process Step: Assessment MSC: NCLEX: Physiological Integrity:

8. A 45-year-old patient has had a repair of a cerebral aneurysm and is presenting signs of increased intracranial pressure (ICP). Which postoperative nursing interventions would be contraindicated? a. Coughing every 2 hours b. Leg exercises every 2 hours c. Monitoring intravenous therapy at 50 ml/hr d. Assessing vital signs every 2 hours

ANS: A Coughing increases ICP. REF: Page 1268, Box 42-6 TOP: Postoperative complications

15. A duodenal ulcer may occur 8 to 14 days after severe burns. Usually, the first symptom is bright red emesis. Which condition matches this description? a. Curling's ulcer b. Paralytic ileus c. Hypoglycemia d. ICU psychosis syndrome

ANS: A Curling's ulcer is a duodenal ulcer that develops 8 to 14 days after severe burns on the surface of the body. The first sign is usually vomiting of bright red blood. REF: Page 1334 TOP: Curling's ulcer

22. Which assessment is considered abnormal when a nurse performs a neurovascular assessment on a patient in skeletal traction? a. Delayed capillary refill b. Bilateral equal pulses c. Absence of pain and swelling d. Limb is the same color as the unaffected side

ANS: A Delayed capillary refill reflects possible impaired tissue perfusion. DIF: Cognitive Level: Comprehension REF: p. 979-980 OBJ: 5 | 6 TOP: Traction KEY: Nursing Process Step: Assessment

12. A family member of an older patient with severe dermatitis says, "I was always so careful to bathe him every day. I guess I just wasn't careful enough." What is the best response by the nurse? a. "Dermatitis is not caused by poor hygiene." b. "Don't worry; we will bathe him thoroughly while he is here." c. "You will have a chance to do better when he is back at home." d. "You shouldn't feel like the skin condition is your fault."

ANS: A Dermatitis is not a condition of poor hygiene. Implying that the family member is responsible for the condition is belittling and not therapeutic. DIF: Cognitive Level: Application REF: p. 1195 OBJ: 8 TOP: Dermatitis KEY: Nursing Process Step: Implementation

7. What is a common mistake that hinders communication when communicating with the hearing impaired? a. Overaccentuating words b. Facing the patient when speaking c. Speaking in conversational tones d. Speaking into the ear with the hearing aid

ANS: A Do not overaccentuate words. Speak in a normal tone; do not shout or raise the pitch of voice. PTS: 1 DIF: Cognitive Level: Analysis REF: Page 1870, Health Promotion OBJ: 14 TOP: Communication KEY: Nursing Process Step: Implementation

7. A hearing-impaired patient is having problems communicating with staff members. Which behaviors would continue to hinder communication? a. Overaccentuating words b. Facing the patient when speaking c. Speaking in conversational tones d. Speaking into the affected ear

ANS: A Do not overaccentuate words. Speak in a normal tone; do not shout or raise the pitch of voice. REF: Page 1869, Health Promotion box

17. What should a nurse who is documenting and reporting the signs and symptoms of an infection underneath a cast include in the medical record? a. Elevated temperature b. Tingling and decreased sensation c. Full pulses and absence of pain d. Swelling and diminished motor function

ANS: A Elevated temperature on the affected extremity may be a symptom of an infection under the cast. DIF: Cognitive Level: Application REF: p. 973 OBJ: 3 TOP: Complications of a Fracture KEY: Nursing Process Step: Assessment

25. Which action should a nurse implement to diminish swelling of the residual limb in the postoperative period after an above-the-knee amputation? a. Elevate the foot of the bed on blocks. b. Elevate the residual limb on pillows. c. Elevate the head of the bed 15 degrees. d. Turn the patient on the affected side.

ANS: A Elevating the foot of the bed on blocks will help diminish edema and will not cause hip contracture deformity. DIF: Cognitive Level: Application REF: p. 999 OBJ: 4 TOP: Prevention of Postoperative Edema KEY: Nursing Process Step: Implementation MSC:

28. A patient is a 32-year-old woman whose mother recently died from malignant melanoma. She asks the nurse about what she can do to prevent the development of malignant melanoma in herself and her children. The best response by the nurse includes which information regarding risk factors for melanoma? a. Avoid exposure to the sun and use protective measures when exposure occurs. b. Have all nevi removed. c. Watch for changes in moles, especially on the back. d. There is nothing that prevents malignant melanoma, but it is curable if detected early.

ANS: A Encourage the patient to protect skin from the sun by wearing protective clothing including a hat with 4-inch brim, applying sunscreen all over the body, and avoiding the midday sun from 10 am to 4 pm. DIF: Cognitive Level: ApplicationREF: Page 1328, NCP 43-2 OBJ: 12 TOP: Melanoma

11. A patient is on postoperative day 2 after a nephrectomy. The nurse is aware that the most effective way to increase her peristalsis is a. ambulation. b. an enema. c. encouraging hot liquids. d. administering a laxative.

ANS: A Encouraging activity (turning every 2 hours, early ambulation) assists GI activity. REF: Page 1288, Box 42-10 TOP: Postoperative complications

2. A patient, age 63, has cancer of the left breast. After a modified radical mastectomy, she has been receiving chemotherapy. Her grandson, who visited a few days ago, now has varicella (chickenpox). The nurse should observe her carefully for signs of a. herpes zoster. b. herpes simplex type I. c. herpes simplex type II. d. impetigo.

ANS: A Herpes zoster is caused by the same virus that causes chickenpox (Herpes varicella). The greatest risk occurs to patients who have a lowered resistance to infection, such as those on chemotherapy or large doses of prednisone, in whom the disease could be fatal because of the patient's compromised immune system. REF: Pages 1308, 1310 TOP: Chemotherapy

18. A patient, age 20, is admitted with severe eczema. In planning the care for her, the nurse should plan to a. keep the skin well hydrated. b. change the dressings every day. c. keep the skin clean and dry. d. administer antibiotics for the infection.

ANS: A Hydration of the skin is the key to treatment. REF: Page 1318 TOP: Eczema Step: Planning

26. A client arrives in the emergency department undergoing CPR after being rescued from submersion in cold water. The nurse should recognize that CPR should be continued until the client's: a. core temperature reaches 94 degrees Fahrenheit and cardiac status is evaluated b. electrocardioversion can be attempted c. family member can authorize "do not resuscitate" orders d. reason for being in cold water is determined

ANS: A Hypothermia can occur from overexposure to very cold weather or from prolonged submersion in cold water. Rewarming is essential to resuscitation. Gradual warming is instituted with warm blankets, warmed oxygen, and warmed IV fluids.

7. A patient has been receiving penicillin, acetaminophen with codeine, and hydrochlorothiazide for 4 days. He now has a urinary tract infection. A sulfonamide has been prescribed to be taken three times per day. Several hours after the second dose , he complains of pruritus. The nurse observes a generalized erythema and rash. The most appropriate nursing intervention would be to hold a. all medications, and notify the physician of the signs and symptoms. b. the penicillin. c. the acetaminophen with codeine. d. the sulfonamide.

ANS: A If a patient develops wheals or hives in an allergic reaction to drugs (urticaria), then all medications should be held. REF: Page 1316 TOP: Medication

5. Sjögren's syndrome is associated with which eye disorder? a. Keratoconjunctivitis sicca b. Conjunctivitis c. Blepharitis d. Opaque lens disorder

ANS: A If the patient with keratoconjunctivitis sicca has associated dry mouth, the patient has Sjögren's syndrome (an immunologic disorder characterized by deficient fluid production by the lacrimal, salivary, and other glands, resulting in abnormal dryness of the mouth, eyes, and other mucous membranes). Complaints of dry eye are caused by a variety of ocular disorders characterized by decreased tear secretion or increased tear film evaporation. REF: Pages 1851-1852 TOP: Dry eye disorders

10. A patient, age 37, sustained partial- and full-thickness burns to 26% of her body surface area. The greatest fluid loss resulting from her burns will usually occur a. within 12 hours after burn trauma. b. 24 to 36 hours after burn trauma. c. 24 to 48 hours after burn trauma. d. 48 to 72 hours after burn trauma.

ANS: A In a burn injury, usually the greatest fluid loss occurs within the first 12 hours. REF: Pages 1330-1331 TOP: Burns Step: Planning

8. Which action should a nurse implement to make a patient with atopic dermatitis more comfortable? a. Instruct the patient to wear loose clothing. b. Add alcohol to the bath water. c. Provide a diet low in fat. d. Increase the room temperature between 78° F and 80° F.

ANS: A Loose clothing and a cool atmosphere allow the skin to stay cool and reduce sweating. Alcohol is drying to the skin. DIF: Cognitive Level: Application REF: p. 1195 OBJ: 7 TOP: Atopic Dermatitis KEY: Nursing Process Step: Implementation

15. Which decreased laboratory value indicates an increased risk of pressure ulcers and decreased wound healing? a. albumin b. elevated fasting blood glucose c. c-reactive protein (CRP) d. sodium

ANS: A Low protein levels are results of long term protein deficiency. Decreased albumin increases risk of pressure ulcers and decreases wound healing.

2. What is the simplest method of triaging clients in an emergency department (ED)? a. basic life-support principles of airway, breathing, and circulation b. Maslow's hierarchy of physiologic, safety, love, self-esteem, and self-actualization needs c. holistic perspective of physiologic, psychological, sociocultural, intellectual, and spiritual aspects d. health continuum from wellness to illness

ANS: A Many hospitals use the American Heart Association's basic life-support principles of airway, breathing, and circulation (ABCs). Not all clients who use the ED require immediate care.

12. Morphine sulfate is the drug of choice for clients experiencing a cardiopulmonary emergency because it: a. decreases both pain and anxiety b. is a vasodilator and a bronchodilator and provides pain relief c. relieves the severe pain usually associated with this type of emergency d. helps restore cardiovascular function

ANS: A Morphine sulfate assists the client with a cardiopulmonary emergency to decrease both pain and the stress associated with this condition.

30. On the patient's return to the medical-surgical unit, the nurse performs an abdominal assessment. To assess bowel sounds, the nurse auscultates the lower abdomen for a. 1 minute. b. 5 to 20 seconds. c. as long as it takes to hear a bowel sound. d. one full inspiration and expiration.

ANS: A Normal peristalsis is gauged by hearing 5 to 30 gurgles per minute. REF: Page 1288, Box 42-10 TOP: Assessment

8. When a client is first given a hearing aid, the nurse should make sure the client and the family understand that hearing aids: a. increase the volume of sound and speech, but not necessarily the clarity b. improve the quality of sounds heard, but not their volume c. stimulate the client's auditory centers d. can determine the amount of amplification actually required by the client

ANS: A Nursing interventions for a client with a hearing aid include teaching the client and the family that a hearing aid will assist with an increase in the volume of sound and speech, but not necessarily the clarity.

18. Treatment of clients who have any type of eye trauma should include: a. antibiotic eye medication b. application of a warm saline dressing c. patching of the affected eye d. patching of the opposite eye

ANS: A Ocular emergencies include foreign bodies in the eye, objects impaled in the eye, an eyeball torn out of its socket, and retinal detachment. An ophthalmologist should be consulted immediately. As a prophylactic measure, all eye traumas are treated with an antibiotic eye medication.

22. A young woman being admitted to the clinic service states that all the members of her family have been hard of hearing. She says her hearing loss became more pronounced when she was pregnant. What term explains this type of hearing loss? a. Otosclerosis b. Ototoxicity c. Otalgia d. Otitis media

ANS: A Otosclerosis is hereditary, develops in young women, and worsens with pregnancy. DIF: Cognitive Level: Comprehension REF: p. 1265 OBJ: 5 TOP: Otosclerosis KEY: Nursing Process Step: Assessment MSC:

6. A nurse is screening patients that the plastic surgeon is considering for phototherapy. Which patient should the nurse exclude? a. A 34-year-old woman with lupus erythematosus b. A 5-year-old child with pneumonia c. A 60-year-old man with a pacemaker d. A 23-year-old woman who is 3 months' pregnant

ANS: A Persons with lupus erythematosus should avoid exposure to UV light. DIF: Cognitive Level: Comprehension REF: p. 1190 OBJ: 6 TOP: Phototherapy KEY: Nursing Process Step: Assessment

6. A home health care nurse is visiting a patient after a total hip replacement. What should the nurse include when teaching the patient how to protect the new joint? a. Put an extension on the toilet seat. b. Keep the legs crossed when at rest. c. Frequently change positions from side to side. d. Slowly pull the knee to the chest twice a day to stretch the hip abductors.

ANS: A Placing an extender on the toilet seat will assist in the objective of not flexing the hip more than 90 degrees. Crossing the legs adducts the hip, which is contraindicated. DIF: Cognitive Level: Comprehension REF: p. 955 OBJ: 8 TOP: Total Hip Replacement and Nursing Implementations KEY: Nursing Process Step: Implementation MSC: NCLEX: Physiological Integrity:

22. What should the nurse include in the plan of care following a tympanoplasty? a. Elevating head of bed with operative side facing upward b. Enforcing bed rest for 72 hours c. Frequent turning, coughing, and deep breathing d. Continuous irrigation of the ear canal with antibiotic solutions

ANS: A Postoperative management for patients who have had a tympanoplasty consists of bed rest until the next morning. The head of the bed is elevated 40 degrees, and the operative side faces upward. PTS: 1 DIF: Cognitive Level: Analysis REF: Page 1889 OBJ: 17 TOP: Otitis media KEY: Nursing Process Step: Implementation

22. The patient, age 42, has chronic otitis media. Otoscopic examination reveals a central perforation of the eardrum with purulent drainage into the ear canal. A CT scan confirms the presence of a cholesteatoma in the middle ear, and he is scheduled for a tympanoplasty. Which postoperative activity does the nurse teach him about preoperatively? a. Elevation of head of bed with operative side facing upward b. Enforce bed rest for 72 hours c. Frequent turning, coughing, and deep breathing d. Continuous irrigation of the ear canal with antibiotic solutions

ANS: A Postoperative management for patients who have had a tympanoplasty consists of bed rest until the next morning. The head of the bed is elevated 40 degrees, and the operative side faces upward. REF: Page 1880, Nursing Diagnoses box TOP: Otitis media

1. A patient has generalized macular-papular skin eruptions and complains of severe pruritus from contact dermatitis. When the nurse administers his therapeutic bath, it is important to remember that a. using Burow's solution helps promote healing. b. rubbing the skin briskly decreases pruritus. c. allowing 20 to 60 minutes to complete the bath will prevent pruritus. d. sterilizing all equipment used will prevent pruritus.

ANS: A Pruritus is responsible for most of the discomfort. Wet dressings, using Burow's solution, help promote the healing process. A cool environment with increased humidity decreases the pruritus. REF: Page 1315 TOP: Pruritus

5. What routine preoperative diagnostic tests are appropriate for a patient anticipating a below-the-knee amputation? a. Pulse volume recording and white blood cell (WBC) count b. Cardiac catheterization and WBC count c. Pulse volume recording and radiographic images d. Thermography and cardiac catheterization

ANS: A Pulse volume recording and WBC count are two diagnostic tests for patients anticipating a below-the-knee amputation. DIF: Cognitive Level: Comprehension REF: p. 991 OBJ: 4 TOP: Diagnostic Tests and Procedures KEY: Nursing Process Step: Implementation MSC:

30. Cellulitis is a potentially serious infection. In adults Staphylococcus aureus can cause which complications? a. Sepsis and meningitis b. Appendicitis and vasculitis c. Tinea pedis and tinea corporis d. Urticaria and psoriasis

ANS: A Sepsis, meningitis, and lymphangitis are the most common complications of cellulitis for adults caused by Staphylococcus aureus bacteria. Tinea pedis/corporis are fungal infections. Urticaria and psoriasis are noninfectious skin disorders. REF: Page 1311 TOP: Bacterial disorders of the skin Step: Evaluation

9. The initial focus of the nursing management of clients who are in shock is to: a. identify the type of shock b. initiate appropriate interventions quickly c. discuss client status with the family d. administer medications as ordered

ANS: A Shock, a condition of profound hemodynamic and metabolic disturbance characterized by inadequate tissue perfusion and inadequate circulation to the vital organs, must be recognized and immediately treated. The initial focus of the nursing management is to identify the type of shock so interventions can be started without delay.

1. A client has a stasis ulcer on the left ankle. Nursing assessment of this open area involves which skin assessment parameter? a. integrity b. moisture c. turgor d. vascularity

ANS: A Skin should be intact (integrity) with no evidence of tissue injury; the nurse should note any broken skin or open areas, and the color, size, and location of any rashes or lesions such as nodules or pustules.

5. The 62-year-old home health patient who is recovering from eye surgery complains of a feeling of "grittiness" in the eye and is having blurred vision. The eyes are reddened and have stringy mucus. What do these complaints indicate? a. Sjögren syndrome b. Early cataracts c. Macular degeneration d. Retinal detachment

ANS: A The Sjögren syndrome of "dry eye" frequently appears after eye surgery. There is insufficient production of tears. Excessive use of antihistamines, antidepressants, and decongestants may cause this syndrome to appear. PTS: 1 DIF: Cognitive Level: Application REF: Page 1858 OBJ: 8 TOP: Sjögren syndrome KEY: Nursing Process Step: Assessment

8. What is the process when the lens of the eye changes its curvature to focus on the retina? a. Accommodation b. Constriction c. Convergence d. Refraction

ANS: A The ability of the lens to alter its curvature as it focuses on the retina is accommodation. PTS: 1 DIF: Cognitive Level: Knowledge REF: Page 1849 OBJ: 16 TOP: Accommodation KEY: Nursing Process Step: Implementation

7. An older woman falls down at church and immediately complains of severe pain in her left hip. Which observation is recognized as the cardinal sign of a fractured hip? a. Shortened left leg compared with the right b. Downward curled toes c. Internal rotation of the left leg d. Hematoma on the left hip

ANS: A The classic sign of a fractured hip is a shortened limb on the affected side, with an externally rotated limb. DIF: Cognitive Level: Comprehension REF: p. 985 OBJ: 1 TOP: Fractured Hip KEY: Nursing Process Step: Assessment

11. Which instruction should a nurse include when providing patient teaching information for a patient who will be self-administering ear drops for an ear infection? a. Tip the affected ear up and keep it in that position for several minutes after instilling the medication. b. Keep the medication in the refrigerator to preserve it. Instill the medication with the affected ear tilted upward. c. Touch the dropper to the opening of the ear canal to ensure that the drops are correctly instilled. d. Warm the ear drops and then tilt the head downward.

ANS: A The head is kept in an upward position to ensure that the drops penetrate deep into the external ear. DIF: Cognitive Level: Application REF: p. 1256-1257 OBJ: 4 TOP: Ear Drops KEY: Nursing Process Step: Implementation MSC:

17. A female client comes to the emergency department saying she was sexually assaulted in her apartment. The nurse's focus in this situation should be to: a. provide accurate, methodical care to the client. b. notify the client's family of the attack. c. refer the client for psychological counseling. d. administer antipregnancy medication when prescribed.

ANS: A The nurse should provide accurate, methodical care to the client in a nonjudgmental manner. Accurate documentation of the forensic evidence is needed to protect the rights of the victim.

31. Which preoperative fear is linked to postoperative behavior? a. Fear of anesthesia and death b. Fear of death and malnutrition c. Fear of unknown and lack of respect d. Fear of malnutrition and addiction to new medications

ANS: A The preoperative anxiety level influences the amount of anesthesia required, the amount of postoperative pain medication needed, and the speed of recovery from surgery. DIF: Cognitive Level: AssessmentREF: Pages 1256, 1260, Box 42-4 TOP: Nursing diagnosis

30. Why would the nurse encourage a group of teenagers to protect their eyes with dark sunglasses while using a UV lamp? a. The lamp can cause cataracts. b. The lamp can cause presbycusis. c. The lamp can cause keratitis. d. The lamp can cause ectropion.

ANS: A The proteins in the lens of the eye are vulnerable to UV light and can develop cataracts. PTS: 1 DIF: Cognitive Level: Comprehension REF: Page 1890 OBJ: 9 TOP: Health promotion KEY: Nursing Process Step: Implementation

15. When the patient in the emergency room complains of seeing flashing lights and a curtain down over his right eye, the nurse recognizes this as a symptom of which condition? a. Detached retina b. Macular degeneration c. Early sign of cataract d. Diabetic retinopathy

ANS: A The standard complaint of a detached retina is the report of seeing flashing lights and having a curtain being drawn over the eyes. PTS: 1 DIF: Cognitive Level: Knowledge REF: Page 1865 OBJ: 9 TOP: Detached retina KEY: Nursing Process Step: Assessment

5. The nurse determines clinical death and initiates CPR immediately because resuscitation is possible if cardiopulmonary arrest has existed for no more than: a. 2 minutes. b. 3 minutes. c. 4 minutes. d. 5 minutes.

ANS: C CPR can reverse clinical death if initiated before 4 minutes. REF: Page 740

17. A nurse is organizing a teaching plan for a patient with gout. What should the nurse caution this patient he is at an increased risk for? a. Kidney stones b. Tophi c. Visual disturbances d. Facial lesions

ANS: A The threat of kidney stones is a lifelong problem for patients with gout. Tophi are symptomatic of the disease but are not a complication. Facial lesions and visual disturbances are noncontributory. DIF: Cognitive Level: Comprehension REF: p. 964 OBJ: 8 TOP: Gout Complications KEY: Nursing Process Step: Planning MSC: NCLEX: Health Promotion and Maintenance:

5. An older patient with osteoarthritis complains of stomach discomfort and shortness of breath after years of taking aspirin for pain relief. What change in pain control medication would be most appropriate for the home health care nurse to suggest? a. Nonsteroidal antiinflammatory drugs (NSAIDs) b. Oral corticosteroids c. Mild exercise d. Warm baths

ANS: A The use of NSAIDs is less irritating than aspirin or glucocorticoids. Mild exercise is good but not for pain relief. DIF: Cognitive Level: Application REF: p. 951 OBJ: 5 TOP: Drug Therapy for Connective Tissue Disorders KEY: Nursing Process Step: Implementation MSC: NCLEX: Physiological Integrity:

18. What should the nurse remind the hearing aid wearer to do when the nurse hears a whistling hearing aid? a. Reinsert the ear mold b. Change the battery c. Recharge the hearing aid d. Wash the ear mold with warm water

ANS: A The whistling hearing aid is usually caused by a poor fit of the ear mold. Reinsertion of the ear mold usually stops the whistling. PTS: 1 DIF: Cognitive Level: Comprehension REF: Page 1880, Box 52-3 OBJ: 13 TOP: Hearing aid KEY: Nursing Process Step: Implementation

26. A patient will have an incision in the lower left abdomen. Which intervention by the nurse will help decrease discomfort in the incisional area when she coughs postoperatively? a. Apply a splint directly over the lower abdomen. b. Keep the patient flat with feet flexed. c. Turn her on her right side. d. Apply a splint above and below the incision.

ANS: A To ease the pressure on the incision, the nurse helps the patient support the surgical site with a pillow, rolled bath blanket, or the heel of the hand. REF: Pages 1267-1268, Skill 42-4 Step 10, NCP 42-1 TOP: Postoperative nursing interventions

10. Which is an appropriate implementation for a patient with severe psoriasis who has a nursing diagnosis of "Disturbed body image, related to skin lesions"? a. Touching the patient often b. Reassuring the patient of a quick remission c. Reminding the patient to bathe often d. Promptly administering medications as needed

ANS: A To touch, interact, and care attentively for a disfigured patient communicates acceptance. DIF: Cognitive Level: Application REF: p. 1199 OBJ: 8 TOP: Psoriasis KEY: Nursing Process Step: Implementation

27. A patient developed a severe contact dermatitis of her hands, arms, and lower legs after spending an afternoon picking strawberries. She states that the itching is severe and she cannot keep from scratching. Which instruction by the nurse will be most helpful in managing the pruritus? a. Use cool, wet dressings and baths to promote vasoconstriction. b. Trim the fingernails short to prevent skin damage from scratching. c. Expose the areas to the sun to promote drying and healing of the lesions. d. Wear cotton gloves and cover all other affected areas with clothing to prevent environmental irritation.

ANS: A Wet dressings and using Burrow's solution help promote the healing process. Cold compresses may be applied to decrease circulation to the area (vasoconstriction). REF: Page 1315 TOP: Contact dermatitis

9. A nurse assessing the results of a Rinne test sees the notation of BC > AC. How should the nurse translate this result? a. Conductive hearing loss b. Sensorineural hearing loss c. Normal hearing d. Cochlear defect

ANS: A When the bone conduction (BC) is greater than the air conduction (AC), the results of the Rinne test will read, BC > AC, which means the patient has a conductive hearing loss. The normal finding for the Rinne test is that AC is greater than BC (AC > BC). DIF: Cognitive Level: Analysis REF: p. 1255 OBJ: 2 TOP: Rinne Test KEY: Nursing Process Step: Assessment MSC:

27. What is a closed amputation designed to do? (Select all that apply.) a. Prepare a weight-bearing limb. b. Cover the stump with tissue and muscle. c. Place sutures immediately over the bone. d. Be staged to closure. e. Be immediately ready for a prosthesis.

ANS: A, B Closed amputations are meant to prepare the limb for weight-bearing activities with tissue and muscle applied to the residual limb. Sutures are not placed over the bone for future comfort and better healing. Closed amputations are not staged because are open amputations, and they may or may not be prepared for an immediate prosthesis. DIF: Cognitive Level: Comprehension REF: p. 992 OBJ: 2 TOP: Closed Amputation KEY: Nursing Process Step: Implementation MSC:

30. What signs of progressive systemic sclerosis does the anonym CREST represent? (Select all that apply.) a. Calcinosis b. Rash c. Esophageal dysfunction d. Sore joints e. Telangiectasis

ANS: A, C, E CREST stands for calcinosis, Raynaud phenomenon, esophageal dysfunction, sclerodactyly, and telangiectasis. DIF: Cognitive Level: Knowledge REF: p. 966-967 OBJ: 6 TOP: CREST KEY: Nursing Process Step:

40. Nursing interventions and patient teaching for the treatment of head lice and scabies include: (Select all that apply.) a. clothing, linens, and bath articles thoroughly cleaned in hot water. b. stress nature and transmission of the disease. c. special carbohydrate diet to promote healing. d. complete isolation from the public.

ANS: A, B Identify involved contacts while stressing importance of preventing transmission of disease. Washable and cloth items should be cleaned in hot water to prevent reinfection. No special diet is required. Isolation is not necessary once medical management is completed. REF: Pages 1325-1326 TOP: Parasitic diseases of the skin

34. What factors must the nurse consider when assessing readiness to learn when teaching health promotion practices for the visually and hearing impaired? (Select all that apply.) a. Cultural beliefs b. Values c. Habits d. Income e. Occupation

ANS: A, B, C The nurse also assesses readiness to learn and teaches health promotion practices (see Patient Teaching boxes). The nurse must consider the patient's culture, beliefs, values, and habits (Cultural Considerations 13-2), as well as the special needs of the older adult (Older Adult Considerations 13-1). REF: Page 1882 TOP: Health promotion

2. Which of the following would be appropriate nursing diagnoses for a client with Ménière disease? a. Activity Intolerance due to vertigo b. Knowledge related to abrupt onset of disease c. Risk for Injury related to vertigo d. Anticipatory Grieving related to diminished quality of life

ANS: A, B, C Nursing Diagnoses that are appropriate for the client with Ménière disease include: Activity Intolerance due to vertigo, Knowledge related to abrupt onset of disease, and Risk for Injury related to vertigo. Anticipatory Grieving related to diminished quality of life is more appropriate for the client with an acoustic neuroma.

33. Which may contribute to otitis media? (Select all that apply.) a. Exposure to cigarette smoke b. Allergies c. Upper respiratory infections d. Swimming e. Trauma f. Prolonged exposure to loud noise

ANS: A, B, C Otitis media is usually caused by an upper respiratory infection with gram-negative bacteria, such as Proteus, Klebsiella, and Pseudomonas. In addition, allergy, exposure to cigarette smoke, mycoplasma, and several viruses may be factors. PTS: 1 DIF: Cognitive Level: Comprehension REF: Page 1881 OBJ: 16 TOP: Otitis media KEY: Nursing Process Step: Assessment

33. Which may contribute to otitis media? (Select all that apply.) a. Exposure to cigarette smoke b. Allergies c. Upper respiratory infections d. Lower respiratory infections e. Trauma f. Prolonged exposure to loud noise

ANS: A, B, C Otitis media is usually caused by gram-negative bacteria, such as Proteus, Klebsiella, and Pseudomonas. In addition, allergy, exposure to cigarette smoke, mycoplasma, and several viruses may be factors. REF: Page 1872 TOP: Otitis media

34. What factors must the nurse consider when assessing readiness to learn when teaching health promotion practices for the visually and hearing impaired? (Select all that apply.) a. Cultural beliefs b. Values c. Habits d. Income e. Occupation

ANS: A, B, C The nurse must consider the patient's culture, beliefs, values, and habits, as well as the special needs of the older adult. PTS: 1 DIF: Cognitive Level: Knowledge REF: Page 1891 OBJ: N/A TOP: Health promotion KEY: Nursing Process Step: Assessment

28. What should a nurse consider when selecting possible nursing diagnoses for a 32-year-old patient who is in anticipatory grieving for an upcoming bilateral above-the-knee amputation? (Select all that apply.) a. Anxiety, related to knowledge deficit of procedure b. Disturbed body image, related to loss of body part c. Sexual dysfunction, related to perceived disfigurement d. Disturbed self-image, related to loss of independence e. Activity intolerance, related to bed rest

ANS: A, B, C, D All concerns mentioned except the bed rest would need to be addressed. DIF: Cognitive Level: Application REF: p. 997 OBJ: 5 TOP: Preoperative Nursing Diagnosis KEY: Nursing Process Step: Planning MSC:

27. A nurse questions an older patient about the age-related changes she has experienced in her connective tissue, which have lessened her mobility. What do these changes most commonly include? (Select all that apply.) a. Loss of bone, which may cause fragile bones b. Thickening of the tendons, causing loss of strength c. Bony deposits in the joints, causing pain and altered movement d. Hardening of cartilage, causing more friction in joints e. Diminished energy, causing decreased activity

ANS: A, B, C, D Reduced energy, although observed in older adults, is not caused by a change in connective tissue. DIF: Cognitive Level: Comprehension REF: p. 940 OBJ: 7 TOP: Age-Related Changes in Connective Tissue KEY: Nursing Process Step: Assessment MSC: NCLEX: Physiological Integrity:

26. Which major situational occurrences might lead to amputations? (Select all that apply.) a. Trauma b. Disease c. Tumors d. Congenital defects e. Carelessness

ANS: A, B, C, D The categories of occurrences that lead to amputation are trauma, disease, tumors, and congenital defects. Carelessness frequently leads to trauma. DIF: Cognitive Level: Knowledge REF: p. 990 OBJ: 1 TOP: Causes for Amputation KEY: Nursing Process Step: Implementation MSC:

37. Major functions of the skin include: (Select all that apply.) a. excretion of wastes. b. protection. c. vitamin D synthesis. d. temperature regulation. e. prevention of dehydration.

ANS: A, B, C, D, E Functions of the skin include protection from the environment (pathogenic organisms, foreign substances, natural barrier against infection), temperature regulation, prevention of dehydration, excretion of waste products, and vitamin D synthesis. REF: Page 1295, Box 43-1 TOP: Functions of the skin

35. What are the major senses? (Select all that apply.) a. Taste b. Touch c. Smell d. Sight e. Hearing/Balance

ANS: A, B, C, D, E The five major senses are taste, touch, smell, sight, and hearing/balance. REF: Page 1838 TOP: Health promotion

2. A nurse is participating in a skin cancer screening clinic at a local hospital. Which descriptors should be used when the nurse is documenting her observations of a potential skin lesion? (Select all that apply.) a. type b. color c. size d. location e. amount and color or drainage f. odor of drainage

ANS: A, B, C, D, E, F Any skin lesions should be identified according to type and described regarding color, size, and location. Describe the amount, color, odor, and appearance of any drainage that might be present. The nurse documents assessment findings clearly, concisely, and completely.

28. What are the goals of therapy for patients with rheumatic arthritis? (Select all that apply.) a. Decrease inflammation. b. Balance activity and rest. c. Promote adaptation to limitations. d. Plan frequent periods of bed rest. e. Supply patient education and support.

ANS: A, B, C, E Bed rest of any long period increases the problems of immobility. DIF: Cognitive Level: Comprehension REF: p. 958-960 OBJ: 6 TOP: Goals for Therapy for Rheumatoid Arthritis KEY: Nursing Process Step: Implementation MSC: NCLEX: Health Promotion and Maintenance:

36. A postoperative patient who had a left inguinal hernia repair is ready for his discharge instructions. Which information should the nurse provide? (Select all that apply.) a. Care of the wound site and any dressings b. When he may operate a motor vehicle c. Signs and symptoms to report to the physician d. Call the physician's office once he arrives home e. Report bowel movements to the physician f. Actions and side effects of any medications

ANS: A, B, C, F As the day of discharge approaches, the nurse should be certain that the patient has vital information. REF: Page 1292, Box 42-13 TOP: Discharge instructions Step: Planning

32. Select all the conditions that may cause conductive hearing loss. (Select all that apply.) a. Buildup of cerumen b. Foreign bodies c. Otosclerosis of external auditory canal d. Trauma e. Exposure to ototoxic drugs f. Otitis media with effusion

ANS: A, B, C, F Common causes of conductive hearing loss are buildup of cerumen and otitis media with effusion (escape of effusion). Other conditions that may result in conductive hearing loss are foreign bodies, otosclerosis, and stenosis of the external auditory canal. Sensorineural hearing loss is usually due to trauma, infectious processes, or exposure to ototoxic drugs. PTS: 1 DIF: Cognitive Level: Knowledge REF: Page 1878 OBJ: 12 TOP: Hearing loss KEY: Nursing Process Step: Assessment

32. Select all the conditions that may cause conductive hearing loss. a. Buildup of cerumen b. Foreign bodies c. Otosclerosis of external auditory canal d. Trauma e. Exposure to ototoxic drugs f. Otitis media with effusion

ANS: A, B, C, F Common causes of conductive hearing loss are buildup of cerumen and otitis media with effusion (escape of effusion). Other conditions that may result in conductive hearing loss are foreign bodies, otosclerosis, and stenosis of the external auditory canal. Sensorineural hearing loss is usually due to trauma, infectious processes, or exposure to ototoxic drugs. REF: Page 1869 TOP: Hearing loss

26. Which conditions can be improved with negative pressure therapy? (Select all that apply.) a. Pressure ulcers b. Skin grafts c. Burns d. Dehisced surgical wounds e. Eczema

ANS: A, B, D All ulcers, skin grafts, and dehisced wounds respond well to negative pressure therapy. DIF: Cognitive Level: Comprehension REF: p. 1190 OBJ: 6 TOP: Negative Pressure Therapy KEY: Nursing Process Step: N/A

25. Which age-related skin changes should a nurse anticipate when performing a physical assessment on an 80-year-old man? (Select all that apply.) a. Increased nasal hair b. Flattened nails c. Small macular lesions at the hairline d. Increased hair on the helix of the ear e. Presence of seborrheic keratosis

ANS: A, B, D, E Increased hair in the nostrils and ear, flattened discolored nails, and seborrheic keratosis are common age-related skin changes. Macular lesions are abnormal. DIF: Cognitive Level: Knowledge REF: p. 1184 OBJ: 2 TOP: Age-Related Skin Changes KEY: Nursing Process Step: Assessment

29. How should a nurse modify a teaching plan for an older adult who has had an above-the-knee amputation? (Select all that apply.) a. Offer smaller units of information at a time. b. Increase time for learning. c. Place less emphasis on chronic health problems. d. Clarify the reality of phantom pain. e. Include frequent repetition.

ANS: A, B, D, E Older adults are quite capable of learning, but the amount of information offered at one time should be lessened, the time for learning should be increased, the probability of phantom pain should be clarified, and repetition should be frequently used. DIF: Cognitive Level: Application REF: p. 1000-1001 OBJ: 4 TOP: Teaching the Older Adult KEY: Nursing Process Step: Planning MSC:

26. Which patient behaviors should alert a nurse to a possible hearing deficit? (Select all that apply.) a. Watches the speaker's mouth b. Gives inappropriate answers to questions c. Pulls at the ears d. Fails to respond when spoken to e. Turns the good ear to the speaker

ANS: A, B, D, E Pulling at the ear is not a signal for hearing loss; all of the other options are. DIF: Cognitive Level: Comprehension REF: p. 1261 OBJ: 1 TOP: Behavioral Cues to Hearing Deficit KEY: Nursing Process Step: Assessment MSC:

1. A nurse has been asked to speak to a new group of nurses during orientation about the ways that the nurse can assist the hearing impaired client. Which of the following should the nurse include in her presentation? (Select all that apply.) a. Speak slowly after getting the client's attention. b. Face the client and sit or stand at eye level. c. Move in close to the client within 6-12 inches of the client's face. d. Use short simple sentences and give the client time to respond. e. Speak in a higher pitched voice. f. Use written materials such as a notepad and pen to assist communication. g. If the client has a hearing aid, make sure it is in client's ear and battery is functional.

ANS: A, B, D, F, G Strategies to assist the hearing-impaired client include: speak slowly and distinctly after getting the client's attention, face the client and sit or stand to be at eye level with the client, use short, simple sentences and give the client time to respond, use a lower pitched voice to speak to the client because high frequency sounds are lost first, use written materials when possible to communicate information, keep a notepad and pen or pencil available to write down new or unfamiliar words and concepts, decrease competing noises, such as a television or radio, when communicating with the client, if sign language is the client's preferred method of communication, locate a person who understands sign language, and if the client wears a hearing aid, make sure that the battery is functional, it is turned on, and is adjusted to a comfortable level.

39. A common diagnostic criterion for identifying systemic lupus erythematosus (SLE) is: (Select all that apply.) a. butterfly rash over nose and cheeks b. photosensitivity. c. severe abdominal pain. d. skin ulcers. e. polyarthralgias and polyarthritis. f. immobility.

ANS: A, B, E Butterfly rash on face, sensitivity to sunlight, polyarthralgias and polyarthritis are some of the main criteria leading to the diagnosis of SLE. REF: Pages 1322-1323, Box 43-2 TOP: Systemic lupus erythematosus

28. A patient complains that his hearing aid is not working. What actions should a nurse implement to assess the device? (Select all that apply.) a. Check to see if the device is turned on. b. Clean the earpiece and remove cerumen clogged in the vent. c. Open the earpiece to see if the microphone wire is connected. d. Examine the interior of the earpiece for water. e. Validate that the battery is correctly placed.

ANS: A, B, E Cleaning the earpiece to remove clogged cerumen and checking the device to see if it is turned on and if the battery is placed correctly are all good options. The earpiece should not be opened. If the hearing aid is still not working, it should be evaluated by the dealer. DIF: Cognitive Level: Application REF: p. 1258 OBJ: 4 TOP: Hearing Aids KEY: Nursing Process Step: Implementation MSC:

27. Which characteristics are present when crutches are properly fitted? (Select all that apply.) a. The axilla piece is 3 to 4 fingerbreadths below the axilla. b. They fit close to the axilla for secure support. c. They are measured and adjusted when the patient is in the tripod position. d. Adjusted handgrips allow for a 45-degree flexion of the elbow. e. They are padded so patient can bear weight on the axilla piece when ambulating.

ANS: A, C Crutches should allow for 3 to 4 fingerbreadths between the axilla and the axilla piece, the crutches should be adjusted when the patient is in the tripod position, the elbow flexion should be adjusted for a 30-degree flexion, and the weight should not be borne on the axilla because of the possibility of nerve damage. DIF: Cognitive Level: Comprehension REF: p. 980 OBJ: 5 TOP: Crutch Adjustment KEY: Nursing Process Step: N/A

38. In preparing the patient for abdominal surgery, the Assistive Personnel (AP) can perform which interventions? (Select all that apply.) a. Vital signs b. Insertion of N/G tube c. Enema d. Height and weight e. Obtain operative consent f. Sterile gowning

ANS: A, C, D The AP can perform vital signs, enema, and height and weight. REF: Page 1274 TOP: Nursing diagnosis

2. A client has absorbed more than .80 GY and has developed acute radiation syndrome. Which body systems could be affected by this dose of radiation? (Select all that apply.) a. Hematopoietic b. Urinary c. Gastrointestinal d. Cerebrovascular/CNS e. Integumentary

ANS: A, C, D, E The client who absorbs more than 0.75 Gy can develop acute radiation syndrome (ARS). Symptoms of ARS depend on the dose of radiation and include: Hematopoietic-deficiency of WBCs and platelets leading to bleeding, anemia, infections, impaired wound healing, and immunodeficiency; Gastrointestinal-loss of mucosal barrier and cells lining intestines leading to fluid and electrolyte loss, vomiting, diarrhea, loss of normal flora, and sepsis; Cerebrovascular/CNS-cerebral edema, hyperpyrexia, hypotension, confusion, and disorientation; Skin-loss of epidermis and possibly dermis. Signs and symptoms in one or more of these areas appear immediately after exposure.

1. The basic principles of first aid developed for emergency medicine are part of the triage and include which of the following? (Select all that apply.) a. Assess physical environment for self-protections. b. Notify family for treatment consent. c. Remove the client from danger. d. Obtain and maintain an open airway. e. Assess breathing. f. Use Glasgow Coma Scale. g. Obtain arterial blood gases.

ANS: A, C, D, E, F The basic principles of first aid developed for emergency medicine are part of the triage and include assess physical environment for self-protections, remove the client from danger, obtain and maintain an open airway and assess breathing and circulation, and determine disability through the use of the Glasgow Coma Scale.

35. Which of the following are causes of cataracts? (Select all that apply.) a. Long-term use of corticosteroids b. Hypotension c. Congenital from exposure to maternal rubella d. Diabetes mellitus e. Exposure to sand and dust f. Smoking

ANS: A, C, D, F Among the many causes of cataracts are long-term corticosteroid use, maternal rubella, diabetes mellitus, and smoking. PTS: 1 DIF: Cognitive Level: Application REF: Page 1859 OBJ: 9 TOP: Cataracts KEY: Nursing Process Step: Assessment

27. Which common characteristics might a patient with conductive hearing loss display? (Select all that apply.) a. Hears adequately in noisy settings b. Hears sounds but has difficulty understanding speech c. Has improved hearing with hearing aids d. Has a history of diabetes mellitus e. Speaks in a normal volume

ANS: A, C, E Persons with conductive hearing loss can hear in a noisy setting and can have improved hearing with the use of hearing aids. Persons with conductive hearing loss speak at a normal or soft volume because they can hear themselves. Muffled sounds and a history of diabetes would be associated with sensorineural hearing loss. DIF: Cognitive Level: Comprehension REF: p. 1260-1261 OBJ: 5 TOP: Common Characteristics in Persons with Conductive Hearing Loss KEY: Nursing Process Step: Assessment MSC:

1. Which of the following are effects of aging on the skin? (Select all that apply.) a. skin vascularity and the number of sweat and sebaceous glands decrease, affecting thermoregulation b. inflammatory response and pain perception increases c. thinning epidermis and prolonged wound healing make the elderly more prone to injury and skin infections d. skin cancer is less common in the elderly e. increased rate of growth of nails f. decreased number of hair follicles resulting in the thinning of hair g. loss of subcutaneous tissue causes skin sagging and wrinkling

ANS: A, C, F, G Aging has several effects on the skin. These include changes in skin vascularity and the number of sweat and sebaceous glands decrease, affecting thermoregulation. The inflammatory response and pain perception diminish, increasing the risk of adverse effects from noxious stimuli. A thinning epidermis and prolonged wound healing make the elderly more prone to injury and skin infections. Skin cancer is more common among the elderly. Decreased blood flow to the skin causes dryness of the skin. The use of skin lotions containing alcohol can cause drying of the skin, increasing the risk of injury. Moisture-enhancing products should be used instead. Clients will also experience a decreased rate of growth of nails with longitudinal ridges and thickening of the nail (especially toenails). There is a decreased number of hair follicles, therefore, a thinning of the hair.

37. Two considerations for the older adult surgical patient include (Select the two that apply.) a. pre- and postoperative teaching. b. lower morbidity and mortality. c. quick assessment skills. d. surgery causes much physiological stress.

ANS: A, D Surgery places greater stress on older than on younger patients. Teaching should be given at the older person's level of understanding. REF: Page 1256, Life Span Considerations box TOP: Older adult considerations Step: Planning

26. To what can delayed union of a fracture be attributed? (Select all that apply.) a. Inadequate immobilization b. Hormone replacement therapy c. Long-term use of corticosteroids d. Infection e. Poor nutrition

ANS: A, D, E Delayed union can be caused by inadequate immobilization, infection, poor nutrition, and poor alignment of the bone fragments. DIF: Cognitive Level: Knowledge REF: p. 974 OBJ: 3 TOP: Delayed Union KEY: Nursing Process Step: N/A

36. What would a nurse do when the patient arrives in the PACU after a left stapedectomy? (Select all that apply.) a. Turn the patient to his right side b. Change dressing as it becomes soiled c. Turn patient every 2 hours d. Leave the bed flat e. Medicate immediately on the complaint of nausea

ANS: A, D, E The bed is left in the flat position and the patient is positioned with the operated side facing up, the patient is not turned, and the dressing is not changed by the nurse. The patient should be medicated immediately on complaint of nausea to prevent vomiting and possible disruption of graft. PTS: 1 DIF: Cognitive Level: Analysis REF: Page 1887 OBJ: 17 TOP: Stapedectomy KEY: Nursing Process Step: Implementation

23. What should a nurse include when educating a patient with Ménière disease? a. "When you feel dizzy, just stay in bed and take your medications." b. "Decrease your sodium intake and take your diuretic medication between attacks." c. "Vestibular rehabilitation might help, and you can still drink your morning coffee." d. "Your vertigo will get better if you take your medications. You won't need any relaxation techniques."

ANS: B A low-sodium diet and diuretic medications between attacks will prevent edema, which could cause an attack. DIF: Cognitive Level: Application REF: p. 1267 OBJ: 7 TOP: Inner Ear KEY: Nursing Process Step: Implementation MSC:

24. A patient who had an enucleation of the right eye has been admitted PACU. What should the nurse include in the plan of care? a. Turn, cough, and deep breathe every 3 hours b. Apply a pressure dressing over the right eye socket c. Document dressing assessment every 2 hours d. Turn on the affected side

ANS: B A pressure dressing will be applied to the right eye socket and the dressing should be checked every hour for the first 24 hours. PTS: 1 DIF: Cognitive Level: Application REF: Page 1872 OBJ: 11 TOP: Infections/inflammatory disorders KEY: Nursing Process Step: Assessment

1. A nurse is aware that a patient who is having his leg amputated is also having a prosthesis fitted during at the same time as the surgery. Which fact should the preoperative teaching plan include? a. Extra preoperative medications will be needed. b. A rigid dressing will be applied to accommodate the prosthesis. c. A series of temporary prostheses will be put in place before the permanent one. d. Wiring the residual limb will be needed to ensure acceptance of the prosthesis.

ANS: B A rigid dressing will be applied to the residual limb to accommodate the prosthesis immediately after surgery. DIF: Cognitive Level: Application REF: p. 993 OBJ: 3 TOP: Preoperative Teaching KEY: Nursing Process Step: Planning MSC:

9. Which sign or symptom suggests that a patient with impaired skin integrity is developing a systemic infection? a. Lesion on the patient's leg that is swollen and warm to the touch b. Temperature that has risen to 101° F c. Blood pressure that has risen from 126/84 to 130/86 mm Hg d. Request by the patient for medication for severe itching

ANS: B A rise in temperature is a systemic response. Normal blood pressure, warmth, swelling, and itching are not evidence of a systemic skin infection. DIF: Cognitive Level: Comprehension REF: p. 1195 OBJ: 3 TOP: Systemic Infection KEY: Nursing Process Step: Assessment

27. A patient, age 23, has been diagnosed with external otitis on return from vacation at the beach. The physician places an ear wick in the external ear canal and prescribes antibiotic otic drops. Before the patient leaves the clinic, the nurse reviews the management of the disorder with her. Which statement by her indicates the need for further instruction? a. "I may use aspirin for the pain." b. "I should remove the wick tomorrow." c. "I should apply the ear drops to the cotton wick placed in my ear canal." d. "I may use warm compresses to the outside of my ear for comfort."

ANS: B A wick is inserted into the ear canal to prevent loss of medication from the canal and to maintain continuous absorption of the medication. REF: Page 1871 TOP: External otitis Step: Evaluation

4. The patient, age 42, notices that she is having difficulty reading typed print. The nurse would explain that the decreased ability to accommodate for near vision is called a. senility. b. presbyopia. c. myopia. d. refraction.

ANS: B Measurement of visual acuity can determine refractory errors such as presbyopia, the inability to focus on close objects. DIF: Cognitive Level: ApplicationREF: Page 1843, Table 53-2 OBJ: 6 TOP: Vision

16. A client who is a victim of a penetrating trauma has bowel loops protruding through the abdominal wall. Which of these actions should the nurse take FIRST? a. Administer narcotics if indicated by pain level expressed by the client. b. Cover the exposed intestine with saline-soaked gauze. c. Insert a large-bore intravenous (IV) access. d. Prepare the client for a peritoneal lavage procedure.

ANS: B Abdominal emergencies can have many causes and can result in multiple injuries. Internal hemorrhage is a potential complication. If loops of intestines are exposed to the outside air, they should be covered with saline-soaked gauze. IV access, oxygen administration, and standard protocol for managing the ABCs are instituted immediately.

20. A patient, age 29, is diagnosed with genital herpes. She is receiving acyclovir (Zovirax). Which would indicate a therapeutic response? a. Decrease in pruritus b. Decrease in pain c. Increase in WBC count d. Increase in activity tolerance

ANS: B Acyclovir (Zovirax) is an antiviral agent that can alter the course of the disease. REF: Page 1305 TOP: Herpes Step: Evaluation

18. Which nursing diagnosis has the highest priority after surgery for the open reduction and external fixation of an ankle? a. Risk for activity intolerance b. Risk for infection c. Risk for impaired physical mobility d. Risk for constipation

ANS: B All are possible diagnoses, but risk for infection would have the highest priority because bone infections are serious complications. DIF: Cognitive Level: Analysis REF: p. 983-984 OBJ: 6 TOP: Nursing Diagnosis KEY: Nursing Process Step: Nursing Diagnosis

16. A client may have cellulitis. Which laboratory test will the physician order to detect an acute inflammatory process? a. ESR b. C-reactive protein c. prealbumin d. chloride

ANS: B An abnormal protein appears in the blood of clients with an acute inflammatory process. C-reactive protein is used to monitor the progress of clients with infectious processes. This test is more sensitive than erythrocyte sedimentation rate (ESR).

20. An 85-year-old patient has had age-related changes in the cochlea. What is the most appropriate nursing action for the nurse to implement? a. Speak slowly. b. Provide assistance with ambulation. c. Speak in a lower tone. d. Communicate with the patient in writing.

ANS: B Assisting the patient when ambulating will diminish the risk of a fall. Changes in the cochlea will cause dizziness and ataxia. DIF: Cognitive Level: Application REF: p. 1252 OBJ: 5 TOP: Age-Related Changes KEY: Nursing Process Step: Assessment MSC:

28. A patient and a nurse develop a preoperative teaching plan. In teaching the patient to cough effectively after surgery, the nurse should tell her to practice a. breathing through her nose, holding her breath, and exhaling slowly. b. taking three deep breaths and coughing from the chest. c. inhaling while contracting the abdominal muscles and exhaling while contracting the diaphragm. d. taking short, frequent panting breaths and coughing from the throat to clear accumulated mucus.

ANS: B Because lung ventilation is vital, the nurse assists the patient to turn, cough, and breathe deeply every 1 to 2 hours until the chest is clear. Having practiced this combination preoperatively, the patient is usually able to adequately remove trapped mucus and surgical gases. REF: Pages 1265-1266, Skills 42-2, 42- 3 TOP: Prevention of postoperative complications

12. Which complaint made by a 64-year-old patient during a health interview would alert the nurse to the possibility of cataracts? a. Pain in the eyes b. Difficulty driving at night c. Loss of peripheral vision d. Dry eyes

ANS: B Blurring of vision and difficulty driving at night is often the first subjective symptom reported by a patient who has cataracts. PTS: 1 DIF: Cognitive Level: Application REF: Page 1860 OBJ: 9 TOP: Cataracts KEY: Nursing Process Step: Assessment

12. A patient has a family history of cataracts. He asks what symptom would be present if he begins to develop them. The nurse might respond that the first symptom of a cataract is usually a. pain in the eyes. b. blurring of vision. c. loss of peripheral vision. d. dry eyes.

ANS: B Blurring of vision is often the first subjective symptom reported by a patient who has cataracts. REF: Page 1853 TOP: Cataracts

11. A 78-year-old retired teacher with a history of osteoporosis has fallen in her bathroom and sustained a subcapital femoral fracture. She is scheduled for an open reduction and internal fixation (ORIF) procedure in the morning. Which type of traction will most likely be implemented? a. Bryant b. Buck c. Pelvic d. Crutchfield tongs

ANS: B Buck traction is used to stabilize the fracture. The other options are not applicable. DIF: Cognitive Level: Application REF: p. 979-980 OBJ: 5 TOP: Traction KEY: Nursing Process Step: Evaluation

13. A nurse is caring for an obese patient who has been bedridden for a long time and who has a nursing diagnosis of "Risk for infection, related to obesity." Where is the best location for the nurse to assess for the moist red lesions of Candida albicans? a. Scalp, behind the ears b. Abdominal skin folds c. Shaft of the penis d. Sacrum and bony prominences

ANS: B C. albicans infection appears most often in skin folds. DIF: Cognitive Level: Comprehension REF: p. 1198 OBJ: 3 TOP: Yeast Infection KEY: Nursing Process Step: Assessment

23. A 3-year-old child has ingested a household cleaner, which the poison control center states is caustic. Which of these actions should the nurse take? a. Administer 15 cc syrup of ipecac, and repeat in 20 minutes to induce vomiting. b. Give copious amounts of water to dilute the substance. c. Give two 8-ounce glasses of milk to neutralize the acidity of the poison. d. Insert a nasogastric tube, and remove as much of the substance as possible.

ANS: B Clients who have ingested caustic agents should be given copious amounts of water to dilute the substance.

6. Which type of hearing loss indicates an inability of the sound waves to reach the inner ear? a. common b. conductive c. mixed d. sensorineural

ANS: B Conductive hearing loss is an inability of the sound waves to reach the inner ear. Sensorineural hearing loss involves an abnormal or diseased inner ear or auditory nerve. The sensorineural hearing loss associated with aging is called presbycusis.

20. How does cutaneous T-cell lymphoma differ from squamous cell and basal cell carcinomas? a. Does not metastasize b. Has a cause unrelated to sun exposure c. Can be treated with radiation d. Can be treated topically

ANS: B Cutaneous T-cell carcinoma appears in areas protected from the sun. All three neoplasms can metastasize, and all three neoplasms can be treated by radiation or topically. DIF: Cognitive Level: Comprehension REF: p. 1205-1206 OBJ: 6 TOP: Cutaneous T-Cell Carcinoma KEY: Nursing Process Step: Implementation

21. What are late signs of hemorrhage in the postoperative period after an amputation? a. Restlessness and increased respirations b. Cyanosis and hypotension c. Confusion and seizures d. Headache and hypertension

ANS: B Cyanosis and hypotension are late signs of hemorrhage in the postoperative period after an amputation. DIF: Cognitive Level: Comprehension REF: p. 997 OBJ: 4 TOP: Complications KEY: Nursing Process Step: Assessment MSC:

24. During the first 24 hours after a burn, fluid replacement is the treatment priority. Which assessment should alert the nurse that the fluid protocol is ineffective? a. Rectal temperature of 101° F b. Urine output of 20 mL/hr c. Crackles in the lower left lobe d. Significant edema in the burn area

ANS: B Decreased urinary output indicates that poor perfusion to the kidney still remains. Temperature elevation and edema are to be expected. Crackles in a patient who is dormant are not causes for alarm. DIF: Cognitive Level: Application REF: p. 1210 OBJ: 5 TOP: Burns: Fluid Replacement KEY: Nursing Process Step: Assessment

3. A disaster can BEST be described as: a. a consequence of natural or man-made events b. a situation or event posing a serious threat to public health c. the result of conditions beyond the scope of normal emergencies d. a legal term related to unexplained situations with unpredictable outcomes

ANS: B Disaster triage systems are used in the event of a community disaster, either natural or man-made. A different approach to categorizing clients is required because of the potential for large numbers of casualties.

9. After CPR has been initiated on an adult patient, the nurse assesses the effectiveness of CPR by confirming: a. an EKG pattern with each compression. b. a palpable carotid pulse during each compression. c. a compression depth of 1 1/2 to 2 inches. d. pupils that change from pinpoint to dilated.

ANS: B During effective CPR, a carotid pulse is palpable during each compression. REF: Page 744

23. The nurse debriding a wound explains the purpose of debridement is to: a. increase the effectiveness of the skin graft. b. prevent infection and promote healing. c. promote suppuration of the wound. d. promote movement in the affected area.

ANS: B Débridement is the removal of damaged tissue and cellular debris from a wound or burn to prevent infection and to promote healing. REF: Page 1335 TOP: Burns

25. A patient with a fractured pelvis says that she will not ambulate because of pain. What should a nurse inform the patient can be prevented with early ambulation? a. Back injury b. DVT c. Callus formation d. Disuse syndrome

ANS: B Early ambulation, although painful, avoids many of the complications of immobility such as DVT, constipation, and atrophy. DIF: Cognitive Level: Comprehension REF: p. 974 OBJ: 3 | 5 TOP: Early Ambulation with Fractured Pelvis KEY: Nursing Process Step: Implementation

5. What information should a nurse provide to a patient with vitiligo receiving phototherapy? a. "Expose yourself to the sun for several hours before treatment to acclimate the skin surface." b. "Wear protective clothing." c. "Wear loose clothing such as sleeveless T-shirts and shorts after the treatment." d. "Leave off sunglasses after treatment so your eyes can more quickly accommodate."

ANS: B Eight hours before and after each treatment, the patient should wear protective clothing, sunglasses, and sunscreen to decrease added ultraviolet exposure from other sources. DIF: Cognitive Level: Application REF: p. 1190 OBJ: 6 TOP: Phototherapy KEY: Nursing Process Step: Implementation

12. A patient is transferred from the operating room to the recovery room after undergoing an open reduction and internal fixation (ORIF) of his left ankle. Which is the first assessment to make? a. Check ankle dressings. b. Check airway for patency. c. Check intravenous site. d. Check vital signs.

ANS: B Evaluation of the patient follows the ABCs of immediate postoperative observation: airway, breathing, consciousness, and circulation. REF: Page 1281, Table 42-6 TOP: Nursing assessment

25. A client collapses during a marathon, complaining of headache, weakness, dizziness, nausea, and muscle cramps. The client's skin is moist, cool, and pale. Which of these actions should the nurse take? a. Move client to a cool area, and administer copious amounts of water. b. Loosen constrictive clothing, pour water over client, and place client near a fan. c. Immerse client in cool-water bath, and initiate IV fluids. d. Initiate IV fluid replacement, place client on cardiac monitor, remove clothing, and pour cool water over client.

ANS: B Extreme heat also causes potentially serious problems, especially in very young or elderly clients. Nursing interventions include loosening constrictive clothing, pour water over client to reduce body temperature, and place client near a fan.

16. Most patients with Ménière's disease are treated with a. surgery. b. diuretics. c. hearing aids. d. analgesics.

ANS: B Fluid restriction, diuretics, and a low-salt diet are prescribed in an attempt to decrease fluid pressure. REF: Page 1877 TOP: Ménière's disease

25. What must a patient do following a left vitrectomy? a. Remain flat in bed for 48 hours b. Position self in a face-down position for 4 to 5 days c. Assume a side-lying position with the left side down for 3 days d. Keep head upright and cushioned with pillows for 24 hours

ANS: B Following a vitrectomy, the patient must assume a face-down position or turn the face to the right side for 4 to 5 days. PTS: 1 DIF: Cognitive Level: Application REF: Page 1874 OBJ: 11 TOP: Vitrectomy KEY: Nursing Process Step: Planning

9. A patient is prescribed eyedrops that constrict the pupil, permitting aqueous humor to flow. The nurse would reinforce the teaching by referring to the drops as a. mydriatics. b. miotics. c. osmotics. d. inhibitors.

ANS: B Miotics are agents that cause the pupil to contract or constrict. REF: Page 1861 TOP: Medication

14. A patient asks why systemic glucocorticoid medications are used as the last choice for the treatment of rheumatoid arthritis. What is the nurse's most informative reply? a. "The other drugs are just as effective and work in similar ways." b. "They are used as a last choice or for short periods because they have many side effects." c. "Those drugs are given three or four times daily, which is more difficult for patients to remember." d. "A higher incidence of vomiting occurs with prolonged use."

ANS: B Glucocorticoids are used as a last choice because they have many side effects. DIF: Cognitive Level: Comprehension REF: p. 949 OBJ: 5 TOP: Drug Therapy of Rheumatoid Arthritis KEY: Nursing Process Step: Implementation MSC: NCLEX: Physiological Integrity:

14. What is the greatest danger in the early postoperative period after an amputation? a. Infection b. Hemorrhage c. Pain d. Edema

ANS: B Hemorrhage is the greatest danger in the early postoperative period after an amputation. DIF: Cognitive Level: Comprehension REF: p. 997 OBJ: 4 TOP: Complications KEY: Nursing Process Step: Assessment MSC:

24. Two days after a total hip replacement, a patient is being discharged. Which statement indicates that the patient understands the discharge teaching? a. "I can sit comfortably with my legs crossed." b. "I will ask my husband to tie my shoes for me." c. "I am glad I won't have to use that bulky pillow between my legs at night." d. "My straight dining room chair will be helpful when I do the hip flexion exercises."

ANS: B If the patient bends over to tie her own shoes, her hips would have more than 90 degrees of flexion. DIF: Cognitive Level: Comprehension REF: p. 955 OBJ: 8 TOP: Hip Arthroplasty KEY: Nursing Process Step: Evaluation MSC: NCLEX: Physiological Integrity:

4. A child has been sent home from school with pruritus and honey-colored crusts on his lower lip and chin. A probable diagnosis would be a. chickenpox. b. impetigo. c. shingles. d. herpes simplex type I.

ANS: B Impetigo is seen at all ages, but is particularly common in children. The crust is honey-colored and easily removed. REF: Page 1312 TOP: Infection

22. A patient has been walking in the woods. He complains of severe pruritus. The nurse notes an erythematous area on his lower legs. The first nursing intervention for dermatitis venenata would be to a. administer Benadryl, 50 mg IM. b. wash area with copious amounts of water. c. apply cool compresses continuously. d. expose area to heat and air.

ANS: B In dermatitis venenata (poison oak or ivy), the patient should wash the affected part immediately after contact with the offending allergen. REF: Page 1316 TOP: Pruritus

1. What are the two main functions of the inner ear? a. amplification and sound transmission b. hearing and equilibrium c. pressure equalization and hearing d. sound transmission and pressure equalization

ANS: B In the inner ear, the nerves respond to different sounds and initiate neural responses that are sent along the auditory nerve (cranial nerve VIII) to the brain. The inner ear also maintains equilibrium.

6. A client has a skin condition in which the melanocytes are destroyed causing milk-white patches of depigmented skin surrounded by normal skin. This condition is called: a. keratinitis b. vitiligo c. melanocyte dysfunction d. dermatitis

ANS: B In vitiligo, melanocytes are destroyed, causing milk-white patches of depigmented skin surrounded by normal skin.

4. An informed consent was to be obtained from the patient for his scheduled open cholecystectomy. Which circumstance could prevent the patient from signing his informed consent? a. Pain radiating to the scapula b. An injection of Demerol, 75 mg IM, 1 hour ago c. The presence of jaundice and scleral icterus d. His concern over his insurance company not covering the procedure

ANS: B Informed consent should not be obtained if the patient is disoriented and under the influence of sedatives. REF: Page 1261 TOP: Informed consent

13. An important component in the assessment of a client's skin is: a. only performing a skin surface assessment b. always provide privacy during assessment c. allowing a client a choice in the areas for assessment d. removal of all clothing and sheets at the same time for a full body assessment

ANS: B It is important to always provide privacy during skin assessment.

10. The nurse is performing a brief physical assessment on a client at the beginning of the shift. The nurse notices that the client has yellowing skin, mucous membranes and most notably, yellow sclera. The nurse documents which of the following descriptors? a. pallor b. jaundice c. cyanosis d. erythema

ANS: B Jaundice is the yellowing of the skin, mucous membranes, and sclera. Pallor is pale skin, especially in face, conjunctiva, nail beds, and oral mucous membranes. Cyanosis is described as bluish discoloration noticed in lips, earlobes, and nail beds. Erythema is a reddish hue to the skin as in sunburn and inflammation or increased blood flow.

9. A 30-year-old African American had surgery 6 months ago. Her incisional site is now raised, indurated, and shiny. This tissue growth is most likely a(n) a. angioma. b. keloid. c. melanoma. d. nevus.

ANS: B Keloids, which originate in scars, are hard and shiny and are seen more often in African Americans than Caucasians. DIF: Cognitive Level: Knowledge REF: Pages 1301, 1303, 1306, 1307, Table 43-1, Figure 43-16 TOP: Keloid Step: Evaluation

1. Displaying her hands, a patient asks, "Do you think my liver is OK? Look at all these liver spots!" What is the most appropriate nursing response? a. "The spots could mean something is wrong; I will make a note of it." b. "The spots are normal aging changes and have nothing to do with your liver." c. "Have you recently been exposed to hepatitis?" d. "Don't worry about them. They will fade during the winter."

ANS: B Lentigines on sun-exposed areas are called liver spots because of their color; they have nothing to do with the liver or any disease process. They are normal changes of aging. DIF: Cognitive Level: Comprehension REF: p. 1184 OBJ: 2 TOP: Liver Spots KEY: Nursing Process Step: Implementation

27. What is the first indication of macular degeneration? a. The loss of peripheral vision b. The loss of central vision c. The loss of color discrimination d. Eye fatigue

ANS: B Macular degeneration is characterized by the slow loss of central and near vision. PTS: 1 DIF: Cognitive Level: Analysis REF: Page 1864 OBJ: 9 TOP: Macular degeneration KEY: Nursing Process Step: Assessment

11. For a client with Ménière's disease whose nursing diagnosis is Risk for injury related to vertigo, which intervention is essential? a. Administer medications for vertigo when the symptoms begin. b. Avoid glaring, bright lights. c. Have a family member stay with the client at all times. d. Teach the client to ambulate slowly when moving around the room or the hallway.

ANS: B Ménière's disease is characterized by tinnitus, vertigo, and unilateral fluctuating hearing loss. Nausea, vomiting, or diaphoresis may occur. The cause is unknown. Medications are prescribed to control the symptoms, and avoiding glaring, bright lights can reduce the risk of injury.

10. The client has a disorder that is characterized by the classic triad of tinnitus, vertigo, and unilateral fluctuating hearing loss. The nurse is aware that the client has which type of problem? a. acoustic neuroma b. Ménière's disease c. otitis media d. otosclerosis

ANS: B Ménière's disease is characterized by tinnitus, vertigo, and unilateral fluctuating hearing loss. Nausea, vomiting, or diaphoresis may occur. The cause is unknown. Medications are prescribed to control the symptoms.

3. A 75-year-old patient has normal age-related changes in his ear. What change should not be considered a normal change in the aging patient? a. Dry and wrinkled skin on the auricle b. Otitis externa c. Dry cerumen d. Hair in the ear canal

ANS: B Otitis externa is an outer ear infection and therefore an exception. The other three options are normal age-related changes. DIF: Cognitive Level: Comprehension REF: p. 1251 OBJ: 5 TOP: Age-Related Changes KEY: Nursing Process Step: Assessment MSC:

19. Which condition frequently results from chronic or recurrent bacterial otitis media extending into the bone and structures surrounding the middle ear? a. acoustic neuroma b. mastoiditis c. Ménière's disease d. otosclerosis

ANS: B Otitis externa, also called "swimmer's ear," generally is a bacterial infection of the external ear canal. Mastoiditis, inflammation of the mastoid bone, is most often the result of chronic bacterial otitis media. Antibiotics are prescribed for both infections.

15. A client comes to the emergency department complaining of abdominal pain. Nursing assessment should focus on which of the following? a. Acquiring a history of previous surgical procedures. b. Identifying location, duration, severity, and radiation of the pain. c. Determining normal bowel and bladder patterns. d. Obtaining history of food allergies.

ANS: B Pain assessment includes identifying location, duration, severity, and radiation of the pain. This should be done to determine that cause of the pain.

11. A client is exhibiting a paradoxical breathing pattern. The nurse should recognize this unique pattern is associated with which of these conditions? a. cardiac tamponade b. flail chest c. myocardial infarction d. pneumothorax

ANS: B Paradoxical breathing pattern is a unique pattern is associated with flail chest.

14. Which nursing diagnosis is most appropriate for a patient having ear surgery? a. Disturbed body image b. Risk for injury c. Acute confusion d. Ineffective protection

ANS: B Patients who have had ear surgery are at risk for vertigo, fluid accumulation, or pressure in the operative ear. Because of the surgery and potential postoperative conditions, the patient may be at risk for a fall. DIF: Cognitive Level: Application REF: p. 1259-1260 OBJ: 7 TOP: Care Planning for Ear Surgery KEY: Nursing Process Step: Planning MSC:

11. A client presents to the urgent clinic about some little red dots all over his body. The nurse examines the client and notices pinpoint hemorrhagic spots. The nurse documents this observation as: a. telangiectasia b. petechiae c. ecchymosis d. nevi

ANS: B Petechiae is described as pinpoint hemorrhagic spots. Telangiectasia is a condition where there is permanent dilation of groups of superficial capillaries and venules. Ecchymosis is a large, irregular, hemorrhagic area or a bruise.

24. A 75-year-old patient reports having difficulty hearing in crowds but can hear just fine at home with his wife. What hearing disorder should the nurse suspect? a. Otitis media b. Presbycusis c. Ototoxicity d. Central deafness

ANS: B Presbycusis is a conductive hearing loss associated with normal aging and is caused by changes in the cochlea. DIF: Cognitive Level: Comprehension REF: p. 1269 OBJ: 5 TOP: Presbycusis KEY: Nursing Process Step: Implementation MSC:

30. The best prevention of serious complications of ear disorders like infections, mastoiditis and brain abscesses is: a. Strong antibiotics and isolation b. Early detection and treatment c. Surgery and rehabilitation d. Bed rest and early speech reading techniques

ANS: B Prevention of serious complications of ear disorders—such as infections, mastoiditis, and brain abscess—requires early detection and treatment. REF: Page 1883, Key Points TOP: Health promotion

10. A patient with multiple serious injuries sustained in a motorcycle accident is lying beside his wrecked motorcycle unconscious and bleeding when the rescuer arrives at the scene. The rescuer's priority action would be: a. assessing blood loss. b. assessing respiratory status. c. obtaining vital signs. d. organizing laypeople at the scene.

ANS: B Priority intervention is to assess respiratory status. REF: Page 741

13. A nursing report on a newly admitted patient who is profoundly deaf says that the patient is confused and difficult to assess because she does not appropriately respond to questions or sometimes fails to respond at all. What should be the first action of the oncoming nurse? a. Consider asking the physician to assess the patient for dementia. b. Assess the patient to determine whether her hearing aids are in. c. Report to the physician that the patient is exhibiting signs of the sundown syndrome. d. Assess the patient's medications to check for an overdose.

ANS: B Profoundly deaf persons can be mistakenly assessed as being confused or disoriented when not wearing their hearing aids. DIF: Cognitive Level: Application REF: p. 1258 OBJ: 5 TOP: Hearing Aids KEY: Nursing Process Step: Planning MSC:

2. A confused patient has been restrained because of combativeness and hyperactivity. What skin assessment may occur as a result of the restraints? a. Lentigines b. Senile purpura c. Senile angiomas d. Seborrheic keratoses

ANS: B Purpura are purple bruises that resolve very slowly and are usually the result of minor trauma. DIF: Cognitive Level: Comprehension REF: p. 1184 OBJ: 2 TOP: Senile Purpura KEY: Nursing Process Step: Assessment

18. A nurse is educating a patient with gout about a low-purine diet. Which food choice by the patient would indicate the need for further teaching? a. Pizza with pepperoni b. Seafood platter with scallops and mussels c. Chicken salad with nuts d. Tuna sandwich with potato chips

ANS: B Seafood, such as scallops and mussels, are high in purine. DIF: Cognitive Level: Application REF: p. 964 OBJ: 8 TOP: Low-Purine Diet KEY: Nursing Process Step: Evaluation MSC: NCLEX: Health Promotion and Maintenance:

8. The patient arrived at the emergency department in pain and bleeding profusely with the following vital signs: BP 80/54, P 102, RR 22. The nurse recognizes these symptoms as indicative of: a. inadequate perfusion. b. circulatory shock. c. massive vasodilation. d. heart failure.

ANS: B Shock results from failure of the circulatory system to provide sufficient blood circulation. REF: Page 747

19. What nursing action should be implemented in the postoperative care for a patient with replantation of the right thumb? a. Decreasing the temperature of the room to 70° F b. Elevating the hand but keeping it below the level of the heart c. Offering coffee, tea, or cola to help increase fluid intake d. Placing an antiembolus sleeve on the right arm

ANS: B Slight elevation of the hand will encourage drainage but not affect arterial perfusion. The room temperature should be 80° F. The patient should avoid all caffeine drinks and tight clothing or dressings. DIF: Cognitive Level: Application REF: p. 1002 OBJ: 4 TOP: Nursing Care of Replantation KEY: Nursing Process Step: Assessment MSC:

8. A patient with peripheral vascular disease (PVD) and diabetes asks what he can do to help prevent an amputation. What is the best response of the nurse? a. "There is not really anything you can do to help." b. "Stopping smoking would help prevent vasoconstriction." c. "You will not need to check your blood glucose levels." d. "It is important to eat big meals so your body can heal."

ANS: B Smoking causes vasoconstriction, which aggravates PVD. DIF: Cognitive Level: Comprehension REF: p. 992 OBJ: 3 TOP: Prevention of Disease KEY: Nursing Process Step: Implementation MSC:

20. What is the BEST initial intervention to control rapid spreading of venom in an extremity such as the foot? a. specific antivenom administration b. application of a rubber band c. application of a tourniquet d. aspiration of venom from the bite site

ANS: B Snakebites do not always result in poisoning. Application of a rubber band, not application of a tourniquet, is the best initial intervention to control rapid spreading of venom in an extremity.

12. Which characteristic should a nurse recognize as diagnostic of rheumatoid arthritis? a. Absence of pain b. Symmetric bilateral joint swelling c. Evening stiffness that improves with activity d. Increased appetite

ANS: B Symmetric bilateral joint swelling is a classic symptom of rheumatoid arthritis. DIF: Cognitive Level: Comprehension REF: p. 957 OBJ: 6 TOP: Rheumatoid Arthritis KEY: Nursing Process Step: Implementation MSC: NCLEX: Physiological Integrity:

9. When the newly blind male home health patient asks the nurse how he might get assistance, who might the nurse suggest he contact? a. American Red Cross b. American Foundation for the Blind for a list of agencies c. Local hospital social worker d. The public health department

ANS: B The American Foundation for the Blind has lists of agencies to assist and educate the visually impaired patient. PTS: 1 DIF: Cognitive Level: Analysis REF: Page 1852 OBJ: 15 TOP: Medications KEY: Nursing Process Step: Implementation

2. The Good Samaritan Law will protect all people who offer assistance if: a. they are licensed. b. the person acts prudently. c. there is licensed supervision. d. the patient improves.

ANS: B The Good Samaritan Law will protect any person who follows a prudent course of action. REF: Page 739

1. The nurse is aware that the patient has 20/40 vision. This means that the patient can see at 20 feet what the normal eye can see at feet. a. 100 b. 200 c. 300 d. 400

ANS: B The Snellen Eye Chart tests visual acuity. A vision evaluation of 20/40 means that the patient can see at 20 feet what the person with normal vision can see at 200 feet. PTS: 1 DIF: Cognitive Level: Application REF: Page 1850 OBJ: 7 TOP: Snellen evaluation KEY: Nursing Process Step: Assessment

18. A physician asks a nurse to take a smear from herpetic lesions in an older patient's hip to diagnose the disorder. What is the most probable test that will be performed? a. Culture and sensitivity test to a bactericide b. Tzanck smear to test for viral culture c. Complete blood count to assess the white blood count for response to a pathogen d. Titration for the strength of the pathogen

ANS: B The Tzanck test rapidly confirms the specific virus. The results are available sooner than they would be from a culture. DIF: Cognitive Level: Comprehension REF: p. 1202 OBJ: 7 TOP: Tzanck Smear KEY: Nursing Process Step: Planning

20. A common test used to distinguish between conductive and sensorineural hearing loss is the: a. Whisper test b. Weber test c. Romberg test d. Tympanometry

ANS: B The Weber test is inaccurate and unreliable as screening and diagnostic tests but it does distinguish between conductive and sensorineural hearing loss.

18. Which nursing diagnosis is most appropriate when considering the impact of a hearing deficit when planning care for a child who has been diagnosed with a hearing impairment? a. Risk for injury, related to hearing impairment b. Risk for social isolation, related to hearing impairment c. Knowledge deficit, related to hearing impairment d. Anxiety, related to hearing impairment

ANS: B The loss of hearing and the mild stigma associated with hearing impairment place the newly diagnosed child at risk for social isolation. DIF: Cognitive Level: Application REF: p. 1262 OBJ: 7 TOP: Impact of Hearing Impairment KEY: Nursing Process Step: N/A MSC:

14. The nurse is attempting to control bleeding in a patient with a profusely bleeding scalp wound. The most effective initial treatment of bleeding is to: a. elevate the head. b. apply direct pressure. c. apply an ice pack. d. apply indirect pressure.

ANS: B The most effective general treatment of bleeding is to apply direct pressure. REF: Page 749

18. When should the nurse offer prescribed analgesics to a patient who is 24 hours postoperative? a. Only when the patient asks. b. Regularly every three to four hours before pain gets severe. c. Only when the physician orders. d. Only when the patient is in severe pain.

ANS: B The nurse should ask the patient every 3-4 hours if something is needed for pain because some patients will not ask for an analgesic. REF: Page 1285 TOP: Medication administration Step: Planning

22. A patient who had taken a poisonous substance was brought to the emergency department. The nurse should first: a. give syrup of ipecac. b. contact the poison control center. c. give milk to coat the stomach. d. observe for symptoms.

ANS: B The nurse should immediately call the poison control center. REF: Page 755 TOP: Poison

11. A patient who had a below-the-knee amputation 24 hours earlier is complaining of burning pain in his left foot. Which intervention is most appropriate for the nurse to implement? a. Remind the patient that it is only phantom pain. b. Medicate the patient with the prescribed pain remedy. c. Remind him that such sensations will go away in a few weeks. d. Distract the patient with conversation.

ANS: B The nurse should medicate a patient for pain. Phantom pain is real. Although distraction is a possible intervention, it is not the most effective. DIF: Cognitive Level: Application REF: p. 999 OBJ: 4 TOP: Phantom Pain KEY: Nursing Process Step: Implementation MSC:

17. Which nursing diagnosis should take priority in a nursing care plan for a patient with Ménière disease? a. Social isolation, related to anxiety b. Risk for injury, related to falls c. Risk for deficient fluid intake, related to weakness d. Nutrition: Less than body requirements, related to fatigue

ANS: B The nursing diagnosis that should take priority is that of preventing injury to the patient. A patient with Ménière disease is prone to falls because of dizziness. DIF: Cognitive Level: Application REF: p. 1266 OBJ: 7 TOP: Nursing Care Plan for Ménière Disease KEY: Nursing Process Step: Assessment MSC:

16. Which patient is most appropriate for a nurse to refer to home health care? a. A married man with a laundry room on the first floor b. A single woman with a bedroom in a rooming house c. A student living in a college dormitory but going home to stay with parents d. A woman staying with her daughter and son-in-law at their one-story home

ANS: B The patient will need help with laundry and other activities of daily living. DIF: Cognitive Level: Analysis REF: p. 984 OBJ: 6 TOP: Nursing Assessment of a Patient with a Fracture KEY: Nursing Process Step: Assessment

13. What is an appropriate outcome for a nursing diagnosis, "Anxiety, related to perceived threat of disability?" a. Comfort is increased; verbalized pain is less. b. Anxiety is relieved; the patient verbalizes concern related to disability. c. Grief is resolved; the patient expresses an acceptance of loss. d. Residual limb is cleaned; no exudate, redness, or edema is observed.

ANS: B The relief of anxiety speaks directly to the nursing diagnosis. options are possible outcome goals but do not pertain to this diagnosis. DIF: Cognitive Level: Application REF: p. 997 OBJ: 4 | 5 TOP: Nursing Care KEY: Nursing Process Step: Planning MSC:

4. A nurse is caring for a patient with osteoarthritis. What is the best recommendation by the nurse to this patient to control chronic pain? a. Administer analgesics only when needed. b. Administer analgesics as prescribed on a routine basis. c. Plan activities with no rest periods to complete the activities quickly. d. Wear high-heeled shoes to keep the body in alignment.

ANS: B The routine administration of prescribed analgesic medications is the most appropriate treatment for chronic pain. DIF: Cognitive Level: Application REF: p. 952 OBJ: 8 TOP: Osteoarthritis KEY: Nursing Process Step: Implementation MSC: NCLEX: Physiological Integrity:

7. The sebaceous glands secrete an oily substance called sebum that lubricates the skin helping to keep the skin soft and pliable. What is the other function of sebum? a. help regulate body heat b. protects skin by killing bacteria c. promote sensation of touch and pain d. melanocyte production

ANS: B The sebaceous glands secrete an oily substance called sebum that lubricates the skin, helping to keep the skin soft and pliable. Sebum also protects the skin by killing bacteria. The amount of sebum secreted is influenced by hormones.

16. The nurse will assess for when the older adult home health patient complains that the entire right side of his head hurts and he cannot chew without pain. a. mumps b. external otitis c. otitis media d. labyrinthitis

ANS: B The symptoms of painful head, painful chewing, and pain when the auricle is moved all indicate external otitis, frequently caused by compacted cerumen. PTS: 1 DIF: Cognitive Level: Knowledge REF: Page 1880 OBJ: 16 TOP: External otitis KEY: Nursing Process Step: Assessment

21. The nurse is assisting a victim of an accident who requires bandaging of the right lower extremity. When applying the bandage, the nurse should: a. use sterile material. b. leave the toes exposed. c. bandage the extremity tightly. d. bend the knee after bandaging.

ANS: B The tips of the toes should remain exposed to assess circulation. REF: Page 753 TOP: Bandaging

19. What information should a nurse stress when teaching a patient with Ménière disease about managing the disorder? a. Limiting fluid intake b. Avoiding the use of alcohol and tobacco c. Using antiemetic medications sparingly d. Staying active during the day

ANS: B The use of alcohol and tobacco products affects the amount of fluid in the middle ear, worsening the symptoms of Ménière disease. The patient with Ménière disease should drink adequate fluid, use antiemetic medications as needed, and conserve energy during the day. DIF: Cognitive Level: Application REF: p. 1266 OBJ: 7 TOP: Ménière Disease KEY: Nursing Process Step: Implementation MSC:

29. What is the responsibility of the nurse regarding informed consent? a. Explain the surgical options. b. Explain the operative risks. c. Obtain the patient's signature. d. Check form for appropriate signatures.

ANS: C A witness is only verifying that this is the person who signed the consent and that it was a voluntary consent. The witness (often a nurse) is not verifying that the patient understands the procedure. REF: Page 1261 TOP: Informed consent

25. The nurse answers the call light of a newly admitted patient. The patient tells the nurse she is blind and asks the nurse to assist her to the bathroom. Which action by the nurse is most appropriate? a. Take the patient by the arm and lead her slowly to the bathroom. b. Walk slightly ahead of the patient and allow the patient to hold the nurse's elbow. c. Give the patient exact descriptive directions to the bathroom so that she can walk there independently. d. Lead the patient to the bathroom by the hand, describing the location and providing verbal clues.

ANS: B The walking companion should precede the patient by about 1 foot, and the patient's hand should be on the companion's elbow to provide security. REF: Page 1846, Figure 53-5 TOP: Blindness Step: Planning

1. Two days after surgery for a crushed pelvis, a certified nursing assistant (CNA) reports that the patient is complaining of a shortness of breath and is demonstrating signs of confusion and restlessness. What should a nurse suspect, from these signs alone, that the patient has developed? a. Impending shock b. Fat embolus c. Anxiety d. Neurovascular compromise

ANS: B These are the classic symptoms of a fat embolus that has escaped from the crushed marrow. DIF: Cognitive Level: Analysis REF: p. 974 OBJ: 3 | 6 TOP: Complications: Fat Embolism KEY: Nursing Process Step: Assessment| Nursing Process Step: Evaluation

25. What action would best benefit the patient diagnosed with bursitis of the shoulder? a. Lifting a 5-lb weight as a daily exercise b. Walking the fingers of the affected arm up the wall c. Splinting the affected arm to keep the shoulder immobile d. Performing gentle push-ups on the floor

ANS: B Walking the fingers up the wall is a gentle exercise to increase range of motion. DIF: Cognitive Level: Comprehension REF: p. 968 OBJ: 6 TOP: Exercises for Bursitis KEY: Nursing Process Step:

17. The nurse has initiated measures to promote suppuration of a carbuncle. Which assessment would indicate that these measures have been successful? a. The area is erythematous and scaly. b. The area has begun to drain exudate. c. The area is not erythematous or edematous. d. The area has stopped draining exudate.

ANS: B Warm soaks two to three times a day can be used to speed the process of suppuration (production of purulent material). Patients should be taught not to touch the exudate. REF: Page 1313 TOP: Skin eruptions Step: Evaluation

17. A farm worker who has been kicked in the stomach by a mule passes a foul, black, tarry stool. The nurse suspects: a. loose stool. b. melena. c. hematuria. d. hemoptysis.

ANS: B When internal bleeding occurs, the patient may demonstrate hemoptysis, hematemesis, melena, or hematuria. REF: Page 751 TOP: Melena

7. When a patient suddenly experiences respiratory difficulty in the cafeteria, the nurse begins assessment for foreign body airway obstruction. The most appropriate question to the victim is: a. "What did you swallow?" b. "Are you choking?" c. "Are you OK?" d. "Can I help you?"

ANS: B With complete airway obstruction, the victim cannot speak. Ask, "Are you choking?" With this question the nurse pinpoints the problem and can perform the Heimlich with no wasted time. REF: Page 746

9. Which patient is most likely to develop a connective tissue disease? a. A teenage girl who swims b. A 30-year-old woman who plays tennis c. A 35-year-old male golfer d. A 40-year-old male computer analyst

ANS: B Women have a greater chance than men of developing connective tissue disease. DIF: Cognitive Level: Comprehension REF: p. 939 OBJ: 7 TOP: Connective Tissue Disease KEY: Nursing Process Step: Planning MSC: NCLEX: Health Promotion and Maintenance:

29. What actions would be best for patients with osteoarthritis to seek the assistance of physical therapy? (Select all that apply.) a. Isotonic exercises b. Moist heat application c. Instruction with a transcutaneous electrical nerve stimulation (TENS) unit d. Measures to increase range of motion e. Measures to increase strength

ANS: B, C, D, E Isotonic exercises place increased stress on the joints. DIF: Cognitive Level: Comprehension REF: p. 953 OBJ: 6 TOP: Benefits of Physical Therapy KEY: Nursing Process Step: Implementation MSC: NCLEX: Health Promotion and Maintenance:

26. A dark-skinned patient has been admitted to the hospital in severe respiratory distress. To determine whether the patient is cyanotic, the nurse uses what knowledge of skin assessment? a. It is not possible to assess color changes in patients with dark skin. b. Cyanosis in patients with dark skin can be seen only in the sclera. c. Cyanosis can be seen in the lips and mucous membranes of patients with dark skin. d. Cyanosis will blanch out with direct pressure to the soles of the feet in dark- skinned patients.

ANS: C Assessment of color is more easily made in areas where the epidermis is thin, such as the lips and mucous membrane. REF: Pages 1303-1304, Cultural Considerations TOP: Cyanosis

13. What are the primary clinical manifestations of otosclerosis? a. dizziness, possible facial weakness, tinnitus, and hearing loss b. ear pain, fever, and redness of the auricle and ear canal c. subtle changes in hearing and low-pitched tinnitus d. vertigo, tinnitus, and unilateral fluctuating hearing loss

ANS: C Otosclerosis is a conductive hearing loss that occurs when the ossicles become softened, spongy, highly vascular, and partially or totally fixed. Treatment options include hearing aids or surgery to remove the stapes and implant a plastic prosthesis. Some clients may opt to skip treatment.

20. Serum potassium levels are usually determined before surgery to a. assess kidney function. b. determine respiratory insufficiency. c. prevent dysrhythmias related to anesthesia. d. measure functional liver capability.

ANS: C Serum electrolytes are evaluated if extensive surgery is planned or the patient has extenuating problems. One of the essential electrolytes examined is potassium; if potassium is not available in adequate amounts, dysrhythmias can occur during anesthesia. REF: Page 1260 TOP: Preoperative assessment

21. When a client has a laceration of the skin, which of these must be initiated after the nurse cleanses the wound? a. application of pressure dressing b. administration of pain medication c. prophylactic administration of antibiotics d. suturing of laceration

ANS: C Soft-tissue injuries include minor abrasions, lacerations, puncture wounds, contusions, bites of all varieties, and burns. Most of these do not require emergency care, but clients often seek medical attention because of fear. All injuries should be cleansed. Infection is a major consideration, so antibiotics are often administered. The currency of the client's immunizations, especially diphtheria and tetanus, should be verified.

14. Clients with head injuries should be placed in which of these positions? a. dorsal recumbent b. prone c. semi-Fowler's d. side-lying

ANS: C A client with head, neck, or spinal cord trauma should be immobilized. A client with head injury should be placed in a semi-Fowler's position to decrease risk of bleedings and to reduce intracranial pressure (ICP).

17. The nurse takes into consideration that the Weber test indicated a conductive hearing loss in a patient because the patient reported hearing the tone: a. equally in both ears. b. as a shrill noise. c. louder in his affected ear. d. very faintly.

ANS: C A conductive hearing loss can be diagnosed by the Weber test. A person with a conductive loss will hear the noise louder in his affected ear. PTS: 1 DIF: Cognitive Level: Analysis REF: Page 1885 OBJ: 16 TOP: Weber test KEY: Nursing Process Step: Assessment

12. In assessing a brown skinned individual for pallor, the nurse should examine which area (s) of the body? a. palms b. soles of feet c. mucus membranes in mouth/conjunctiva d. pinna

ANS: C A dark skinned person will demonstrate pallor by having dull, grayish or ashen skin color. A brown skinned individual will have a more yellowish brown color. For these individuals, assess pallor by examining the mucus membranes of the mouth and conjunctiva. They will appear pale pink or ash gray.

33. Melanocytes give rise to the pigment melanin which is responsible for skin color. The melanocytes can be found in a. Dermis b. Superficial fascia c. Epidermis d. Loose connective tissue

ANS: C A layer in the epidermis contains highly specialized cells called melanocytes. REF: Page 1296 TOP: Structure of the skin

1. A patient visits the physician for a routine physical examination that involves testing distance vision. As she faces the Snellen chart, the nurse is to instruct the patient to a. use both eyes to read the chart. b. read the chart from right to left. c. cover one eye while testing the other. d. use any one eye since they will be the same.

ANS: C A major diagnostic eye test is the Snellen test. While instructing a patient to perform this test, the nurse will have the patient stand or sit 20 feet from the chart and cover one eye to read the letters on the chart. REF: Page 1844, Table 53-2 TOP: Physical examination

24. A home health care nurse suspects a neuroma in a patient who had an above-the-knee amputation 1 month earlier. Which complaint by the patient led the nurse to suspect a neuroma? a. Area of swelling and bruising on distal portion of residual limb b. Prickling sensation over residual limb c. Sharp severe pain in the residual limb d. Area of numbness on distal portion of residual limb

ANS: C A neuroma causes sharp severe pain in the residual limb. A neuroma occurs when severed nerve endings attempt to regenerate. The neuroma requires excision by the surgeon. DIF: Cognitive Level: Comprehension REF: p. 996 OBJ: 4 TOP: Neuroma KEY: Nursing Process Step: Assessment MSC:

13. Frequent assessment of a postoperative patient is essential. One of the first signs and symptoms of hemorrhage may be a. increasing blood pressure. b. decreasing pulse. c. restlessness. d. weakness, apathy.

ANS: C A pulse that increases and becomes thready combined with a declining blood pressure, cool and clammy skin, reduced urine output, and restlessness may signal hypovolemic shock. REF: Page 1284 TOP: Postoperative complications

25. During an intake physical examination, a patient reports that he has been taking 10 aspirin tablets a day for his arthritis. What question should the nurse ask based on this information? a. "Can you hear high-pitched sounds?" b. "Have you noticed deafness in just one ear?" c. "Do you have ringing in your ears?" d. "Do you experience dizziness when you stand?"

ANS: C A ringing in the ears (tinnitus) is an indication of aspirin toxicity. The patient should be advised to stop taking aspirin. DIF: Cognitive Level: Application REF: p. 1252-1253 OBJ: 5 TOP: ASA Toxicity KEY: Nursing Process Step: Implementation MSC:

14. What information is most essential for a nurse to gather when interviewing a young woman who is taking the drug isotretinoin (Accutane) for acne? a. Usual weight b. Family history of breast cancer c. Current method of birth control d. Drugs previously used

ANS: C Accutane can cause severe fetal deformities. DIF: Cognitive Level: Comprehension REF: p. 1200 OBJ: 6 TOP: Acne Treatment KEY: Nursing Process Step: Assessment

21. A patient comes to the primary care clinic complaining of a head cold and ear pain with drainage. What should the nurse suspect this patient is experiencing? a. Otitis externa b. Hearing loss c. Acute otitis media d. Mastoiditis

ANS: C Acute otitis media is connected with colds and drainage from the ear. A hearing loss may be experienced as well, but the pain and drainage place the need to intervene for the infection first. DIF: Cognitive Level: Comprehension REF: p. 1264 OBJ: 1 TOP: Middle Ear KEY: Nursing Process Step: Assessment MSC:

23. When the patient stares at the black dot on an Amsler grid, what should the nurse ask him to report? a. Any color visible on the grid b. Fading of the edges of the grid c. Any distortion of the grid d. Movement of the black dot

ANS: C Amsler grid, a diagnostic tool for retinal disorders, requires that the patient look at the dot on the grid and report any distortion in the grid lines. PTS: 1 DIF: Cognitive Level: Application REF: Page 1850, Figure 52-3 OBJ: 9 TOP: Aging KEY: Nursing Process Step: Assessment

11. When the worried mother of an accident victim asks the nurse how much circulating blood an average adult male is supposed to have, the nurse replies: a. 8 pints. b. 10 pints. c. 12 pints. d. 14 pints.

ANS: C An average adult male has 12 pints of blood. REF: Page 748

1. Which of these phrases BEST describes an emergency within the context of health care? a. a sudden disruption in normal functioning with serious consequences b. a disturbance in homeostasis that prevents normal body activity c. a medical condition requiring immediate intervention d. a health care crisis occurring when normal care procedures cannot be carried out

ANS: C An emergency within the context of health care is a medical condition requiring immediate intervention. Clients and families define an emergency according to their own perceptions.

25. Which patients would be at greatest risk during surgery? a. 78-year-old taking an analgesic agent b. 43-year-old taking an antihypertensive agent c. 27-year-old taking an anticoagulant agent d. 10-year-old taking an antibiotic agent

ANS: C Anticoagulants alter normal clotting factors and thus increase risk of hemorrhaging. They should be discontinued for 48 hours before surgery. REF: Page 1257, Box 42-3, Table 42-5 TOP: Individual's ability to tolerate surgery

25. Which may indicate a malignant melanoma in a nevus on a patient's arm? a. Even coloring of the mole b. Decrease in size of the mole c. Irregular border of the mole d. Symmetry of the mole

ANS: C Any change in color, size, or texture and any bleeding or pruritus of a nevus deserves investigation. A malignant melanoma is a cancerous neoplasm in which pigment cells or melanocytes invade the epidermis, dermis, and sometimes the subcutaneous tissue. REF: Page 1328, Figure 43-18 TOP: Melanoma

7. A nurse is caring for a patient with pruritus. Which implementation can the nurse perform without a physician's order? a. Apply topical corticosteroids to affected areas. b. Administer an antihistamine. c. Apply lubricant to unbroken skin. d. Bathe the patient in an oatmeal bath.

ANS: C Application of a lotion or lubricant to unbroken skin may be done without an order. DIF: Cognitive Level: Application REF: p. 1194 OBJ: 7 TOP: Pruritus KEY: Nursing Process Step: Implementation

15. A nurse, in conjunction with a patient, establishes a plan to treat the pain associated with arthritis. What is the most effective strategy? a. Avoid exercise to spare painful joints. b. Use narcotics for pain relief. c. Apply warm, moist compresses before doing activity. d. Avoid assistive devices that encourage dependence.

ANS: C Applying heat before exercise loosens the joints and decreases pain. DIF: Cognitive Level: Comprehension REF: p. 952 OBJ: 8 TOP: Arthritis KEY: Nursing Process Step: Planning MSC: NCLEX: Physiological Integrity:

19. The nurse is caring for a 26-year-old patient who was burned 72 hours ago. He has partial-thickness burns to 24% of his body surface area. He begins to excrete large amounts of urine. The nurse should a. slow the IV solution and monitor for burn shock. b. monitor for signs of infection. c. assess for signs of fluid overload. d. raise the foot of the bed and apply blankets.

ANS: C As the blood volume increases, the cardiac output increases to increase renal perfusion. The result includes diuresis. However, a great risk for the patient includes fluid overload because of the rapid movement of fluid back into the intravascular space. REF: Page 1331 TOP: Burns

14. What action should a nurse implement when caring for a patient diagnosed with a compound fracture? a. Limit narcotics for 8 hours after surgery. b. Monitor the patient's respirations every hour. c. Assess for pulses distal to the injury. d. Verify that the patient is not allergic to sulfa.

ANS: C Assessing for pulses distal to the injury is performed to monitor for ineffective tissue perfusion. DIF: Cognitive Level: Application REF: p. 982-983 OBJ: 6 TOP: Care of the Patient after Surgery for a Fracture KEY: Nursing Process Step: Implementation

3. A patient is cleaning the garage and splashes a chemical in his eyes. The initial priority after the chemical burn is to a. transport to a physician immediately. b. cover the eyes with a sterile gauze. c. irrigate with H2O for 15 minutes or longer. d. irrigate with normal saline solution for 1 to 5 minutes.

ANS: C Burns are medically treated with a prolonged, 15- to 20-minute or longer tap-water flush immediately after burn exposure. REF: Page 1864 TOP: Trauma Step: Planning

5. A patient who sustained a simple fracture of the left fibula 7 days earlier asks in what stage of bone healing he might be. What stage of healing should the nurse relay to the patient? a. Hematoma formation b. Ossification c. Callus formation d. Fibrocartilage formation

ANS: C Callus formation occurs at the end of the first week after injury. DIF: Cognitive Level: Comprehension REF: p. 972 OBJ: 2 TOP: Healing Process of Fractures KEY: Nursing Process Step: Implementation

19. A patient in a full body cast (spica) complains of nausea and abdominal distention. What potential complication should a licensed vocational nurse (LVN) suspect? a. Constipation b. Compartment syndrome c. Cast syndrome d. Shock

ANS: C Cast syndrome is an uncommon complication for a person in a spica cast, in which compression of a portion of the duodenum occurs between the mesenteric artery and the spinal column. DIF: Cognitive Level: Analysis REF: p. 977-979 OBJ: 6 TOP: Cast Syndrome KEY: Nursing Process Step: Assessment

19. A male patient brings home a note from the occupational nurse that states, "You have acute bacterial conjunctivitis." This condition is more commonly called a. infection. b. tearing. c. pink eye. d. color blindness.

ANS: C Conjunctivitis is an inflammation of the conjunctiva caused by bacterial or viral infection, allergy, or environmental factors. It is commonly called pink eye. REF: Page 1850 TOP: Conjunctivitis

21. The nurse is assisting with the sponge and instrument count in the operating room. The operative phase in which the nurse is assisting is called the a. perioperative phase. b. preoperative phase. c. intraoperative phase. d. postoperative phase.

ANS: C Counting of sponges, needles, and instruments with the scrub nurse before surgery and before closing the wound is done during the intraoperative phase of the surgery. REF: Page 1260, Box 42-7 TOP: Intraoperative responsibilities

1. A 51-year-old professional tennis instructor is newly diagnosed with osteoarthritis. What is the nurse's best explanation to the patient when asked what this diagnosis means? a. Presence of antibodies in the synovial fluid b. Dislocation of the patella over the tibia c. Degeneration of articular cartilage d. Body's autoimmune response

ANS: C Degeneration of articular cartilage is one of the pathophysiologic changes of arthritis. DIF: Cognitive Level: Knowledge REF: p. 950 OBJ: 6 TOP: Osteoarthritis KEY: Nursing Process Step: Assessment MSC: NCLEX: Physiological Integrity:

19. What should the nurse advise the 20-year-old to do who has been put on cefaclor (Ceclor) for a resistant otitis media? a. Store suspension at room temperature b. Discontinue drug when symptoms abate c. Avoid alcoholic beverages d. Take with meals only

ANS: C Drinking alcohol is discouraged while on Ceclor. The drug should be taken in its entirety and stored in the refrigerator. The drug can be taken with or without meals. PTS: 1 DIF: Cognitive Level: Knowledge REF: Page 1880, Table 52-5 OBJ: 16 TOP: Ceclor KEY: Nursing Process Step: Implementation

20. An older adult patient is at risk for constipation after sustaining a pelvic fracture. Which nutritional suggestion by the nurse is most appropriate? a. Select food with high sodium content. b. Avoid foods high in dietary fiber. c. While immobilized, drink at least 2 to 3 L of fluids daily. d. Include milk products at every meal.

ANS: C During periods of immobilization, a daily fluid intake of 2 to 3 L is recommended to promote bowel and bladder function. Food with sodium causes fluid retention. Dietary fiber helps diminish constipation. DIF: Cognitive Level: Application REF: p. 983 OBJ: 6 TOP: Nutrition Concepts with Fractures KEY: Nursing Process Step: Planning

35. What are the high-risk conditions that may affect perioperative procedures? (Select all that apply): a. Age, health, occupation, mental status b. Financial income, health, nutritional status c. Age, mental state, nutritional status, health d. Occupation, age, nutritional status, health e. Financial Income, occupation, age, health

ANS: C Each system of the body is affected by the patient's age, health, nutritional status, and mental state. DIF: Cognitive Level: AssessmentREF: Page 1260 TOP: Nursing diagnosis

19. What nursing interventions will minimize the effects of venous stasis? a. Pillows under the knee in a position of comfort b. Sitting with the feet flat on the floor c. Early ambulation d. Gentle leg massage

ANS: C Early ambulation has been a significant factor in hastening postoperative recovery and preventing postoperative complications. REF: Page 1288, Box 42-10 TOP: Interventions

17. A client has an increased lab value that is associated with a parasitic skin infection. Which lab test in a white blood cell with differential will indicate this condition? a. basophils b. lymphocytes c. eosinophils d. monocytes

ANS: C Eosinophils are increased in allergic reactions or parasitic infestation. The normal rate for eosiniphils is 1%-3%.

23. What should a nurse be sure to frequently assess when caring for a burn patient with eschar formation around an entire arm? a. Urine output b. Pain level c. Capillary refill d. Breath sounds

ANS: C Eschar that encompasses a limb can compromise circulation. DIF: Cognitive Level: Application REF: p. 1210 OBJ: 5 TOP: Eschar: Impaired Circulation KEY: Nursing Process Step: Planning

20. A nurse is caring for a patient with a recent below-the-knee amputation. What should the nurse recommend to this patient to prevent the loss of calcium and protein? a. Drink 1 to 2 L of fluid daily. b. Ingest at least four milk products each day. c. Ambulate 30 minutes a day. d. Take vitamin supplements daily.

ANS: C Even a small amount of ambulation will decrease the loss of calcium and protein. DIF: Cognitive Level: Application REF: p. 1000 OBJ: 4 TOP: Postoperative Ambulation KEY: Nursing Process Step: Implementation MSC:

24. Which is true about a greenstick fracture? a. Line of the fracture goes across the bone in right angles to the longitudinal axis. b. Periosteum is not torn away from the bone. c. Fracture is incomplete, and one side is bent. d. Fracture occurred in one of the long bones of the body.

ANS: C Greenstick fractures are most commonly seen in children, with the bone broken on one side but only bent on the other. DIF: Cognitive Level: Knowledge REF: p. 971 OBJ: 1 TOP: Fracture Healing KEY: Nursing Process Step: Implementation

6. Four hours after a stapedectomy the patient complains that hearing has not improved at all. What knowledge would the nurse use to shape a response? a. A large percentage of stapedectomies are not successful b. It will take at least 10 days for the graft to heal c. Hearing will not return until edema subsides d. Hearing will improve after irrigation of the ear

ANS: C Hearing improvement will not be noted until edema subsides and the packing is removed. PTS: 1 DIF: Cognitive Level: Application REF: Page 1887 OBJ: 17 TOP: Stapedectomy KEY: Nursing Process Step: Implementation

30. A patient who was admitted after receiving a blow to the head begins to show signs of shock. How should the patient be positioned? a. With the head lower than the body b. Flat with the legs elevated c. Flat on the back d. In a side-lying position

ANS: C If head injuries are suspected, the victim must be kept flat. REF: Page 748

17. A 32-year-old construction worker has suffered a penetrating wound to his right eye. The best intervention for anyone to perform at the scene while waiting to be transported to the hospital is to a. gently remove the object. b. wipe away the blood and tears. c. cover both eyes with a paper cup and tape. d. do nothing; rush to the hospital.

ANS: C Immediately after a penetrating wound injury, both eyes should be covered while transporting the patient to the hospital because both eyes work in synchrony. A Styrofoam cup provides adequate coverage and is readily available. REF: Page 1865, Safety Alert! TOP: Trauma

19. Initial treatment of simple strains and sprains uses the RICE formula, which means: a. regular iced compression of extremity b. radiological examination, isotonic movement, continuity, and edema c. rest, ice, compression, and elevation d. restrain, immobilize, comply with treatment, and eliminate causative factors

ANS: C Initial treatment of simple strains and sprains includes rest, ice, compression, and elevation.

27. Although informed about the proposed surgical procedure, the patient has only vague responses about the postoperative period. A nursing diagnosis at this time would be a. Impaired verbal communication. b. Impaired gas exchange. c. Deficient knowledge, postoperative. d. Acute pain.

ANS: C Knowledge, deficient regarding implications of surgery related to information misinterpretation is a correct nursing diagnosis. REF: Page 1256, Box 42-11 TOP: Nursing process/diagnosis Step: Planning

23. During a nursing history related to a patient's activity-exercise functional health pattern, the patient comments that he cannot bend over and lift an object without becoming dizzy and that he frequently has to stop physical activities because of dizziness and nausea. Which structure of this patient's ear is most likely to have a dysfunction causing his symptoms? a. Middle ear b. Organ of Corti c. Semicircular canals d. Cochlea

ANS: C Labyrinthitis is an inflammation of the labyrinthine canals of the inner ear. REF: Page 1841 TOP: Aging

15. A nurse is caring for a patient that has undergone replantation of a body part. What might the saliva of leeches be used to treat in this patient? a. Inadequate arterial blood flow b. Venous insufficiency c. Venous congestion d. Increased arterial blood flow

ANS: C Leeches are used to treat venous congestion. DIF: Cognitive Level: Comprehension REF: p. 1003 OBJ: 4 | 5 TOP: Replantation KEY: Nursing Process Step:

31. The nurse counsels the 16-year-old boy that playing his music at high volume can result in impairment in hearing related to: a. damaged tympanic membrane. b. protective buildup of cerumen. c. damage of the fine hair cells in the organ of Corti. d. rupture of the oval window.

ANS: C Long-term exposure to loud noises can damage the fine hair cells in the organ of Corti, which causes a conductive hearing loss. PTS: 1 DIF: Cognitive Level: Knowledge REF: Page 1890 OBJ: 12 TOP: Health promotion KEY: Nursing Process Step: Assessment

31. The aging process can be a factor in the patient's smell and taste. Which may contribute to altered nutrition for the elderly? a. Increase in the receptors in the nasal cavities b. Increase in papillae of the tongue c. Medications d. Myopia

ANS: C Medications often affect the taste of food and can contribute to altered nutrition. A decreased number of receptors in the nasal cavities and papillae of the tongue results in changes in smell and taste. Most affected are the sweet and salty tastes. DIF: Cognitive Level: Knowledge REF: Page 1843, Life Span Considerations box TOP: Health promotion

18. A client with otitis media has a nursing diagnosis of Acute pain related to inflammation in the middle ear. The nursing intervention would include: a. administering warm saline flushes to the affected ear b. applying an ice pack to the affected ear every 2 hours c. administering analgesics and antibiotics as ordered d. telling the client that the symptoms will subside in 3 to 4 days

ANS: C Otitis media, an inflammation of the middle ear, is often the cause of temporary conductive hearing loss. Otitis media occurs more frequently in children than in adults. Antibiotics are usually the medication of choice. Analgesics may be prescribed to control pain.

11. A patient with severe psoriasis who is to be treated with the systemic drug methotrexate sodium anxiously asks, "Is this cancer drug safe? Are there some side effects I need to know about?" What is the best response by the nurse? a. "Yes, methotrexate is used to treat cancer and psoriasis, and it has no severe side effects." b. "No, it is not a cancer drug, but you should ask your physician about concerns regarding your therapy." c. "We use this drug to treat many kinds of patients, including patients with cancer. You will have periodic blood tests." d. "I don't know if it is used with patients with cancer, but the drug can be used when conditions are as severe as yours."

ANS: C Methotrexate is an immunosuppressive drug used to treat psoriasis that is nonresponsive to other protocols. Periodic blood tests are performed to assess for leukopenia. The other options either do not answer the patient's question or offer erroneous information. DIF: Cognitive Level: Application REF: p. 1196 OBJ: 6 TOP: Methotrexate Sodium KEY: Nursing Process Step: Implementation

13. A nurse arrives at an accident scene where the victim has just received an electrical burn. The nurse's primary concern is a. the extent and depth of the burn. b. the sites of entry and exit. c. the likelihood of cardiac arrest. d. control of bleeding.

ANS: C Most electrical burns result in cardiac arrest, and the patient will require CPR or astute cardiac monitoring. REF: Page 1333 TOP: Burns

13. A client who is being treated for head trauma has just started to hiccup. What is the FIRST nursing priority? a. Administer prescribed muscle relaxant. b. Administer prescribed sedation. c. Notify the health care provider. d. Obtain vital signs including a neurological screening.

ANS: C Notify the health care provider when a client with head trauma has increased intracranial pressure (ICP). Hiccups, straining, coughing, and sneezing are all activities that increase ICP.

9. A possible goal for a client with a hearing impairment would be, "The client will participate in conversations and other social situations." Which nursing intervention is MOST appropriate to attain this goal? a. consulting social services as needed b. identifying hospital and community resources for people who are hearing impaired c. speaking slowly and distinctly, giving the client time to respond d. using written materials whenever possible to communicate information

ANS: C Nursing intervention for a client with a hearing impairment would include facing the client, speaking slowly and distinctly, and giving the client time to respond.

31. Nursing interventions for treatment of herpes virus infections are: a. Keep the lesions moist to prevent pain. b. Apply cold, moist compresses to lesions. c. Focus on treating symptoms and preventing spread of virus. d. Cleanse lesions with alcohol.

ANS: C Nursing interventions for treating herpes virus infections should focus on treating specific symptoms and preventing spreading of the disease. Lesions should be kept dry. Warm, moist compresses may be applied. DIF: Cognitive Level: ImplementationREF: Pages 1305, 1312 TOP: Viral disorders of skin

6. A 75-year-old patient reports to a nurse that although she has cleaned her ears with cotton-tipped applicators for weeks, she still cannot hear her television unless the volume is loud, and she misses a great deal of conversations. What should the nurse anticipate when examining her ears? a. Otitis externa b. Purulent drainage c. Dry cerumen across the canal d. Pearly tympanic membrane

ANS: C Obstruction of the external canal with cerumen will result in a hearing loss. Cleaning the ears with something such as an applicator will pack the cerumen in the canal. DIF: Cognitive Level: Comprehension REF: p. 1251 OBJ: 6 TOP: External Auditory Canal KEY: Nursing Process Step: Assessment MSC:

7. The older adult patient may not respond to surgical treatment as well as a younger adult because of a. poor skin turgor resulting in dehydration. b. disturbed body image related to surgical incision. c. his or her body's response to physiological changes. d. decreased peristalsis related to general anesthesia.

ANS: C Of specific concern in older adults is the body's response to temperature changes, cardiovascular shifts, respiratory needs, and renal function. REF: Page 1256 TOP: Older adult patient

4. An 80-year-old man with diabetes has had vascular problems with his feet and lower legs for 10 years and is scheduled for a left below-the-knee amputation. Which remark by the patient indicates an understanding of the procedure? a. "I am glad this amputation will end my diabetic problems." b. "After they have hacked my leg, I won't be able to drive." c. "If this heals well, how long until I get a prosthesis?" d. "I hate that my left knee is going to be useless without a foot."

ANS: C Only this option indicates that the patient realizes the extent of the surgery. DIF: Cognitive Level: Comprehension REF: p. 993 OBJ: 4 TOP: Amputation KEY: Nursing Process Step: Evaluation MSC:

3. A patient has herpes zoster (shingles). A local antiviral agent, which is useful in delaying the progression of herpetic diseases, was prescribed. This medication is a. lorazepam (Ativan). b. hydroxyzine (Atarax). c. acyclovir (Zovirax). d. hydrocortisone (Solu-Cortef).

ANS: C Oral and intravenous acyclovir (Zovirax), when administered early, reduces the pain and duration of the virus. REF: Page 1308 TOP: Anti-infective Step: Planning

12. During the admission of a patient scheduled for an amputation, a patient relates that she is a practicing Orthodox Jew. What arrangements are appropriate for the nurse to make for this patient? a. A veil should cover the amputated part. b. A rabbi must be present for the surgery. c. The amputated part should be buried. d. A family member should be present to read the Torah.

ANS: C Orthodox Jews bury all body parts. DIF: Cognitive Level: Application REF: p. 1000 OBJ: 4 TOP: Nursing Care for the Orthodox Jew KEY: Nursing Process Step: Implementation MSC:

1. A nurse reads in a patient's history that the patient has experienced otalgia. How should the nurse interpret this term? a. Difficulty hearing b. Buildup of cerumen c. Ear pain d. Ringing in the ears

ANS: C Otic- is the root word for ear, and -algia is the root term for pain of any type. DIF: Cognitive Level: Knowledge REF: p. 1252 OBJ: 5 TOP: Definitions KEY: Nursing Process Step: Assessment MSC:

17. The nurse explains to the client that the condition in which inflammation occurs in the middle ear and can cause temporary conductive hearing loss is: a. acoustic neuroma b. Ménière's disease c. otitis media d. otosclerosis

ANS: C Otitis media, an inflammation of the middle ear, is often the cause of temporary conductive hearing loss. Otitis media occurs more frequently in children than in adults. Antibiotics are usually the medication of choice.

14. What is the preferred surgical technique for improving hearing loss due to otosclerosis? a. meatoplasty b. otothesis c. stapedectomy d. tympanoplasty

ANS: C Otosclerosis is a conductive hearing loss that occurs when the ossicles become softened, spongy, highly vascular, and partially or totally fixed. Treatment options include hearing aids or surgery to remove the stapes (stapedectomy) and implant a plastic prosthesis. Some clients may opt to skip treatment.

13. What should a patient who has had a cataract repair avoid? a. The use of eye patches b. The use of sunglasses c. The lifting of heavy objects d. Reading for long periods of time

ANS: C Postcataract patients should avoid any activity that increases the intraocular pressure, such as lifting heavy objects, stooping, and bending. PTS: 1 DIF: Cognitive Level: Comprehension REF: Page 1860 OBJ: 11 TOP: Blindness KEY: Nursing Process Step: Planning

32. Ideally, preop teaching should be done a. immediately before surgery to eliminate fear. b. 2 months in advance so the patient can prepare. c. 1 to 2 days before the surgery when anxiety is not as high. d. in the surgical holding area.

ANS: C Preop teaching is provided 1 to 2 days prior to surgery when anxiety is low. DIF: Cognitive Level: ImplementationREF: Page 1260 TOP: Nursing diagnosis

11. A nurse is collecting a health history from a patient with a connective tissue disease. What is the most important inquiry by the nurse? a. Family history of atherosclerosis b. Last time the patient had his or her blood tested c. History of a prior injury to a specific body part d. Family history of a fracture

ANS: C Previous injuries may be relevant to a patient's current problem. DIF: Cognitive Level: Comprehension REF: p. 940 OBJ: 3 TOP: Connective Tissue Disease Assessment KEY: Nursing Process Step: Assessment MSC: NCLEX: Health Promotion and Maintenance:

21. Which finding should produce the most concern when performing pin care for a patient with an external fixator? a. Crusts around the pin b. Serous drainage on the dressing c. Purulent drainage d. Absence of pain

ANS: C Purulent drainage is the only abnormal finding. DIF: Cognitive Level: Comprehension REF: p. 976 OBJ: 5 | 6 TOP: Pin Care KEY: Nursing Process Step: Assessment

5. A 94-year-old patient is receiving gentamicin sulfate (Garamycin) in a continuous intravenous (IV) infusion. The nurse adds to the nursing care plan the diagnosis "Risk for injury." What nursing action should be implemented? a. Pull side rails in place. b. Assist with ambulation. c. Measure intake and output. d. Provide for a possible seizure.

ANS: C Reduced urine output would cause the drug to stay in the system rather than being excreted, which could result in a drug saturation. Gentamicin is ototoxic and can cause hearing impairment. DIF: Cognitive Level: Application REF: p. 1253 OBJ: 7 TOP: Gentamicin KEY: Nursing Process Step: Assessment MSC:

32. Parasitic skin diseases are identified as pediculosis or scabies. The difference between pediculosis and scabies is a. pediculosis only occurs in poor living conditions. b. scabies only occurs when there is poor hygiene. c. scabies is caused by mites that burrow into the skin. d. lice is caused by mites that attach themselves to bare areas of the skin.

ANS: C Scabies is caused by mites that penetrate the skin by burrowing. Pediculosis and scabies occur in any environment or in anyone using good personal hygiene. Lice attach themselves to the shaft of hair follicles. REF: Page 1325 TOP: Parasite disorders of the skin

6. A client has second-and third-degree burns to the legs and feet. The nurse assesses the client has an increased heart rate, decreased blood pressure, and cool and clammy skin. The nurse should recognize this client is in what type of shock? a. anaphylactic b. cardiogenic c. hypovolemic d. toxic

ANS: C Shock, a condition of profound hemodynamic and metabolic disturbance characterized by inadequate tissue perfusion and inadequate circulation to the vital organs, must be recognized and immediately treated. Hypovolemic shock is easily recognizable when bleeding is external but more difficult when the bleeding is internal.

24. A patient has been admitted to the hospital with burns to his upper chest. The nurse notes singed nasal hairs. It would be important for the nurse to assess this patient frequently for a. decreased activity. b. bradycardia. c. respiratory complications. d. hypertension.

ANS: C Signs and symptoms of inhalation injury include singed nasal hairs. Breathing difficulties may take several hours to occur. REF: Page 1331 TOP: Burns

2. A nurse notes that a client's skin is loose, thin, and wrinkled. Which of these skin parameters has the nurse assessed? a. integrity b. moisture c. texture d. turgor

ANS: C Skin texture is normally soft and smooth; the nurse should note any variations such as skin that is wrinkled, rough, or scaling.

34. Southeast Asian and Native American patients often do not make eye contact when preoperative teaching is being performed because a. they aren't educated. b. they aren't paying attention. c. they believe eye contact is disrespectful. d. they believe they are superior to the nurse.

ANS: C Southeast Asians and Native Americans may believe eye contact is disrespectful. REF: Page 1258, Cultural Considerations box OBJ: N/A TOP: Nursing diagnosis Step: Assessment

15. An excited mother of a teenage boy with severe acne furiously reports to the nurse, "I've told him a thousand times he should bathe more often! I've kept after him about all that junk food he eats. I jump on him when I see him squeezing his zits. I tried to get him to scrub his face three times a day!" Which statement indicates the most likely cause of the boy's acne? a. Poor personal hygiene b. Ingestion of junk food c. Squeezing lesions d. Need for facial scrubs

ANS: C Squeezing the lesions may cause them to spread and push the infection deeper into the follicles. Poor personal hygiene, eating junk food, and the need for facial scrubs are myths. DIF: Cognitive Level: Application REF: p. 1200 OBJ: 7 TOP: Acne KEY: Nursing Process Step: Planning

3. The skin of a client's feet is very taut with edema. Which of these skin parameters has the nurse assessed? a. integrity b. texture c. turgor d. vascularity

ANS: C Turgor is an assessment of skin hydration. Normally skin moves freely. A pinched fold of skin returns to normal position within 3 seconds. Skin will be taut with edema, slack with dehydration, and rigid in some diseases such as scleroderma.

20. When a patient arrives in the emergency department with a sucking wound to the left chest, the nurse should: a. place several layers of gauze dressing over the wound. b. place the patient in a supine position. c. cover the wound with an airtight dressing taped on three sides. d. turn the patient to the left side.

ANS: C Sucking chest wounds should be dressed with a flutter dressing so that air can escape the pleural space, but no more air can be sucked in. REF: Page 752 TOP: Sucking chest wounds

24. An appendectomy during a hysterectomy would be classified as a. major, emergency, diagnostic. b. major, urgent, palliative. c. minor, elective, ablative. d. minor, urgent, reconstructive.

ANS: C Surgery is classified as elective, urgent, or emergency. Surgery is performed for various purposes, which include diagnostic studies, ablation (an amputation or excision of any part of the body or removal of a growth or harmful substance), and palliative (therapy to relieve or reduce intensity of uncomfortable symptoms without cure), reconstructive, transplant, and constructive purposes. REF: Page 1254, Table 42-1 TOP: Types of surgery

4. A daughter of an older adult patient who has just returned from surgery is distressed about her father's pale, cold hands and feet. What is the best response by the nurse after covering the patient with an extra blanket? a. "Don't be concerned. It is quite cold in the operating room. Your dad will be warm in a minute." b. "Older patients like your dad get a little shocky during surgery." c. "When patients have blood loss during surgery, superficial vessels close off temporarily, resulting in cold extremities." d. "We are watching the disturbed circulation in your dad's hands and feet very carefully."

ANS: C The 10% of the blood network that is in the skin can be reduced by constriction and shunted to the vital organs. DIF: Cognitive Level: Application REF: p. 1174 OBJ: 1 TOP: Skin Blood Reservoir KEY: Nursing Process Step: Implementation

10. The nurse clarifies that the difference between a photorefractive keratectomy (PRK) and a laser in-situ keratomileusis (LASIK) is that a LASIK: a. reshapes the central cornea. b. makes partial-thickness radial incisions in the cornea. c. removes some internal layers of the cornea. d. implants intracorneal rings.

ANS: C The LASIK procedure removes some of the internal layers of the cornea affecting the central zone of vision. PTS: 1 DIF: Cognitive Level: Knowledge REF: Page 1853 OBJ: 11 TOP: Visual acuity KEY: Nursing Process Step: Implementation

3. The Patient's Bill of Rights states that a patient must give his or her permission for any specific test or procedure to be performed. What is the legal term for this permission? a. Verbal consent b. Medical documentation c. Informed consent d. Informed decision

ANS: C The Patient's Bill of Rights affirms that the patients must give informed consent (permission obtained from the patient to perform a specific test or procedure) before the beginning of any procedure. REF: Page 1260 TOP: Informed consent

29. When the nurse arrives on the scene of a fire, the first thing to do for a burn victim is to: a. apply dressings. b. cover with a blanket. c. cool the burn immediately. d. apply topical ointment.

ANS: C The burn should be cooled immediately to stop the burning process. REF: Page 761 TOP: Burns

24. The patient with heat stroke has been undressed and treated with cold packs and a fan. The patient's temperature is now down to 101.2° F. When the patient starts to shiver, the emergency department nurse should: a. raise the head of the bed. b. offer warm liquids. c. remove cold packs and fan. d. continue with cooling interventions.

ANS: C The cooling techniques have caused the patient to shiver, which will increase the patient's temperature. REF: Page 757 TOP: Heat stroke

12. What nursing action should be implemented when irrigating a patient's ear? a. Straighten the ear canal and irrigate with a large-tipped bulb syringe. b. Direct the solution to the middle of the canal to avoid damaging the ear. c. Use a body temperature solution and have the patient hold a basin under the ear while directing the solution toward the top of the canal. d. Repeat the irrigation with hotter water.

ANS: C The irrigation is done with warm water using a small-tipped syringe. The flow is directed upward. If the cerumen does not wash out, the procedure can be repeated but with the same water temperature. DIF: Cognitive Level: Application REF: p. 1257 OBJ: 3 TOP: Irrigation KEY: Nursing Process Step: Implementation MSC:

8. A patient has acne vulgaris. When the nurse explains this condition, it is most important to a. stress the importance of strict hygiene. b. discuss the connection of diet and stress. c. explore how this condition is affecting his self-image and lifestyle. d. describe in detail the proper use of prescribed medication.

ANS: C The nurse must assess and consider what acne means to a person. Most patients acknowledge that acne affects their self-image. REF: Page 1319 TOP: Self-image

6. A patient is scheduled for a stapedectomy. Appropriate postoperative teaching should include: a. Hourly changing cotton from external ear canal b. Gently blowing both nares simultaneously c. Teaching patient to open mouth when sneezing or coughing d. Limiting activities for 3 weeks

ANS: C The nurse must include patient teaching about opening the mouth when sneezing or coughing or blowing the nose gently on one side at a time for 1 week. REF: Page 1880, Patient Teaching box TOP: Stapedectomy

13. A patient has been blind for the past 10 years. He is hospitalized with congestive heart failure (CHF). In the care of a long-term blind individual, it is important to a. speak loudly to get the patient's attention. b. schedule a consultation with an occupational therapist to teach activities of daily living. c. announce when you enter and leave the room. d. initiate a referral to the Department of Health and Human Services.

ANS: C The nurse should announce when entering or leaving the room, so that a blind person is not put in the position of talking to someone who is no longer there. REF: Page 1846, Box 53-1 TOP: Blindness Step: Planning

26. A visitor in the hospital slips and falls. The arm appears dislocated and the visitor is unable to move it. The nurse's first action is to: a. apply cold packs. b. check range of motion. c. splint the arm. d. apply an Ace® bandage.

ANS: C The nurse should splint the arm where it lies and not attempt to move or rearrange the limb. REF: Pages 759-760 TOP: Fractures

15. What significant instruction should a nurse include to a patient being discharged after ear surgery? a. Use stool softeners with caution. b. Assume your usual activities. c. Avoid blowing your nose. d. Shampoo your hair with baby shampoo.

ANS: C The patient should avoid blowing the nose to prevent back pressure in the eustachian tube. The patient should take stool softeners, limit activity until balance returns, and delay shampooing. DIF: Cognitive Level: Application REF: p. 1259-1260 OBJ: 7 TOP: Nursing Diagnosis and Outcome Criteria KEY: Nursing Process Step: Implementation MSC:

3. A patient with osteoarthritis in both knees and shoulders states that she cannot manage her household tasks without pain. Which is the most appropriate nursing diagnosis for this patient? a. Ineffective coping, related to pain b. Risk for disuse syndrome c. Impaired physical mobility, related to pain d. Activity intolerance, related to pain

ANS: C The patient's concern is not so much the pain but her difficulty in accomplishing household tasks. DIF: Cognitive Level: Application REF: p. 953 OBJ: 8 TOP: Osteoarthritis KEY: Nursing Process Step: Planning MSC: NCLEX: Physiological Integrity:

5. The pigment that gives skin its color is called: a. vitiligo b. sebum c. melanin d. nevi

ANS: C The pigment that gives the skin its color is called melanin. The more melanin present, the darker the skin color. Exposure to ultraviolet light (sun) causes an increase in the production of melanin, which darkens (tans) the skin and provides some protection against the harmful effects of sun-tanning.

23. A patient has been stung by a bee and is brought to the emergency department. The nurse observes the sting site and identifies that the stinger is still in the skin. The nurse should: a. remove it with sterile tweezers. b. soak the area with a cold compress. c. scrape the stinger with the side of a knife. d. squeeze the surrounding tissue to expel the stinger.

ANS: C The stinger should be removed with the side of a knife by scraping to avoid forcing more venom into the skin. REF: Page 755 TOP: Wounds

9. What special precaution should a nurse implement when assisting with the application of a short arm plaster cast? a. Dampen the skin to make the stockinette adhere. b. Tape the arm before applying the stockinette. c. Smooth the stockinette to prevent a pressure ulcer. d. Roll the stockinette tightly above and below the margins of the cast.

ANS: C The stockinette is smoothed on the limb before applying the casting material to help reduce the threat of a pressure ulcer. DIF: Cognitive Level: Application REF: p. 977 OBJ: 5 TOP: Cast Care KEY: Nursing Process Step: Implementation

8. An obese client may have several inches of: a. dermis b. epidermis c. subcutaneous tissue d. vellus

ANS: C The subcutaneous tissue is primarily connective and adipose (fatty) tissue. Here the skin is anchored to muscles and bones. An individual's nutritional status and genetic makeup dictate the amount of subcutaneous tissue present. Emaciated persons have very little subcutaneous tissue, whereas obese persons may have several inches of subcutaneous tissue.

5. While inspecting the tympanic membrane with an otoscope, the nurse should expect to see which shape of the membrane? a. convex b. bulging c. concave d. striated

ANS: C The tympanic membrane is normally concave on otoscopic exam, so a convex or bulging tympanic membrane is an important sign of an acute infectious process.

3. Functions of the middle ear include: a. creation of cerumen b. equilibrium c. pressure equalization d. closure of the oval window

ANS: C The tympanic membrane is the boundary between the outer and middle ear. The three bones (ossicles) of the middle ear are the malleus (hammer), incus (anvil), and stapes (stirrup). The malleus is attached to the tympanic membrane; the incus joins the malleus to the stapes, which rests on the oval window. The vibration created by sound waves passes through the outer ear canal to the tympanic membrane and then to these three bones. The eustachian tube opens into the pharynx from the middle ear to equalize pressure on both sides of the eardrum.

2. Which structure is located within the middle ear? a. vestibule b. semicircular canals c. stapes d. oval window

ANS: C The tympanic membrane is the boundary between the outer ear and middle ear. The three bones (ossicles) of the middle ear are the malleus (hammer), incus (anvil), and stapes (stirrup). The malleus is attached to the tympanic membrane; the incus joins the malleus to the stapes, which rests on the oval window. The vibration created by sound waves passes through the outer ear canal to the tympanic membrane and then to these three bones. The eustachian tube opens into the pharynx from the middle ear to equalize pressure on both sides of the eardrum.

4. Once CPR has been initiated at the accident site, it can only be terminated when the: a. victim is clinically dead. b. victim is brain dead. c. paramedics arrive. d. rescuer perceives CPR is futile.

ANS: C There is a moral obligation to continue CPR once it has been initiated unless the rescuer is exhausted and cannot continue, trained medical personnel take over CPR, or a licensed physician pronounces the victim dead. REF: Page 740

13. A patient with bilateral avascular necrosis of the hips is to walk with crutches using a four-point gait for 6 weeks after her bone decompression surgeries. Which statement would indicate that the patient understands this technique? a. "The axillary bars on the crutches should support my weight when I walk." b. "I will move both crutches and then swing my legs to the crutches—2 and 2 equals 4!" c. "I will move my right crutch and then my left leg and then the left crutch and my right leg." d. "I will move both crutches and then swing my legs through the crutches together."

ANS: C This option describes the correct sequence for a four-point gait, which allows bearing of weight and one foot to be placed in front of the other. DIF: Cognitive Level: Comprehension REF: p. 981 OBJ: 5 TOP: Crutch Walking KEY: Nursing Process Step: Evaluation

1. When administering first aid in emergency situations, the nurse must first survey victims for severity of injuries. This is called: a. the Good Samaritan Law. b. an emergency interview. c. triage. d. taking vital signs.

ANS: C This process of patient classification is called triage. REF: Page 739

6. A patient, age 46, reports to his physician's office with urticaria and papules on his hands and arms. He says, "It itches so badly." In assessing the patient, the nurse should gather data regarding recent a. travel to foreign countries. b. upper respiratory tract infection. c. changes in medication. d. contact with people who have an infectious disease.

ANS: C Urticaria is the term applied to the presence of wheals or hives in an allergic reaction commonly caused by drugs, food, insect bites, inhalants, emotional stress, or exposure to heat or cold. REF: Page 1317 TOP: Urticaria

22. The verbalization of microvascular precautions is a criterion for measuring the achievement of which outcome? a. Adequate circulation in the replanted limb b. Pain relief c. Patient knowledge of therapeutic measures d. Adjustment to change in appearance and function

ANS: C Verbalizing the information related to microvascular precautions is evidence of the patient's achievement of having acquired the necessary knowledge. DIF: Cognitive Level: Knowledge REF: p. 1002 OBJ: 5 TOP: Nursing Care Planning KEY: Nursing Process Step: Evaluation MSC:

6. When assessing the adult victim for pulselessness, the CPR rescuer should palpate the most reliable and accessible pulse, which is the: a. radial. b. brachial. c. carotid. d. femoral.

ANS: C When assessing the adult victim for pulselessness, the most reliable and accessible pulse is the carotid. REF: Page 742

16. The nurse acknowledges that all preoperative nursing interventions have been performed by signing which document? a. Nurse's notes b. Anesthesia record c. Preoperative checklist d. Physician's order sheet

ANS: C When the nurse signs the preoperative checklist, that nurse assumes responsibility for all areas of care included on the list. REF: Page 1277, Figure 42-10 TOP: Preoperative checklist

28. When assessing a patient who has suffered a burn injury, the nurse classifies the burn as a deep partial-thickness burn based on the observation of: a. painful reddened skin. b. charred skin with milky-white areas. c. erythema and blisters. d. erythema, pain, and swelling.

ANS: C With deep partial-thickness burns, blister formation may be seen with erythema. REF: Page 761 TOP: Burns

1. The patient is 38 years old and is in her second postoperative day after placement of an intramedullary rod in her left femur. She is receiving analgesia via a patient- controlled analgesia (PCA) device. The inappropriate intervention related to caring for a patient with a PCA is: a. Maintaining the system. b. Recording activations of the system. c. Administering the analgesia to the patient. d. Monitoring the patient's pain.

ANS: C With the PCA system of medication administration, the patient can self-administer an analgesic by pressing a control button. The nurse should not give medication doses by pushing the control button. REF: Page 1286 TOP: Medication administration

3. A nurse is conducting a safety seminar and reminds the audience that upper extremity amputations are most frequently caused by trauma. Which population has the highest incidence of this type of amputation? a. School-aged girls b. School-aged boys c. Young men d. Young women

ANS: C Young men are at greater risk from work trauma because they are traditionally the ones working with farm and heavy machinery. DIF: Cognitive Level: Comprehension REF: p. 990 OBJ: 1 TOP: Incidence KEY: Nursing Process Step: Implementation MSC:

37. What should the nurse do when assisting a blind person to walk in an unfamiliar hospital environment? (Select all that apply.) a. Discourage the use of the cane b. Advise the patient to walk quickly c. Describe the surroundings d. Encourage the patient to ask for verbal cues e. Place patient hand on nurse's shoulder or elbow

ANS: C, D, E The patient should be given verbal cues about the environment. Allow the patient to hold the nurse's shoulder or elbow while the nurse walks in front, and encourage the use of a cane to let the patient "examine" the boundaries and obstacles. PTS: 1 DIF: Cognitive Level: Application REF: Page 1851 OBJ: N/A TOP: Assisting blind to walk KEY: Nursing Process Step: Implementation

38. During primary survey assessment of a burn patient, the nurse checks for early signs of carbon monoxide poisoning, which include: (Select all that apply.) a. dizziness. b. urticaria. c. vomiting. d. headache. e. vertigo. f. unsteady gait.

ANS: C, D, F Early signs of carbon monoxide poisoning include headache, nausea, vomiting, and unsteady gait. REF: Page 1333 TOP: Carbon monoxide

36. Place the steps in order for the medical treatment during the emergent phase of burn management from the list below. Place a comma between each answer choice (a, b, c, d, etc.). a. Transport victim to hospital. b. Cover victim with clean cloth or sheet. c. Stop, drop, and roll. d. Remove all now-adherent clothing and jewelry. e. Provide an open airway. f. Control any bleeding.

ANS: C, E, F, D, B, A REF: Page 1333 TOP: Burns

17. Which nursing interventions would be appropriate after a wound evisceration? a. Place the patient in high Fowler's position. b. Give the patient fluids to prevent shock. c. Replace the dressing with sterile fluffy pads. d. Apply a warm, moist normal saline sterile dressing.

ANS: D Cover the wound with a sterile towel moistened with sterile physiological saline (warm). REF: Page 1284, Figure 42-15 TOP: Postoperative interventions

18. A machinist visits the industrial nurse's clinic with a deep laceration of the thigh. The nurse should initially: a. splint the thigh and apply tape to approximate the edges. b. apply ice and a pressure dressing to the thigh. c. give a tetanus booster injection. d. wash the laceration with an antiseptic.

ANS: D Lacerations should be cleaned thoroughly and bandaged to approximate the edges. REF: Page 752 TOP: Wounds

27. The patient is brought to the emergency department after having fractured his arm 12 hours ago. The arm is very edematous from the fingers to the elbow, and the patient cannot move it. Initially the nurse should: a. test range of motion. b. take the vital signs. c. place ice packs on the arm. d. check fingers for capillary refill.

ANS: D Swelling from the fracture can impede circulation. REF: Page 759 TOP: Injuries

2. The patient tells the nurse that he is legally blind. This information provides the nurse with which information to use in planning care? a. No vision enhancement techniques would be appropriate for this patient, because he is totally blind. b. This patient probably has some light perception, but no usable vision. c. This patient has some usable vision, which enables him to function at an acceptable level. d. Further questioning is needed to determine how this patient's visual impairment affects his normal functioning.

ANS: D "Legal blindness" refers to individuals with a maximum visual acuity of 20/200 with corrective eyewear and/or visual field sight capacity reduced by 20 degrees. Categories have been established to help determine the exact extent of the vision loss and what assistive measures are appropriate for the individual. The nurse will need more information as to the exact extent of the vision loss for this patient. REF: Page 1845 TOP: Blindness Step: Planning

2. The patient tells the nurse that he is legally blind. How would this information impact the nurse's plan of care for this patient? a. The patient would be considered totally blind. b. This patient probably has some light perception, but no usable vision. c. This patient has some usable vision, which enables function at an acceptable level. d. The nurse would need to determine how this patient's visual impairment affects normal functioning.

ANS: D "Legal blindness" refers to individuals with a maximum visual acuity of 20/200 with corrective eyewear and/or visual field sight capacity reduced by 20 degrees. Categories have been established to help determine the exact extent of the vision loss and what assistive measures are appropriate for the individual. The nurse will need more information as to the exact extent of the vision loss for this patient. PTS: 1 DIF: Cognitive Level: Analysis REF: Page 1851 OBJ: N/A TOP: Legal blindness KEY: Nursing Process Step: Planning

23. Which postoperative observation must be reported to the physician immediately? a. Brownish-red drainage on the dressing, which is damp b. Respirations of 20 breaths/min c. Pulse of 72 beats/min d. Bright-red bleeding

ANS: D Bright-red bleeding is not expected and indicates hemorrhage. Direct pressure should be applied over the dressing and the physician or charge nurse should be notified. DIF: Cognitive Level: Comprehension REF: p. 997 OBJ: 4 TOP: Complications KEY: Nursing Process Step: Assessment MSC:

12. A nurse is told that a patient has a compound comminuted fracture. What characteristic of the bone in this type of fracture causes the nurse to be concerned? a. It is bent but not completely broken, and the bent piece protrudes through the skin. b. It is compressed, and bone pieces protrude through the skin. c. It is twisted, and the fragments are separated. d. It is broken into two or more pieces, and bone fragments protrude through the skin.

ANS: D A compound comminuted fracture is a severe fracture with the bone broken in two or more pieces, with the pieces broken into small fragments and a portion of the bone protruding through the skin. DIF: Cognitive Level: Comprehension REF: p. 971-972 OBJ: 1 TOP: Fracture Types KEY: Nursing Process Step: Assessment

18. At the internist's office, a client is being treated for an infected blister on a heel. The physician is attempting to determine what microorganism is causing the infection. Which test will be ordered? a. ESR b. c-reactive protein c. WBCs d. culture and sensitivity (C & S)

ANS: D A culture and sensitivity (C & S) aids in diagnosis of an infection and determines the offending microorganism. Sensitivity testing identifies which antibiotics will kill or inhibit growth of the microorganism.

3. When the nurse is assessing victims in an emergency situation, the first priority is to assess for: a. hemorrhage. b. fractures. c. mobility. d. abnormal breathing.

ANS: D A life-threatening situation of the highest priority is arrested or abnormal breathing. REF: Page 739

16. A patient with diabetes says that he needs a hearing aid because he cannot hear well, and everything sounds garbled and distant. What type of hearing loss should the nurse suspect? a. Mixed hearing loss b. Conductive hearing loss c. Central hearing loss d. Sensorineural hearing loss

ANS: D A patient with long-term diabetes may have a sensorineural hearing loss that is not helped by hearing aids. DIF: Cognitive Level: Comprehension REF: p. 1261 OBJ: 5 TOP: Types of Hearing Loss KEY: Nursing Process Step: Assessment MSC:

11. A patient has an infectious/inflammatory process of the eyelid. The primary goal of nursing intervention is a. administering antibiotics. b. flushing the eye with sterile ophthalmic solution. c. maintaining bed rest. d. preventing further infection.

ANS: D A primary objective of nursing care for the patient with an infectious or inflammatory process of the eyelids is prevention of the spread of infection. Hand hygiene is essential before contact with the eye. REF: Page 1850 TOP: Infectious/inflammatory disorders Step: Planning

2. A 73-year-old patient with diabetes was admitted for below-the-knee amputation of his right leg. Removal of his right leg is an example of which type of surgery? a. Palliative b. Diagnostic c. Reconstructive d. Ablative

ANS: D Ablative is a type of surgery where an amputation, excision of any part of the body, or removal of a growth and harmful substance is performed. REF: Page 1254, Table 42-1 TOP: Types of surgery

21. When the eye adjusts to seeing objects at various distances, it is called a. PERRLA. b. refraction. c. focusing. d. accommodation.

ANS: D Accommodation: The eye is able to focus on objects at various distances. REF: Page 1840 TOP: Aging

16. What is the MOST common treatment for acoustic neuroma? a. pharmacological treatment with antihistamines b. placement of a polyethylene tube in the eardrum c. stapedectomy by laser d. surgical excision of the tumor

ANS: D Acoustic neuroma is a slow-growing and usually benign tumor of the vestibular portion of the inner ear. The main symptoms are dizziness, tinnitus, and hearing loss. Treatment includes surgical excision of the tumor.

13. A nurse is educating an osteoporotic patient taking alendronate (Fosamax).Which instruction should the nurse stress? a. Take the drug after breakfast. b. Avoid the use of supplemental vitamin D. c. Decrease fluid intake. d. Sit or stand for 30 minutes after administration.

ANS: D After taking the drug Fosamax with 8 oz of fluid, the patient should sit or stand for 30 minutes so the drug will be evenly distributed. The drug is taken on an empty stomach. DIF: Cognitive Level: Application REF: p. 948 OBJ: 5 TOP: Fosamax KEY: Nursing Process Step: Implementation MSC: NCLEX: Physiological Integrity:

19. A nurse organizes a nursing care plan on the nursing diagnosis of "Acute pain, related to postherpetic neuralgia." Which is the least appropriate implementation? a. Give antiviral medication as prescribed. b. Generously administer pain medication. c. Offer guided imagery or distraction techniques. d. Have the patient ambulate several times daily.

ANS: D Ambulation certainly is not helpful for the pain. Very little helps the neuralgic pain except direct implementation. DIF: Cognitive Level: Application REF: p. 1202 OBJ: 7 TOP: Herpes Zoster KEY: Nursing Process Step: Planning

17. A patient who has a below-the-elbow prosthesis shows the home health care nurse the residual limb, which is red, edematous, and warm to the touch. What should the nurse instruct this patient to do? a. Apply soothing lotion to the residual limb before replacing the prosthesis. b. Dampen the prosthetic limb sock to hydrate and cool the residual limb. c. Pad the socket with lamb's wool and replace the prosthesis. d. Leave the prosthesis off and notify physician.

ANS: D An inflamed residual limb suggests an infection. The prosthesis should be removed, the limb should be gently cleansed with soap and water, and the physician should be notified. DIF: Cognitive Level: Application REF: p. 1000 OBJ: 5 TOP: Infection KEY: Nursing Process Step: Implementation MSC:

13. A nurse assesses an accident victim who has bright red blood spurting from a laceration on his right forearm. After applying direct pressure and elevating the limb, the nurse should apply pressure to the right: a. subclavian artery. b. radial artery. c. ulnar artery. d. brachial artery.

ANS: D Arterial bleeding is characterized by the spurting of bright red blood and can be controlled by direct pressure, elevation, and indirect pressure on the appropriate pressure point. The brachial artery is the closest pressure point to the injury. REF: Page 749, Figure 24-12

7. A patient reports that her hearing loss has become more severe over the past 3 months. The clinic nurse makes arrangements for an evaluation for a hearing aid. What health care provider should provide this service? a. Otologist b. Otolaryngologist c. Audiometrist d. Audiologist

ANS: D Audiologists assess patients for hearing aids. The other specialists treat ear, nose, and throat (ENT) disorders. DIF: Cognitive Level: Knowledge REF: p. 1253 OBJ: 7 TOP: Audiometry KEY: Nursing Process Step: Implementation MSC:

3. One of the housekeepers splashes a chemical in the eyes. What should be the first priority? a. Transport to a physician immediately b. Cover the eyes with a sterile gauze c. Irrigate with H2O for 5 minutes d. Irrigate with normal saline solution for 20 minutes

ANS: D Burns are medically treated with a prolonged, 15- to 20-minute or longer normal saline flush immediately after burn exposure. PTS: 1 DIF: Cognitive Level: Analysis REF: Page 1871 OBJ: 11 TOP: Chemical burn of eye KEY: Nursing Process Step: Implementation

17. An 80-year-old patient comes to the emergency department with extreme pain and itching in the hip and leg and has herpetic vesicular lesions on the left hip. What should the nurse inquire about patient exposure to? a. HSV, type 1 b. HSV, type 2 c. Smallpox d. Chickenpox

ANS: D Chickenpox is a virus that lies latent in the neural sheath and can be activated as shingles in older adults. DIF: Cognitive Level: Application REF: p. 1201-1202 OBJ: 7 TOP: Herpes Zoster KEY: Nursing Process Step: Assessment

16. A nurse is providing the open method of treatment for a patient who is 52 years old with burns to her lower extremities. It would be important for the nurse to a. change the dressing using good medical asepsis. b. provide an analgesic immediately after the dressing change. c. perform circulation checks every 2 to 4 hours. d. keep the room temperature at 85° F (24.4° C) to prevent chilling.

ANS: D Chilling may be controlled by keeping the room temperature at 85° F (24.4° C). REF: Page 1336 TOP: Burn treatment

22. A client who has severe third-degree burns must be "packaged," which means the nurse will: a. prepare for application of plaster immobilization cast b. prepare to transport the client to an isolation area c. provide postmortem care and transfer the body to the morgue d. wrap affected areas in sterile dressings, start an IV, and insert specified tubes and catheters

ANS: D Clients with major burns are usually transported to a burn center after the arterial blood gases (ABGs) have been stabilized, where the nurse will wrap affected areas in sterile dressings, start an IV, and insert specified tubes and catheters.

14. Clubbing of the nails occurs with which of the following clinical conditions? a. iron deficiency anemia b. acute severe anemia c. infection of the nail d. long-standing hypoxia

ANS: D Clubbing occurs when long-standing hypoxia is present, particularly with cyanotic heart disease and advanced chronic obstructed pulmonary disease.

7. In which type of hearing loss is the inner ear or cochlear portion of the eighth cranial nerve abnormal or diseased? a. common b. conductive c. mixed d. sensorineural

ANS: D Conductive hearing loss is an inability of the sound waves to reach the inner ear. Sensorineural hearing loss involves an abnormal or diseased inner ear or auditory nerve. The sensorineural hearing loss associated with aging is called presbycusis.

6. A type of anesthesia that requires a depressed level of consciousness is a. regional anesthesia. b. specific anesthesia. c. optional sedation. d. conscious sedation.

ANS: D Conscious sedation is routinely used for procedures that do not require complete anesthesia but rather a depressed level of consciousness. REF: Page 1276 TOP: Conscious sedation

10. An older patient who sustained a fractured hip and femur in a motor vehicle accident is to be in Russell traction for several weeks. What should be the focus of care for the nurse? a. Offering frequent distractions b. Encouraging nutrition c. Offering pain relief d. Preventing deep vein thrombosis (DVT)

ANS: D DVT is a threat for the person who is going to experience lengthy periods of inactivity. Although nutrition, pain relief, and boredom will be nursing concerns as well, the prevention of DVT is the priority. DIF: Cognitive Level: Application REF: p. 974 OBJ: 3 | 5 TOP: Long-Term Complications of Fractures KEY: Nursing Process Step: Evaluation

6. Where is the limb severed with an elbow disarticulation? a. Just above the elbow joint b. Just below the elbow joint c. Between the shoulder and elbow d. Through the elbow joint

ANS: D Disarticulations sever the limbs through the joints. DIF: Cognitive Level: Knowledge REF: p. 990 OBJ: 2 TOP: Type of Amputation KEY: Nursing Process Step: Implementation MSC:

5. A nurse who is assisting in disaster triage following a severe hurricane finds a client who has experienced a massive head injury from flying debris. In which of these categories would the nurse place this client? a. immediate b. delayed c. minimal d. expectant

ANS: D Disaster triage systems are used in the event of a community disaster, either natural or man-made. A different approach to categorizing clients is required because of the potential for large numbers of casualties. Immediate care is given to clients with simple injuries requiring immediate care. Delayed care is provided for clients with multiple injuries requiring extensive care. Minimal care is given to clients with minor injuries. Expectant care is provided for clients with severe injuries likely to lead to death.

8. When a patient has a suspected vestibular disorder, the physician orders an electronystagmography test. Which instruction should the nurse include when educating the patient about this test? a. Use tea or coffee on the morning of test. b. Electrodes will be placed on the scalp. c. Air will be blown into the external ear. d. The patient should have nothing to eat or drink (NPO) 3 hours before the test.

ANS: D Electronystagmography is used to detect vestibular lesions and requires a 3-hour period of NPO before the test. Coffee and tea should also be avoided before the test. DIF: Cognitive Level: Comprehension REF: p. 1255 OBJ: 3 TOP: Testing for Ear Disorders KEY: Nursing Process Step: Planning MSC:

24. A client has experienced severe frostbite to the feet and toes. Which of these actions should the nurse take? a. Administer prophylactic antibiotics. b. Offer the client several cups of hot coffee to assist in rewarming. c. Massage the injured areas as they rewarm, to promote circulation. d. Provide rapid rewarming using warm-water baths.

ANS: D Frostbite is also the result of exposure to cold. The nurse should not massage or rub frostbite injuries, as this can increase the severity of damage to tissue. For frostbite, rewarming of the exposed body part is indicated. If the frostbite is severe, rapid rewarming is essential. This involves placing the frozen area in warm-water baths not exceeding 40.6°C (105°F).

14. Frequent monitoring of the postoperative patient's vital signs assesses which body system? a. Gastrointestinal b. Endocrine c. Neurological d. Cardiovascular

ANS: D Hypotension and cardiac dysrhythmias are the most common cardiovascular complications of the surgical patient, and early recognition and management of these complications before they become serious enough to diminish cardiac output depend on frequent assessment of the patient's vital signs. REF: Pages 1271, 1287, Table 42-4 TOP: Postoperative patient

15. Decreased activity in an obese surgical patient predisposes the patient to which complication? a. Cardiac arrest b. Pneumonia c. Incisional hernias d. Hypoventilation

ANS: D Immediate postoperative hypoventilation can result from drugs (anesthetics, narcotics, tranquilizers, sedatives) incisional pain, obesity, chronic lung disease, or pressure on the diaphragm. REF: Page 1284 TOP: Postoperative complications Step: Planning

20. A Tzanck smear is used to diagnose which skin condition? a. fungal infections b. allergies to environmental agents c. skin cancer d. herpes zoster or varicella

ANS: D In a Tzanck smear, fluid from the base of a vesicle is applied to a glass slide, stained, and examined under a microscope. It is used to diagnose herpes zoster, herpes simplex, varicella, or pemphigus.

19. A Wood's light examination is being performed on a client. The nurse knows this test is performed to diagnose which skin condition? a. herpes b. allergies to environmental agents c. cancerous tissues d. fungal infections of hair and skin

ANS: D In a Wood's light examination the skin and hair are examined under ultraviolet light (black light) in a darkened room. This test is used to diagnose fungal infections (tinea) of hair and skin.

27. An 8-year-old girl is stung by bees several times on the school playground, and the school nurse brings the child to the emergency department. The child is experiencing respiratory difficulties and may require endotracheal intubation. Who can obtain informed consent for this invasive procedure? a. the client b. the school nurse c. only direct consent from a parent or legal guardian d. Informed consent is not needed in emergency situations.

ANS: D In life-threatening emergencies of minor children, informed consent is not needed.

33. In preparation for the return of the surgical patient, the patient's bed and equipment should be in what position? a. Lowest position with side rails elevated with oxygen and suction equipment available b. Highest position with side rails elevated with IV pole and pump at bedside c. Lowest position with side rails down on the receiving side d. Highest position with the side rails down on receiving side and up on opposite side

ANS: D In preparation for the return of the surgical patient, the patient's bed should be in the highest position to be level with the surgical gurney and should have the side rail down on the receiving side, with the opposite side rail up to prevent the patient from falling out of bed during transfer. DIF: Cognitive Level: ImplementationREF: Page 1279 TOP: Nursing diagnosis Step: Planning

14. A patient, age 27, sustained thermal burns to 18% of her body surface area. After the first 72 hours, the nurse will have to observe for the most common cause of burn-related deaths, which is a. shock. b. respiratory arrest. c. hemorrhage. d. infection.

ANS: D Infection is the most common complication and cause of death after the first 72 hours. REF: Page 1334 TOP: Burns

10. When initiating cardiopulmonary resuscitation on a hospitalized client, which of these actions should the nurse take to promote return of respiration? a. Ask a client's family member to perform mouth-to-mouth resuscitation. b. Personally give breaths using mouth-to-nose technique. c. Request that an endotracheal tube (ET) be placed by qualified personnel. d. Use a bag-valve-mask resuscitator.

ANS: D Initiation of cardiopulmonary resuscitation (CPR) includes promoting respiratory function through the use of a bag-valve-mask resuscitator.

20. The patient, age 71, has severe vertigo. A nursing action to help him reduce the symptoms is: a. Avoid sudden movements until the vertigo lessens. b. Avoid noises until the vertigo lessens. c. Encourage fluid intake. d. Lie immobile and hold the head in one position until the vertigo lessens.

ANS: D Lie immobile and hold the head in one position until the vertigo lessens. REF: Page 1876, Patient Teaching box TOP: Vertigo Step: Planning

20. How should the nurse advise a patient who has severe vertigo from labyrinthitis? a. Lean against a wall and not head forward until vertigo lessens. b. Bend at the waist and take several deep breaths. c. Drink an iced drink slowly. d. Lie immobile and hold the head in one position until the vertigo lessens.

ANS: D Lying immobile and holding the head in one position will lessen vertigo. PTS: 1 DIF: Cognitive Level: Application REF: Page 1880, Patient Teaching OBJ: 16 TOP: Vertigo KEY: Nursing Process Step: Implementation

21. What do miotic eyedrops do for a patient with glaucoma? a. Dilate the pupil and sharpen vision b. Lubricate and moisten the dry eye c. Irrigate the surface of the eye d. Constrict the pupil and open the canal of Schlemm

ANS: D Miotic eyedrops allow the pupil to constrict and open the canal of Schlemm to drain the excess fluid. PTS: 1 DIF: Cognitive Level: Application REF: Page 1868 OBJ: 4 TOP: Aging KEY: Nursing Process Step: Assessment

2. A patient asks when he should expect to be up and walking after his below-the-knee amputation. When should the nurse assure him that most people with amputations can fully bear weight? a. 3 weeks b. 1 month c. 6 weeks d. 3 months

ANS: D Most people with amputations can fully bear weight 3 months after surgery. DIF: Cognitive Level: Comprehension REF: p. 993 OBJ: 5 TOP: Amputation Recovery KEY: Nursing Process Step: Implementation MSC:

2. What should a nurse teach an older patient with a newly casted Colles fracture? a. Apply cool compresses to the cast. b. Let the hand and arm dangle to increase the drainage. c. Keep the hand immobile to reduce swelling. d. Move the shoulders to reduce contractures.

ANS: D Movement of the shoulders will help decrease the threat of contracture from immobility. DIF: Cognitive Level: Comprehension REF: p. 987 OBJ: 7 TOP: Colles Fracture KEY: Nursing Process Step: Implementation

4. A small child has had vomiting and diarrhea for more than 12 hours, unrelieved by the medication prescribed by a pediatrician earlier in the day. The child is listless but awake and refuses anything offered by mouth; vital signs are normal. In which of these triage categories would the child be placed? a. emergent b. priority c. nonurgent d. urgent

ANS: D Not all clients who use the emergency department (ED) require immediate care. Clients and families define an emergency according to their own perceptions. Emergent clients require immediate care to sustain life or limb. Urgent clients require care within 1 to 2 hours to prevent a decline in their condition.

12. The nurse should include in client teaching that the MOST common type of conductive hearing loss is: a. acoustic neuroma b. Ménière's disease c. otitis media d. otosclerosis

ANS: D Otosclerosis is a conductive hearing loss that occurs when the ossicles become softened, spongy, highly vascular, and partially or totally fixed. Treatment options include hearing aids or surgery to remove the stapes and implant a plastic prosthesis. Some clients may opt to skip treatment.

19. Which diagnosis is most appropriate for the nurse to select for a patient with Behçet syndrome? a. Activity intolerance, related to unsteady gait b. Risk for injury, related to falls c. Imbalance in nutrition: Less than body requirements, related to anorexia d. Sexual dysfunction, related to pain in genital area

ANS: D Pain from the genital ulcers interferes with sexual expression. DIF: Cognitive Level: Application REF: p. 968 OBJ: 8 TOP: Behçet Syndrome KEY: Nursing Process Step: Planning MSC: NCLEX: Physiological Integrity:

21. A nurse is caring for an adult patient with extensive burns on the front of the trunk, including the genitalia, and the fronts of both legs. How should the nurse document the burn size using the rule of nines? a. 13% b. 17% c. 25% d. 37%

ANS: D Per the rule of nines, the front trunk equals 18%, the fronts of the legs equal 18%, and the genitalia equal 1%. DIF: Cognitive Level: Analysis REF: p. 1207 OBJ: 5 TOP: Burn Estimate KEY: Nursing Process Step: Assessment

11. What does the cataract treatment of phacoemulsification involve? a. "Drying" the cataract with hypertonic saline b. Removing the lens through the anterior capsule c. The insertion of a new lens d. Breaking the cataract with ultrasound

ANS: D Phacoemulsification uses ultrasound to break up the cataract. PTS: 1 DIF: Cognitive Level: Analysis REF: Page 1860 OBJ: 11 TOP: Infectious/inflammatory disorders KEY: Nursing Process Step: Planning

26. How would the nurse explain the purpose of photocoagulation to a diabetic patient with diabetic retinopathy? a. The procedure will destroy the retina, which is not getting enough blood supply. b. The procedure will reduce edema in the macula of the eye. c. The procedure will vaporize fatty deposits that appear in the retina. d. The procedure will destroy new blood vessels, seal leaking vessels, and help prevent retinal edema.

ANS: D Photocoagulation is useful in diabetic retinopathy to cauterize hemorrhaging vessels and to destroy new vessels. PTS: 1 DIF: Cognitive Level: Analysis REF: Page 1863 OBJ: 9 TOP: Diabetic retinopathy KEY: Nursing Process Step: Implementation

26. The patient, age 62, has had insulin-dependent diabetes mellitus for 20 years and has symptoms of proliferate diabetic retinopathy. He is scheduled for his first panretinal photocoagulation treatment. The nurse explains to him that the purpose of this procedure is to a. destroy the retina, which is not getting enough blood supply. b. reduce edema in the macula of the eye. c. vaporize fatty deposits that appear in the retina. d. destroy new blood vessels, seal leaking vessels, and help prevent retinal edema.

ANS: D Photocoagulation is useful in diabetic retinopathy to cauterize hemorrhaging vessels. REF: Page 1856 TOP: Diabetic retinopathy

26. A nurse is educating a patient with gout about the medication probenecid (Benemid). What active effect should the nurse relay when explaining why this medication is prescribed? a. Reduces inflammation in the affected joint b. Relieves pain c. Diminishes swelling d. Increases excretion of uric acid

ANS: D Probenecid (Benemid) increases the excretion of uric acid to reduce the symptoms of gout. DIF: Cognitive Level: Comprehension REF: p. 963 OBJ: 5 TOP: Probenecid KEY: Nursing Process Step: Implementation MSC: NCLEX: Physiological Integrity:

10. Which is true regarding connective tissue function? a. Helps provide a source of storage for calcium b. Stores hormones in the pores of bone tissue c. Controls the distribution of minerals d. Provides protection to body parts

ANS: D Providing protection is a function of connective tissue. DIF: Cognitive Level: Knowledge REF: p. 939 OBJ: 2 TOP: Connective Tissue Function KEY: Nursing Process Step: Assessment MSC: NCLEX: Physiological Integrity:

16. What is the best physiologic reason for a patient with osteoporosis to maintain a regular exercise regimen? a. Involves the patient in her or his own care b. Increases cardiac output c. Promotes better mental health d. Promotes bone formation and improves strength

ANS: D Regular exercise promotes bone formation, which is important for patients with osteoporosis for physiologic reasons. DIF: Cognitive Level: Comprehension REF: p. 961 OBJ: 8 TOP: Osteoporosis KEY: Nursing Process Step: Implementation MSC: NCLEX: Physiological Integrity:

23. What instruction should a nurse include in a teaching plan for a patient with carpal tunnel syndrome? a. Anticoagulants and glucocorticoids b. Methotrexate c. Lubricating ointments d. Splinting to prevent flexion and hyperextension

ANS: D Resting and supporting the joint are first-line treatments. DIF: Cognitive Level: Comprehension REF: p. 968 OBJ: 8 TOP: Carpal Tunnel Syndrome KEY: Nursing Process Step: Planning MSC: NCLEX: Physiological Integrity:

8. A patient has just had a plaster of Paris upper extremity cast placed because of a fractured radius. Which statement indicates that the patient understands the discharge teaching related to cast care? a. "When I get home, I will remove some of the padding if it feels tight so my fingers don't swell." b. "When I get home, I will wrap the cast in plastic so it will conserve the heat." c. "When I get home, I will use a spoon handle to scratch inside if my arm itches." d. "When I get home, I am going to rest in bed with my arm elevated above my heart."

ANS: D Resting with the limb elevated above the heart helps prevent swelling. DIF: Cognitive Level: Comprehension REF: p. 984 OBJ: 5 TOP: Cast Care KEY: Nursing Process Step: Evaluation

7. What is the primary treatment for a client who is in hypovolemic shock? a. administration of epinephrine and diphenhydramine hydrochloride b. antibiotic therapy c. emergency coronary bypass surgery d. fluid replacement

ANS: D Shock, a condition of profound hemodynamic and metabolic disturbance characterized by inadequate tissue perfusion and inadequate circulation to the vital organs, must be recognized and immediately treated. Hypovolemic shock is easily recognizable when bleeding is external but more difficult when the bleeding is internal. The primary treatment for a client who is in hypovolemic shock is fluid replacement.

4. A client has numerous petechiae over the face, neck, and upper chest and areas of ecchymosis on the abdomen and upper legs. Which of these skin parameters has the nurse assessed? a. integrity b. texture c. turgor d. vascularity

ANS: D Skin normally should be clear with no areas of discoloration. The nurse should note evidence of ecchymosis, petechiae, or telangiectasia.

21. Imbalanced nutrition: Less than body requirements is the nursing diagnosis applicable to a patient with progressive systemic sclerosis. What is the most important point for the nurse to teach this patient? a. Eat three large meals spaced throughout the day. b. Schedule rest periods to prevent overtiring. c. Severe stress can trigger vasospasm. d. Eat smaller, more frequent meals.

ANS: D Smaller, more frequent meals may be better tolerated by a patient who has esophageal involvement. DIF: Cognitive Level: Application REF: p. 966 OBJ: 8 TOP: PSS KEY: Nursing Process Step: Implementation MSC: NCLEX: Physiological Integrity:

22. Which assessment by an emergency department nurse most indicates that a burn patient might be at risk for respiratory impairment? a. Burns on the face and neck b. Respiration of 18 breaths/min c. Flaring nares d. Sooty sputum

ANS: D Sooty sputum is the most indicative. Facial burns and flaring nares are not conclusive in themselves. Respiration rate of 18 breaths/min is normal. DIF: Cognitive Level: Application REF: p. 1200 OBJ: 5 TOP: Burns: Respiratory Impairment KEY: Nursing Process Step: Assessment

23. Sudden chest pain combined with dyspnea, cyanosis, and tachycardia is an indication of a. hypovolemic shock. b. dehiscence. c. atelectasis. d. pulmonary embolus.

ANS: D Sudden chest pain combined with dyspnea, tachycardia, cyanosis, diaphoresis, and hypotension is a sign of pulmonary embolism. REF: Page 1285 TOP: Assessment and postoperative complications

7. After a knee replacement, an 87-year-old patient rejects the use of the continuous passive motion (CPM) machine, saying, "I did not march when I was a child, and I am not marching now." What benefits of CPM should the nurse point out to encourage patient use? a. Decrease in pain b. Increase in circulation in the new joint c. Increase in leg strength d. Increase in flexibility for the new joint

ANS: D The CPM machine's major benefit is to increase flexibility, although it does cause discomfort. No strength-building potential is present with passive motion. DIF: Cognitive Level: Application REF: p. 955 OBJ: 8 TOP: Total Hip Replacement KEY: Nursing Process Step: Implementation MSC: NCLEX: Physiological Integrity:

24. During examination of his external ear, the patient tells the nurse that his ear is very painful when touched. The nurse recognizes that this finding is commonly associated with a. a ruptured tympanic membrane. b. a blocked eustachian tube. c. infection of the mastoid bone. d. inflammation of the ear canal.

ANS: D The acute inflammatory or infectious process produces pain with movement of the auricle or chewing, and often the entire side of the headaches. REF: Page 1871 TOP: Infectious/inflammatory disorders

16. A patient who amputated his thumb in a lawnmower accident hands the emergency department nurse his thumb in a glass jar. What is the best action for the nurse to implement? a. Place the thumb in a baggie with iced lactated Ringer solution. b. Wrap the thumb in plastic wrap and place it on ice. c. Leave the thumb in the jar and place it in the refrigerator. d. Wrap the thumb in a cloth saturated with normal saline and place it in a baggie.

ANS: D The amputated part should be wrapped in a towel soaked with normal saline solution, placed in a baggie, and put in a cool bath. DIF: Cognitive Level: Application REF: p. 1001 OBJ: 4 TOP: Care of the Amputated Thumb KEY: Nursing Process Step: Implementation MSC:

7. What do thermographic findings of cool spots in a certain areas of the body indicate? a. Increased blood flow b. Decreased infection c. Increased infection d. Decreased blood flow

ANS: D The area is cool because blood flow is decreased. DIF: Cognitive Level: Comprehension REF: p. 992 OBJ: 3 TOP: Diagnostic Tests KEY: Nursing Process Step: Assessment MSC:

9. The first and MOST important function of the skin is: a. to serve as a barrier to keep essential substances inside the body b. to cushion internal organs c. to maintain normal body temperature d. to protect and serve as a barrier against microorganisms and other substances

ANS: D The first and most important function of the skin is protection. As long as the skin is intact and healthy, it is a barrier against microorganisms and numerous substances that could be harmful to the individual. The skin is also a barrier to keep essential substances such as water and electrolytes inside the body. Another function of skin is to cushion internal organs.

3. What should a nurse ask about when taking the functional assessment of a patient with a skin disorder? a. A sore that is slow to heal b. Unusual hair growth c. Previous skin disorders d. Exposure to chemicals or irritants

ANS: D The functional assessment is a search for clues in the occupation and lifestyle of the patient. The other options all reference medical history and system review. DIF: Cognitive Level: Application REF: p. 1185 OBJ: 5 TOP: Functional Assessment KEY: Nursing Process Step: Assessment

18. A child comes to the school nurse with his index finger partially amputated and hanging by a shred of skin and muscle. What is the best action by the nurse? a. Flush the hand with warm water and wrap it in a towel. b. Carefully cut the skin holding the finger and wrap the finger and hand in a clean towel. c. Pinch the finger to stop the bleeding and take the child to the hospital. d. Wrap the hand and finger securely and place it on an ice water-filled plastic bag.

ANS: D The nurse should leave the shred of skin and muscle intact, wrap the hand and finger in a towel soaked in normal saline, and place it on a cool surface. DIF: Cognitive Level: Application REF: p. 1001 OBJ: 4 TOP: Partial Amputation KEY: Nursing Process Step: Implementation MSC:

12. When assessing a patient who is severely bleeding and at risk for hypovolemic shock, the nurse anticipates: a. slow, labored breathing. b. hot, flushed skin. c. edematous extremities. d. weak, thready pulse.

ANS: D The pulse becomes weak and thready with hypovolemic shock. REF: Page 747

16. A patient who has undergone treatment for herpes simplex virus type 2 (HSV type 2) expresses relief that she is cured. What should the nurse include in her teaching? a. Daily douches of Burow solution are needed. b. HSV is permanently cured by acyclovir (Zovirax). c. Sexual partners are now safe from infection from her. d. HSV lies dormant and can be triggered without any sexual contact.

ANS: D The virus goes dormant but can recur. Herpes is always present. DIF: Cognitive Level: Application REF: p. 1201 OBJ: 7 TOP: Herpes Simplex KEY: Nursing Process Step: Planning

5. A patient has an erythematous patch of vesicles on her scalp, and she complains of pain and pruritus. A diagnosis of tinea capitis is made. The causative organism is a. bacterium. b. virus. c. worm. d. fungus.

ANS: D Tinea capitis is commonly known as ringworm of the scalp. Microsporum audouinii is the major fungal pathogen. REF: Page 1314 TOP: Infection

21. A female patient is seen by the school nurse because of flat lesions that are clear in the center with erythematous borders. In assessing a patient for tinea corporis, the nurse would check a. soles of the feet. b. back of the scalp. c. groin area. d. anterior abdomen.

ANS: D Tinea corporis is known as ringworm of the body. It occurs on parts of the body with little or no hair. REF: Page 1314, Figure 43-8 TOP: Tinea corporis

25. The patient is admitted to the emergency department, having suffered frostbite to the hands, which are grayish-white in color. When attempting to warm the hands, the nurse should: a. have the patient rub the hands together briskly. b. wipe the hands vigorously with a warm towel. c. run tepid water over the hands to warm slowly. d. wrap the hands in hot, moist towels.

ANS: D Warming the hands in moist towels will warm the hands slowly. Friction of frozen body parts should be avoided. REF: Page 757 TOP: Frostbite

23. What action should a nurse implement when dealing with the weights that are applying traction to a patient? a. Remove them to pull the patient up in bed. b. Hold them while the patient is changing positions in bed. c. Hold them for a few minutes if the patient complains of pain. d. Allow them to hang freely.

ANS: D Weights must always hang freely to prevent complications. DIF: Cognitive Level: Application REF: p. 979 OBJ: 5 | 6 TOP: Traction KEY: Nursing Process Step: Implementation

19. The patient's lower chest has been punctured with a knife that is still in place. The nurse should initially: a. remove the knife. b. apply an airtight dressing over the wound. c. place the patient in a modified Trendelenburg position. d. immobilize the knife with dressings and tape.

ANS: D When the patient's lower chest has been punctured with the weapon still in place, the nurse should immobilize the weapon with dressings and tape. REF: Page 751 TOP: Wounds

2. A nurse is assisting with a caloric test and notes that the specific patient response that indicates a hearing disorder is a problem in the labyrinth. Which response did the nurse witness? a. Blinking b. Grimacing c. Headache d. Nystagmus

ANS: D When warm or cold water is introduced into the ear, the appearance of nystagmus is a positive indication that the hearing problem has its cause in the labyrinth. DIF: Cognitive Level: Comprehension REF: p. 1256 OBJ: 2 TOP: Caloric Test KEY: Nursing Process Step: Assessment MSC:

10. A patient undergoing a Weber test says that the sound is louder in her left ear. What should this result indicate? a. Normal hearing b. Nerve damage from listening to loud music c. Blocked ear canal in the right ear d. Conductive hearing loss in the left ear

ANS: D With the Weber test, a conductive hearing loss is determined by the sound being heard loudest in the affected ear. DIF: Cognitive Level: Comprehension REF: p. 1256 OBJ: 2 TOP: Weber Test KEY: Nursing Process Step: Assessment MSC:

41. A patient is transferred from the operating room to the recovery room after undergoing an amputation of his left foot. Place the interventions in the correct order for immediate assessment once the patient enters the PACU. Place a comma between each answer choice (a, b, c, d, etc.). a. System review b. Breathing c. Circulation d. Airway e. Level of consciousness

ANS: D, B, E, C, A REF: Page 1281, Table 42-6 TOP: Nursing assessment

37. Schiötz tonometry is a diagnostic test for .

ANS: glaucoma REF: Page 1860, Figure 53-11 TOP: Diagnostic tests

An elderly client with a diagnosis of osteoporosis and early stage Alzheimer disease requires the use of an assistive device that will maximize stability during ambulation. What device will best meet this client's needs? A) Walker B) Axillary crutches C) Forearm crutches D) Cane

Ans: A Feedback: A walker provides greater stability than crutches or a cane.

A nurse is caring for a client who underwent surgery for the amputation of her right foot. The client has been fitted with a temporary prosthetic limb. Which of the following interventions or outcomes should the nurse prioritize during this period? A) The wound heals and no complications develop. B) The measurement for the prosthetic limb is accurate. C) The client is provided with an ambulatory device. D) The client is able to ambulate with assistance.

Ans: A Feedback: After amputation, the nurse needs to ensure that the client's wound heals and no complications such as joint contractures or infection develop. This is a priority in the client's recovery because complications delay rehabilitation. This action is a priority over measurements, provision of assistive devices, and even the client's ability to ambulate.

A nurse is caring for a client whose fractured ankle is in a cast. The client needs crutches to ambulate. Which of the following would help strengthen this client's upper body in preparation for ambulation? A) Modified hand push-ups B) Isotonic exercises C) Parallel bars D) Tilt table

Ans: A Feedback: Modified hand push-ups strengthen the upper arms, thus helping the client to ambulate using crutches. An exercise regimen to strengthen the upper arms typically includes flexion and extension of the arms and wrists, raising and lowering weights with the hands, squeezing a ball or spring grip, and performing modified hand push-ups in bed. Isotonic exercise builds flexibility and stamina more than strength. Clients use parallel bars as hand rails to gain practice in ambulating. A tilt table is a device that helps clients adjust to being upright and bearing weight on their feet.

A client who tore his quadriceps muscle during a soccer match is being treated at a health care facility. The physician has prescribed exercise for the quadriceps muscles in order to rehabilitate the client. How should the client perform quadriceps setting exercises? A) By alternatively tensing and relaxing the muscles B) By performing modified hand push-ups in bed C) By sitting on the edge of the bed D) By contracting and relaxing the muscles

Ans: A Feedback: The client performs quadriceps setting exercises by alternatively tensing and relaxing the quadriceps muscles. Modified hand push-ups in bed would enable the client to strengthen the upper arms. The client dangles or sits at the edge of the bed in order to normalize blood pressure. Gluteal setting is contraction and relaxation of the gluteal muscles in order to strengthen and tone them.

Which of the following ambulatory aids could a nurse suggest to assist a client who has weakness in one side of his body? A) Cane B) Walker C) Axillary crutch D) Forearm crutch

Ans: A Feedback: The nurse could suggest the use of a cane to a client who has weakness in one side of his body in order to aid ambulation. Canes are hand-held ambulatory devices made of wood or aluminum. A walker is used by clients who require considerable assistance with balance. Clients who need brief, temporary assistance with ambulation are likely to use axillary crutches. Forearm crutches are generally used by experienced clients who need permanent assistance with walking.

A client is prescribed topical benzocaine. The nurse would monitor the client for which of the following as a possible adverse reaction? A) Stinging, tenderness, and sloughing B) Erythema, flaking, and dryness C) Transient burning sensation D) Oiliness or dryness of hair

Ans: A Feedback: The nurse should monitor for stinging, tenderness, and sloughing in the client as adverse reactions to benzocaine. Erythema, flaking, and dryness are adverse reactions to masoprocol. A transient burning sensation is an adverse reaction to collagenase. Oiliness or dryness of the hair is an adverse reaction to selenium sulfide.

A nurse is a caring for a diabetic client whose right leg had to be amputated below the knee due to the repeated development of osteomyelitis and gangrene. The client uses crutches to ambulate and is waiting to be fitted with a prosthetic leg. What gait should the nurse observe in this client? A) Swing-through B) Four-point C) One-point D) Two-point

Ans: A Feedback: The nurse should observe a swing-through gait in a client whose leg had to be amputated and who is waiting to be fitted with a prosthetic leg. Two-point and four-point gaits require the use of both legs. A one-point gait does not exist.

A nurse at a health care facility is caring for clients using crutches to ambulate. In which of the following clients would the nurse observe a four-point walking gait? A) Clients with disabilities such as arthritis or cerebral palsy B) Clients who have normal coordination and balance C) Clients with one amputated, injured, or disabled extremity D) Clients with amputated limbs who are learning to use prosthetic limbs

Ans: A Feedback: The nurse would observe a four-point gait in clients with disabilities such as arthritis or cerebral palsy and who use crutches to ambulate. Clients who have more coordination and balance are more likely to have a two-point gait. A three-point nonñweight-bearing gait can be observed in clients with one amputated, injured, or disabled extremity. A client with an amputated limb learning to use prosthesis would have a three-point partialñweight-bearing gait.

A client at a health care facility is to undergo amputation of the left leg due to intractable arterial and vascular ulcers. The client will be fitted with an immediate postoperative prosthesis (IPOP) after the surgery. What are the functions of the IPOP? Select all that apply. A) Reduces stump swelling B) Promotes intact body image C) Promotes early ambulation D) Helps to compress the vein walls E) Helps to absorb wound drainage from the surgical site

Ans: A, B, C Feedback: An IPOP facilitates early ambulation, promotes intact body image, and controls stump swelling in clients who have undergone surgery for amputation. An IPOP does not serve to absorb drainage or compress vein walls.

A client is prescribed topical betamethasone. Which of the following would the nurse include when explaining the possible adverse reactions that may occur? Select all that apply. A) Burning B) Dryness C) Pruritus D) Nausea E) Fever

Ans: A, B, C Feedback: Localized reactions caused by the topical administration of betamethasone that the nurse should discuss with the client include burning, pruritus, irritation, redness, dryness, allergic contact dermatitis, and secondary infection.

Which of the following would be most important to include in the preadministration assessment of a client who is receiving topical therapy for a skin disorder? Select all that apply. A) Size of the area affected B) Appearance of the lesions C) Report of pain or burning D) Client's weight E) Blood pressure

Ans: A, B, C Feedback: The preadministration assessment involves a visual inspection and palpation of the involved area(s). The areas of involvement, including the size, color, and appearance, are carefully measured and documented. The appearance of the skin lesions, such as rough and itchy patches, cracks between the toes, and sore and reddened areas, is noted so treatment can begin with an accurate database. A specific description is important so that changes indicating worsening or improvement of the lesions can be readily identified. Measuring the client's weight or blood pressure would not be important.

A client is prescribed topical clindamycin therapy. The nurse instructs the client and family about possible systemic effects. The nurse determines that the teaching was successful when they state that they should contact the primary health care provider immediately if which of the following occur? Select all that apply. A) Stomach cramps B) Severe diarrhea C) Bloody stools D) Burning E) Pruritus

Ans: A, B, C Feedback: Topical clindamycin can be absorbed in sufficient amounts to cause systemic effects. If severe diarrhea, stomach cramps, or bloody stools occur, the client should contact the primary health care provider immediately.

A nurse may use a topical antiseptic or germicide for which of the following reasons? Select all that apply. A) To reduce the number of bacteria on skin surfaces B) As a surgical scrub C) As a preoperative skin cleanser D) For washing the hands before and after caring for clients E) On minor cuts and abrasions to prevent infection

Ans: A, B, C, D, E Feedback: A nurse may use a topical antiseptic or germicide for the following reasons: to reduce the number of bacteria on skin surfaces, as a surgical scrub, as a preoperative skin cleanser, for washing the hands before and after caring for clients, and on minor cuts and abrasions to prevent infection.

After teaching a group of nursing students about the use of salicylic acid (Duofilm), the instructor determines that the teaching was successful when the students identify which of the following as a contraindication? Select all that apply. A) Genital warts B) Facial warts C) Mucous membrane warts D) Moles E) Birthmarks

Ans: A, B, C, D, E Feedback: The use of salicylic acid (Duofilm) is contraindicated in the following situations: on moles, birthmarks, warts with hair growing from them, genital or facial warts, warts on mucous membranes, or infected skin.

A client is prescribed a topical corticosteroid. The nurse understands that this drug would be contraindicated in which of the following circumstances? Select all that apply. A) Monotherapy for bacterial skin infections B) Ophthalmic use C) Otic use D) Monotherapy for widespread plaque psoriasis E) Low-potency corticosteroids on the face, groin, or axilla

Ans: A, B, D Feedback: The use of topical corticosteroids is contraindicated in the following circumstances: known hypersensitivity to the drug or any component of the drug; as monotherapy for bacterial skin infections; high-potency corticosteroids for use on the face, groin, or axilla; for ophthalmic use; and as monotherapy in widespread plaque psoriasis.

The nurse might suspect a hypersensitivity reaction has occurred in a client using azelaic acid (Azelex) if the client experiences a combination of which of the following? Select all that apply. A) Pruritus B) Urticaria C) Vomiting D) Erythema E) Mental status changes

Ans: A, B, D Feedback: Topical anti-infectives, like azelaic acid (Azelex), result in a rash, pruritus, urticaria, dermatitis, irritation, and erythema, which may indicate a hypersensitivity reaction to the drug.

The nurse should discuss which of the following adverse reactions with a client prior to the topical administration of anthralin (Miconal)? Select all that apply. A) Hair discoloration B) Discoloration of fingernails C) Discoloration of skin D) Pruritus E) Burning

Ans: A, B, D, E Feedback: Localized reactions caused by the topical administration of anthralin (Miconal) that the nurse should discuss with the client prior to administration include burning, pruritus, irritation, and temporary discoloration of the fingernails and hair.

A client is prescribed a topical corticosteroid. The nurse understands that this drug is used in the treatment of which of the following conditions? Select all that apply. A) Psoriasis B) Eczema C) Asthma exacerbation D) Rheumatoid arthritis E) Insect bites

Ans: A, B, E Feedback: Topical corticosteroids are used in the treatment of the following conditions: psoriasis, eczema, dermatitis, rashes, insect bites, and first- and second-degree burns.

A client is prescribed topical triamcinolone. The nurse is aware that the client may experience systemic adverse reactions. Which of the following might the nurse assess? Select all that apply. A) Hyperglycemia B) Myasthenia gravis C) Cushing's syndrome D) Nausea E) Fever

Ans: A, C Feedback: Systemic reactions caused by the topical administration of triamcinolone (Aristocort) that the nurse should be alert for include hypothalamic-pituitary-adrenal axis suppression, Cushing's syndrome, hyperglycemia, and glycosuria.

Which of the following might nurses use to clean their hands before and after caring for a client? Select all that apply. A) Chlorhexidine (Hibiclens) B) Fluocinonide (Lidex) C) Hexachlorophene (pHisoHex) D) Imiquimod (Aldara) E) Ketoconazole (Nizoral)

Ans: A, C Feedback: Topical antiseptics, like chlorhexidine (Hibiclens) and hexachlorophene (pHisoHex), can be used by nurses to clean their hands before and after caring for a client. Fluocinonide is a topical corticosteroid. Imiquimod is a topical antiviral agent. Ketoconazole is a topical antifungal drug.

1. A nursing instructor is evaluating the students' understanding of topical anti-infectives. The instructor determines that the group understands the information when they pair which drug with the infection treated correctly? Select all that apply. A) Erythromycin (Erygel) - acne vulgaris B) Ketoconazole (Nizoral) - episodes of HSV C) Ciclopirox (Loprox) - tinea pedis D) Acyclovir (Zovirax) - episodes of HSV E) Bacitracin (Baci-Rx) - tinea corporis

Ans: A, C, D Feedback: The following topical anti-infective drugs are matched correctly with the infection they treat: erythromycin (Erygel) - acne vulgaris; ciclopirox (Loprox) - tinea pedis; and acyclovir (Zovirax) - episodes of HSV.

A nurse is instructing a client about how to apply the topical medication prescribed. Which of the following would the nurse identify as an appropriate way to remove the drug from the container? Select all that apply. A) Finger cot B) Clean finger C) Tongue blade D) Gauze pad E) Cotton swab

Ans: A, C, D, E Feedback: The nurse should instruct the client to use a finger cot, tongue blade, gauze pad, or cotton swab to remove the drug from the container and then apply it to the skin.

The nurse is applying topical collagenase to a client. The nurse would assess the client for which of the following as a possible adverse reaction? Select all that apply. A) Mild pain B) Urticaria C) Numbness D) Pruritus E) Dermatitis

Ans: A, C, E Feedback: Possible adverse reactions associated with the topical administration of collagenase (Santyl) include mild, transient pain; possible numbness; and dermatitis.

A nurse administering collagenase (Santyl) topically to a client must be certain not to use which of the following products that can inactivate the enzymes in collagenase (Santyl)? Select all that apply. A) Detergents B) Water C) Iodine D) Silver E) Mercury

Ans: A, D, E Feedback: A nurse administering collagenase (Santyl) topically to a client must be certain not to use detergents or products containing heavy metals, like mercury and silver, which can inactivate the enzymes in collagenase (Santyl).

A nurse is providing care to two clients, one with a Staphylococcus aureus skin infection and another with a Streptococcus pyogenes infection. Which of the following would the nurse expect the primary health care provider to prescribe? Select all that apply. A) Mupirocin (Bactroban) B) Acyclovir (Zovirax) C) Ketoconazole (Nizoral) D) Metronidazole (Metro-Gel) E) Retapamulin (Altabax)

Ans: A, E Feedback: Mupirocin (Bactroban) and retapamulin (Altabax) are topical anti-infectives that can be used to treat Staphylococcus aureus and Streptococcus pyogenes infections of the skin.

A client is prescribed a topical corticosteroid. Which of the following would be appropriate for the nurse to do? A) Clean the area with an antiseptic before applying the drug. B) Apply the topical corticosteroid sparingly. C) Place a sterile cloth over the area of application. D) Rub the application into the skin vigorously.

Ans: B Feedback: Before drug application, the area is washed with soap and warm water unless the primary health care provider directs otherwise. Topical corticosteroids are usually ordered to be applied sparingly. The primary health care provider also may order the area of application to be covered or left exposed to the air. If covered, a dressing, not sterile cloth, would be used. The medication should be rubbed in gently.

An older adult has been admitted to a long-term care facility after unsuccessful attempts to continue living independently. The nurse's admission assessment reveals the presence of long, ingrown toenails that appear to contribute to the resident's unsteady gait and decreased mobility. How should the nurse follow up this assessment finding? A) Have the client tested for type 2 diabetes. B) Arrange for the client to be seen by a podiatrist. C) Cut the client's toenails using a sharp blade rather than regular nail clippers. D) Soak the client's feet in Epson salts and trim them short.

Ans: B Feedback: Elderly clients' problematic toenails may warrant referral to podiatry. It may risk complications to cut them very short or with a sharp blade. Clients with diabetes are particularly at risk for foot complications but the presence of long toenails does not suggest the presence of diabetes.

After teaching a group of nursing students about topical drugs for skin disorders, the instructor determines that the teaching was successful when the students identify which of the following as used to prevent institutional outbreaks of methicillin-resistant Staphylococcus aureus (MRSA)? A) Bacitracin B) Mupirocin C) Retapamulin D) Clindamycin

Ans: B Feedback: Mupirocin is applied to the nasal mucosa to reduce the risk of institutional outbreaks of MRSA. Bacitracin is used for relief of skin infections and to prevent infections with minor cuts and burns. Retapamulin is used to treat impetigo due to staphylococcus or streptococcus. Clindamycin is used to treat acne vulgaris.

A nurse is caring for a client with a strained gluteal muscle at a health care facility. The nurse should understand that which of the following is the primary function of the gluteal muscles? A) Aid in supporting body weight B) Aid in extending the legs C) Aid the client in standing D) Aid in strengthening the upper arms

Ans: B Feedback: The gluteal muscles aid the client in extending the legs. As a group, the gluteal muscles in the buttocks aid in extending, abducting, and rotating the legs; these functions are essential to walking. Exercising the quadriceps enables clients to stand and support their body weight. Exercises such as modified hand push-ups help in strengthening the upper arms.

The primary health care provider has prescribed dexamethasone sodium phosphate for a client being treated for immunologic skin disorder. Assessment of which of the following would lead the nurse to suspect that the client is experiencing an adverse reaction to the drug? A) Redness or mild scaling B) Allergic contact dermatitis C) Dermatitis and irritation D) Photosensitivity

Ans: B Feedback: The nurse should monitor for allergic contact dermatitis as an adverse reaction to dexamethasone sodium phosphate. Redness or mild scaling and photosensitivity are adverse reactions to hexachlorophene. Dermatitis and irritation are adverse reactions to povidone-iodine.

When assisting a client with ambulation using an assistive device such as parallel bars or a walking belt, what should the nurse observe the client for in order to ensure safety? A) Length of stride B) Pallor or dizziness C) Upper arm strength D) Tone of lower body muscles

Ans: B Feedback: When assisting a client with ambulation using an assistive device such as parallel bars or a walking belt, the nurse should observe the client for pallor, weakness, or dizziness. Strength, gait, and muscle tone would be closely observed by the nurse but these variables do not have such a direct bearing on safety.

A 22-year-old woman has begun using axillary crutches following open reduction and internal fixation of an ankle fracture that she suffered during a basketball game. What assessment finding by the nurse should suggest that the client's crutches are fitted correctly? A) The client's elbow joint is locked when she stands upright and grasps the handgrips. B) The client's wrist is slightly hyperextended when she stands upright and grasps her crutches. C) The level of the handgrips is at the client's waist when she stands upright with her crutches. D) The axillary bar is 3 to 4 in. below the client's axilla when she stands upright.

Ans: B Feedback: With axillary crutches, there should be 30 degrees of elbow flexion and slight hyperextension of the wrist when the client is standing in place. The level of the handgrips will be near the client's thigh and the axillary bar should be adjusted within two fingers' space of the axilla.

After conducting an in-service program for a group of nurses about topical drugs used to treat skin disorders, the presenter determines that the teaching was successful when the group identifies which of the following as an appropriate use for topical enzymes? Select all that apply. A) Widespread psoriasis B) Chronic dermal ulcers C) First-degree sunburn D) Severely burned areas E) Tinea pedis

Ans: B, D Feedback: A nurse may apply a topical enzyme to a client with chronic dermal ulcers or severely burned areas. Topical enzymes are not used for widespread psoriasis, first-degree sunburn, or tinea pedis.

A nurse is applying a topical agent that aids in the removal of dead soft tissue by hastening the reduction of proteins into simpler substances. Which of the following might the nurse be applying? Select all that apply. A) Bacitracin (Baci-Rx) B) Papain and urea (Accuzyme) C) Vidarabine (Ara-A) D) Collagenase (Santyl) E) Anthralin (Miconal)

Ans: B, D Feedback: Topical enzymes, like papain and urea (Accuzyme) and collagenase (Santyl), aid in the removal of dead soft tissue by hastening the reduction of proteins into simpler substances.

A client is to receive topical masoprocol. The nurse alerts the client to the possibility of which of the following as an adverse reaction? Select all that apply. A) Mild pain B) Scaling C) Numbness D) Flu-like syndrome E) Dry skin

Ans: B, D, E Feedback: Adverse reactions related to the topical administration of masoprocol (Actinex) include transient burning sensation, rash, dry skin, scaling, and flu-like syndrome.

A nurse is preparing to administer a keratolytic based on the understanding that this drug is used to treat which of the following skin disorders? Select all that apply. A) Psoriasis B) Warts C) Acne vulgaris D) Seborrheic keratoses E) Corns

Ans: B, D, E Feedback: Keratolytics are used to treat the following skin disorders: warts, calluses, corns, and seborrheic keratoses.

A client is diagnosed with psoriasis. Which of the following would the nurse expect the primary health care provider to prescribe as topical treatment? Select all that apply. A) Chlorhexidine (Hibiclens) B) Calcipotriene (Dovonex) C) Vidarabine (Ara-A) D) Imiquimod (Aldara) E) Anthralin (Miconal)

Ans: B, E Feedback: Anthralin (Miconal) and calcipotriene (Dovonex) are topical antipsoriatics used to treat psoriasis.

A client is diagnosed with dermatitis and the skin appears inflamed. Which topical drug would the nurse expect to administer to reduce itching, redness, and swelling caused by dermatitis? Select all that apply. A) Erythromycin (Erygel) B) Fluocinonide (Lidex) C) Penciclovir (Denavir) D) Imiquimod (Aldara) E) Hydrocortisone (Locoid)

Ans: B, E Feedback: Topical corticosteroids, like fluocinonide (Lidex) and hydrocortisone (Locoid), when applied to inflamed skin reduce itching, redness, and swelling caused by dermatitis. Erythromycin is a topical antibiotic; penciclovir and imiquimod are topical antiviral drugs.

The right leg of a client who was involved in a motor vehicle accident needs to be amputated due to complications. When would a permanent prosthetic limb be constructed for this client? A) As soon as wound drainage ceases B) Four to 6 weeks after the leg has been amputated C) When the stump size stabilizes D) Ten to 12 months after the amputation of the leg

Ans: C Feedback: A permanent prosthetic leg is constructed once the stump size has stabilized. Construction of a permanent prosthesis is delayed for several weeks or months until the wound heals and the stump size is relatively stable. It is the condition of the stump, rather than absolute time, that determines the appropriate point for introducing a permanent prosthesis.

When developing the plan of care for a client with a skin lesion requiring topical therapy, which nursing diagnosis would the nurse most likely identify? A) Acute Pain B) Risk for Infection C) Impaired Skin Integrity D) Disturbed Body Image

Ans: C Feedback: Although all the nursing diagnoses apply, the most likely nursing diagnosis would be Impaired Skin Integrity due to the presence of a skin lesion and need for treatment.

A nurse is caring for an elderly client at a health care facility. What problem might a nurse observe in an elderly client as a result of age-related postural changes? A) Use of a four-point gait B) Decreased gluteal setting C) Limited or unsteady mobility D) Appearance of corns or calluses

Ans: C Feedback: Limited or unsteady mobility may be a problem for some older adults as a result of age-related postural changes. Limited or unsteady mobility may lead to the development of a swaying or shuffling gait. A four-point gait is specific to the use of crutches and is not an age-related change. Gluteal setting is a specific exercise that is performed to prepare for ambulation; it is not a problem with mobility.

A nurse is assisting a client at a health care facility to dangle before the client ambulates. The nurse assists the client into the Fowler's position for a few minutes. Which of the following is a possible reason for this action? A) To maintain warmth and show respect for the client's modesty B) To allow the client to use the floor for support C) To maintain safety should the client become dizzy or faint D) To help the client tolerate the sitting position

Ans: C Feedback: The nurse places the client in Fowler's position for a few minutes before dangling to maintain safety should the client feel dizzy or faint due to postural hypotension. The nurse lowers the height of the bed so that the client can use the floor for support. The nurse provides the client with a robe and slippers to maintain warmth and show respect for the client's modesty. The nurse helps the client pivot a quarter of a turn to swing the legs over the side and sit on the edge of the bed, which helps the client adjust to a sitting position.

Chlorhexidine gluconate is being used in a client for preoperative skin preparation. Which of the following if noted would alert the nurse to a possible adverse reaction? A) Taste perversion B) Headache C) Deafness D) Mild erythema

Ans: C Feedback: The nurse should monitor for deafness as an adverse reaction to the use of chlorhexidine gluconate. Taste perversion, headache, and mild erythema are adverse reactions to penciclovir.

A nurse at a health care facility suggests the use of parallel bars for a client who has recently been fitted with a prosthetic limb. How would parallel bars most significantly help this client? A) By providing support if the client loses balance B) By improving the client's lower body strength C) By helping the client to practice ambulating D) By enabling the client to increase confidence

Ans: C Feedback: Using parallel bars as hand rails helps the client gain practice when ambulating. Some clients still need assistance to ambulate independently even after performing strengthening exercises. Support, confidence, and lower body strength are secondary considerations in the use of parallel bars, since the major aim is to help the client practice mobilizing.

A nurse is applying a leg prosthesis to a client at a health care facility. Why is it important for the nurse to observe the ease or difficulty of inserting the stump into the socket when applying the prosthesis? A) To determine the client's muscle strength B) To determine if prosthetic maintenance is required C) To check the number of stump socks that are required D) To determine if lubrication of the joints is required

Ans: C Feedback: When applying a leg prosthesis to a client at a health care facility, the nurse observes the ease or difficulty with which the client inserts the stump into the socket in order to determine the number or thickness of stump socks that would be required. The nurse inspects the stump for evidence of bleeding, edema, skin abrasions, and blisters in order to detect any complications that may delay healing and rehabilitation or interfere with ambulation. Joint connections in the prosthetic limbs need to be examined to determine if lubrication or prosthetic maintenance is required.

A nurse is reviewing topical drugs. The nurse demonstrates understanding of the information when the nurse identifies which drug as temporarily inhibiting the conduction of impulses from sensory nerve fibers? Select all that apply. A) Bacitracin (Baci-Rx) B) Papain and urea (Accuzyme) C) Lidocaine (Xylocaine) D) Dibucaine (Nupercainal) E) Anthralin (Miconal)

Ans: C, D Feedback: Topical local anesthetics, like lidocaine (Xylocaine) and dibucaine (Nupercainal), are examples of topical drugs that temporarily inhibit the conduction of impulses from sensory nerve fibers.

An elderly client is being seen at the clinic after experiencing a fall inside his home. He appears to have suffered no ill effects of the fall but his wife has asked the nurse he might benefit from the use of a cane. The nurse should be aware of what criterion for the safe use of a cane? A) The client's weakness must be bilaterally equal. B) The client must be able to support his full body weight on his arms and upper body. C) The client should first demonstrate that a walker does not provide sufficient support during ambulation. D) The client's weakness must be primarily limited to one side of his body.

Ans: D Feedback: A client who has weakness on one side of the body uses a cane. The client does not need to be able to support his or her full body weight with the upper body in order to use a cane safely and effectively. A walker provides more support than a cane.

A nurse is caring for a client whose legs have been amputated from above the knee due to a workplace accident. The client has been fitted with a prosthetic limb that attaches at his femur. Which of the following components can be found only in this prosthetic? A) Socket B) Shank C) Ankle/foot system D) Knee system

Ans: D Feedback: A knee system, which replaces the knee joint, is the component that can only be found in an above-the-knee prosthetic limb. Permanent prostheses for below-the-knee amputees and above-the-knee amputees include a socket, a shank, and an ankle/foot system.

A nurse is liaising with the physical therapist to help a frail, female client up from a dangling position to ambulate for the first time in several days. In preparation, the nurse has applied a walking belt around the client's waist while she dangles at the bedside. This device will assist in what way during the client's ambulation? A) It provides tactile feedback to the client if she exceeds her safe abilities. B) It can be loosely attached to the client's walker to provide support during rest periods. C) It provides support to the client's abdominal muscles to aid stability during ambulation. D) It provides a point where the nurse can support the client in case she loses balance.

Ans: D Feedback: A walking belt is applied around the client's waist. If the client loses balance, the nurse can support him or her and prevent injuries. It is not intended to provide muscular support or tactile feedback to the client. Attaching the belt to the walker would be unsafe and counterproductive.

A nurse at a health care facility has been assigned the care of an elderly client. For which of the following reasons would the nurse use a tilt table when preparing this client for ambulation? A) To promote the client's muscle tone and strength B) To improve the client's upper arm strength C) To normalize client's blood pressure prior to ambulating D) To help the client adjust to bearing weight on his or her feet

Ans: D Feedback: The nurse would use a tilt table to help the client bear weight on his or her feet. A tilt table is a device that raises the client from a supine to a standing position. It helps clients adjust to being upright and bearing weight on their feet. Dangling helps normalize a client's blood pressure, which may drop when the client rises from a reclining position. The client should perform isometric exercises to improve muscle tone and strength; isotonic exercise would help the client to improve upper arm strength.

A nurse is caring for a client who has been prescribed lidocaine viscous to be used for pain control of the oral mucosa. Which of the following instructions regarding the intake of food should the nurse give the client? A) Drink plenty of water along with food. B) Avoid intake of heavy and fibrous food. C) Ensure the food is not too hot or cold. D) Do not eat food for 1 hour after use.

Ans: D Feedback: When lidocaine viscous is used for oral anesthesia to control pain, the nurse instructs the client not to eat food for 1 hour after use because local anesthesia of the mouth or throat may impair swallowing and increase the possibility of aspiration. The nurse need not instruct the client to drink plenty of water along with food, avoid intake of heavy and fibrous food, or ensure the food is not too hot or cold as these are not relevant with oral anesthesia.


Conjuntos de estudio relacionados

EC364: Game Theory in Economics, Business & Finance

View Set

Chapter 14: Marketing Channels and Supply-Chain Management

View Set

Iggy Chapter 56: Care of Patients with Noninflammatory Intestinal Disorders

View Set

Organizational Behavior Midterm MC Questions

View Set

Fundamentals of Accounting (U of U)

View Set

Equations in One Variable Assignment

View Set

Ch 12: Management Support Systems

View Set